You are on page 1of 203

S GIO DC V O TO

H NI
CHNH THC

K THI TUYN SINH LP 10 THPT


Nm hoc: 2012 2013
Mn thi: Ton
Ngy thi: 21 thng 6 nm 2012
Thi gian lm bi: 120 pht

Bi I (2,5 im)
x 4
. Tnh gi tr ca A khi x = 36
x 2

x
4
x 16

:
2) Rt gn biu thc B
(vi x 0; x 16 )

x 4
x 2
x 4
3) Vi cc ca biu thc A v B ni trn, hy tm cc gi tr ca x nguyn gi tr ca biu thc B(A
1) l s nguyn
Bi II (2,0 im). Gii bi ton sau bng cch lp phng trnh hoc h phng trnh:
12
Hai ngi cng lm chung mt cng vic trong
gi th xong. Nu mi ngi lm mt mnh th
5
ngi th nht hon thnh cng vic trong t hn ngi th hai l 2 gi. Hi nu lm mt mnh th mi ngi
phi lm trong bao nhiu thi gian xong cng vic?
Bi III (1,5 im)
2 1
xy 2

1) Gii h phng trnh:


6 2 1
x y
2) Cho phng trnh: x2 (4m 1)x + 3m2 2m = 0 (n x). Tm m phng trnh c hai nghim
2
2
phn bit x1, x2 tha mn iu kin : x1 x 2 7
Bi IV (3,5 im)
Cho ng trn (O; R) c ng knh AB. Bn knh CO vung gc vi AB, M l mt im bt k
trn cung nh AC (M khc A, C); BM ct AC ti H. Gi K l hnh chiu ca H trn AB.
1) Chng minh CBKH l t gic ni tip.

2) Chng minh ACM


ACK
3) Trn an thng BM ly im E sao cho BE = AM. Chng minh tam gic ECM l tam gic vung
cn ti C
4) Gi d l tip tuyn ca (O) ti im A; cho P l im nm trn d sao cho hai im P, C nm trong
AP.MB
R . Chng minh ng thng PB i qua trung im ca on
cng mt na mt phng b AB v
MA
thng HK
Bi V (0,5 im). Vi x, y l cc s dng tha mn iu kin x 2y , tm gi tr nh nht ca biu thc:
x 2 y2
M
xy
1) Cho biu thc A

GI P N
Bi I: (2,5 im)
1) Vi x = 36, ta c : A =

36 4 10 5

36 2 8 4

2) Vi x , x 16 ta c :
x( x 4) 4( x 4)
x 2 (x 16)( x 2)
x2

B =
=

x 16
x 16 x 16
(x 16)(x 16) x 16

x2 x4
x2
2
2
.
1

3) Ta c: B( A 1)
.

x 16 x 2
x 16 x 2 x 16
B( A 1) nguyn, x nguyn th x 16 l c ca 2, m (2) = 1; 2
Ta c bng gi tr tng ng:
2
x 16 1
1
x
17
15
18
Kt hp K x 0, x 16 , B( A 1) nguyn th x 14; 15; 17; 18

2
14

Bi II: (2,0 im)


Gi thi gian ngi th nht hon thnh mt mnh xong cng vic l x (gi), K x

12
5

Th thi gian ngi th hai lm mt mnh xong cng vic l x + 2 (gi)


1
1
Mi gi ngi th nht lm c (cv), ngi th hai lm c
(cv)
x
x 2
12
12 5
V c hai ngi cng lm xong cng vic trong
gi nn mi gi c hai i lm c 1: = (cv)
5
5 12
Do ta c phng trnh
1
1
5

x x 2 12
x2 x 5

x( x 2) 12
5x2 14x 24 = 0
= 49 + 120 = 169, , 13
7 13 6
7 13 20

4 (TMK)
=> x
(loi) v x
5
5
5
5
Vy ngi th nht lm xong cng vic trong 4 gi,
ngi th hai lm xong cng vic trong 4+2 = 6 gi.
2 1
x y 2

Bi III: (1,5 im) 1)Gii h:


, (K: x, y 0).
6
2
1
x y
4 2

x y 4

H
6 2 1
x y

4 6
10
4 1
5
x 2
x 2
x x
x

.(TMK)

2 1
2 1
2 1
2 y 1

2
2 2 y
x y
x y
2

Vy h c nghim (x;y)=(2;1).
2) + Phng trnh cho c = (4m 1)2 12m2 + 8m = 4m2 + 1 > 0, m
Vy phng trnh c 2 nghim phn bit m
x1 x2 4m 1
+ Theo L Vi t, ta c:
.
2
x1x2 3m 2m
2
2
2
Khi : x1 x2 7 (x1 x2 ) 2x1x2 7
(4m 1)2 2(3m2 2m) = 7 10m2 4m 6 = 0 5m2 2m 3 = 0
3
Ta thy tng cc h s: a + b + c = 0 => m = 1 hay m =
.
5
Tr li: Vy....

Bi IV: (3,5 im)

M
H
E

1) Ta c HCB
900 ( do chn na ng trn k AB)

HKB
900 (do K l hnh chiu ca H trn AB)

=> HCB
HKB
1800 nn t gic CBKH ni tip trong ng trn ng knh HB.
2) Ta c ACM ABM (do cng chn
AM ca (O))

.ca trn k HB)


v ACK HCK
(v cng chn HK
HBK
Vy ACM ACK
900
3) V OC AB nn C l im chnh gia ca cung AB AC = BC v sd AC sd BC

Xt 2 tam gic MAC v EBC c

ca (O)
MA= EB(gt), AC = CB(cmt) v MAC
= MBC
v cng chn cung MC
MAC v EBC (cgc) CM = CE tam gic MCE cn ti C (1)

900 )
Ta li c CMB
450 (v chn cung CB

.
CEM
CMB
450 (tnh cht tam gic MCE cn ti C)

M CME
CEM
MCE
1800 (Tnh cht tng ba gc trong tam gic) MCE
900 (2)
T (1), (2) tam gic MCE l tam gic vung cn ti C (pcm).

C
M
H

N
A

4) Gi S l giao im ca BM v ng thng (d), N l giao im ca BP vi HK.


Xt PAM v OBM :
AP.MB
AP OB
R

Theo gi thit ta c
(v c R = OB).
MA
MA MB

Mt khc ta c PAM
ABM (v cng chn cung
AM ca (O))
PAM OBM
AP OB

1 PA PM .(do OB = OM = R) (3)
PM OM

V AMB
900 (do chn na trn(O)) AMS
900

tam gic AMS vung ti M. PAM


PSM
900

v PMA
PMS
900
PMS
PSM
PS PM (4)

M PM = PA(cmt) nn PAM PMA


T (3) v (4) PA = PS hay P l trung im ca AS.
NK BN HN
NK HN

V HK//AS (cng vung gc AB) nn theo L Ta-lt, ta c:


hay
PA BP PS
PA PS
m PA = PS(cmt) NK NH hay BP i qua trung im N ca HK. (pcm)
Bi V: (0,5 im)
Cch 1(khng s dng BT C Si)
x 2 y 2 ( x 2 4 xy 4 y 2 ) 4 xy 3 y 2 ( x 2 y ) 2 4 xy 3 y 2 ( x 2 y ) 2
3y

4
Ta c M =
=
xy
xy
xy
xy
x
V (x 2y)2 0, du = xy ra x = 2y
y 1
3 y 3

x 2y
, du = xy ra x = 2y
x 2
x
2
3 5
T ta c M 0 + 4 - = , du = xy ra x = 2y
2 2
5
Vy GTNN ca M l , t c khi x = 2y
2
4

Cch 2:
x2 y2 x2 y 2 x y
x y 3x

( )
xy
xy xy y x
4y x 4y
x y
x y
x y
; ta c
2
. 1,
V x, y > 0 , p dng bdt C si cho 2 s dng
4y x
4y x
4y x
du = xy ra x = 2y
x
3 x 6 3
V x 2y 2 . , du = xy ra x = 2y
y
4 y 4 2
3 5
T ta c M 1 + = , du = xy ra x = 2y
2 2
5
Vy GTNN ca M l , t c khi x = 2y
2
Cch 3:
Ta c M =

x2 y2 x2 y 2 x y
x 4 y 3y

( )
xy
xy xy y x
y x
x
x 4y
x 4y
x 4y
2 .
4,
V x, y > 0 , p dng bdt C si cho 2 s dng ;
ta c
y x
y x
y x
du = xy ra x = 2y
y 1
3 y 3

V x 2y
, du = xy ra x = 2y
x 2
x
2
3 5
T ta c M 4- = , du = xy ra x = 2y
2 2
5
Vy GTNN ca M l , t c khi x = 2y
2
Cch 4:
4x2
x2
3x 2 x 2
x2
y2
y2
y2
y2
2
2
2
3
x
3x
Ta c M = x y 4
4 4
4

xy
xy
xy
xy
4 xy
xy
4y
Ta c M =

V x, y > 0 , p dng bdt Co si cho 2 s dng

x2 2
x2
x2 2
; y ta c
y2 2
. y xy ,
4
4
4

du = xy ra x = 2y
x
3 x 6 3
V x 2y 2 . , du = xy ra x = 2y
y
4 y 4 2
xy 3
3 5
T ta c M
+ = 1+ = , du = xy ra x = 2y
xy 2
2 2
5
Vy GTNN ca M l , t c khi x = 2y
2

S GIO DC V O TO
CHNH THC

K THI TUYN SINH LP 10 THPT


TP.HCM Nm hoc: 2012 2013
MN: TON
Thi gian lm bi: 120 pht

Bi 1: (2 im)
Gii cc phng trnh v h phng trnh sau:
a) 2 x 2 x 3 0
2x 3y 7
b)
3x 2 y 4
c) x 4 x 2 12 0
d) x 2 2 2 x 7 0
Bi 2: (1,5 im)
1 2
1
x v ng thng (D): y x 2 trn cng mt h trc to .
4
2
b) Tm to cc giao im ca (P) v (D) cu trn bng php tnh.
Bi 3: (1,5 im)
Thu gn cc biu thc sau:
1
2 x
1
A

vi x > 0; x 1
x x x 1 x x
a) V th (P) ca hm s y

B (2 3) 26 15 3 (2 3) 26 15 3
Bi 4: (1,5 im)
Cho phng trnh x 2 2mx m 2 0 (x l n s)
a) Chng minh rng phng trnh lun lun c 2 nghim phn bit vi mi m.
b) Gi x1, x2 l cc nghim ca phng trnh.
24
Tm m biu thc M = 2
t gi tr nh nht
x1 x22 6 x1 x2
Bi 5: (3,5 im)
Cho ng trn (O) c tm O v im M nm ngoi ng trn (O). ng thng MO ct (O) ti E
v F (ME<MF). V ct tuyn MAB v tip tuyn MC ca (O) (C l tip im, A nm gia hai im M
v B, A v C nm khc pha i vi ng thng MO).
a)
Chng minh rng MA.MB = ME.MF
b)
Gi H l hnh chiu vung gc ca im C ln ng thng MO. Chng minh t gic AHOB
ni tip.
c)
Trn na mt phng b OM c cha im A, v na ng trn ng knh MF; na ng
trn ny ct tip tuyn ti E ca (O) K. Gi S l giao im ca hai ng thng CO v KF.
Chng minh rng ng thng MS vung gc vi ng thng KC.
d)
Gi P v Q ln lt l tm ng trn ngoi tip cc tam gic EFS v ABS v T l trung im
ca KS. Chng minh ba im P, Q, T thng hng.
BI GII
Bi 1: (2 im)
Gii cc phng trnh v h phng trnh sau:
a) 2 x 2 x 3 0 (a)
V phng trnh (a) c a - b + c = 0 nn
3
(a) x 1 hay x
2

2 x 3 y 7 (1)
2x 3y 7

3 x 2 y 4 (2)
x 5 y 3

b)

13 y 13
x 5 y 3
y 1

x2

(1)
(3) ((2) (1) )

((1) 2(3))
(3) ((2) (1) )

c) x 4 x 2 12 0 (C)
t u = x2 0, phng trnh thnh : u2 + u 12 = 0 (*)
1 7
1 7
3 hay u
4 (loi)
(*) c = 49 nn (*) u
2
2
Do , (C) x2 = 3 x = 3
Cch khc : (C) (x2 3)(x2 + 4) = 0 x2 = 3 x = 3
d) x 2 2 2 x 7 0 (d)
= 2 + 7 = 9 do (d) x = 2 3
Bi 2:
a) th:

Lu : (P) i qua O(0;0), 2;1 , 4; 4

(D) i qua 4; 4 , 2;1


b) PT honh giao im ca (P) v (D) l
1 2
1
x x 2 x2 + 2x 8 = 0 x 4 hay x 2
4
2
y(-4) = 4, y(2) = 1
Vy to giao im ca (P) v (D) l 4; 4 , 2;1 .
Bi 3:Thu gn cc biu thc sau:
1
2 x
1
x x x x 2 x
A

x x x 1 x x
x2 x
x 1

2 x 2 x 2 x 1

1
x( x 1) x 1 x 1 x

2 x ( x 1)
x( x 1)

2
vi x > 0; x 1
x

B (2 3) 26 15 3 (2 3) 26 15 3
7

1
1
(2 3) 52 30 3
(2 3) 52 30 3
2
2
1
1

(2 3) (3 3 5) 2
(2 3) (3 3 5) 2
2
2
1
1

(2 3)(3 3 5)
(2 3)(3 3 5) 2
2
2

Cu 4:
a/ Phng trnh (1) c = m2 - 4m +8 = (m - 2)2 +4 > 0 vi mi m nn phng trnh (1) c 2 nghim phn
bit vi mi m.
b
c
b/ Do , theo Viet, vi mi m, ta c: S = 2m ; P = m 2
a
a
24
24
6
2
M=
=
2
2
( x1 x2 ) 8 x1 x2 4m 8m 16 m 2m 4
6

. Khi m = 1 ta c (m 1) 2 3 nh nht
2
(m 1) 3
6
6
M
ln nht khi m = 1 M
nh nht khi m = 1
2
( m 1) 3
(m 1) 2 3
K
Vy M t gi tr nh nht l - 2 khi m = 1
T

Cu 5
B
a) V ta c do hai tam gic ng dng MAE v MBF
Q
MA MF
A
S
MA.MB = ME.MF

Nn
ME MB
(Phng tch ca M i vi ng trn tm O)
V
H
b) Do h thc lng trong ng trn ta c
M
O
F
E
MA.MB = MC2, mt khc h thc lng
trong tam gic vung MCO ta c
MH.MO = MC2 MA.MB = MH.MO
P
nn t gic AHOB ni tip trong ng trn.
C
c) Xt t gic MKSC ni tip trong ng
trn ng knh MS (c hai gc K v C vung).
Vy ta c : MK2 = ME.MF = MC2 nn MK = MC.
Do MF chnh l ng trung trc ca KC
nn MS vung gc vi KC ti V.
d) Do h thc lng trong ng trn ta c MA.MB = MV.MS ca ng trn tm Q.
Tng t vi ng trn tm P ta cng c MV.MS = ME.MF nn PQ vung gc vi MS v l ng trung
trc ca VS (ng ni hai tm ca hai ng trn). Nn PQ cng i qua trung im ca KS (do nh l trung
bnh ca tam gic SKV). Vy 3 im T, Q, P thng hng.

S GIO DC V O TO

K THI TUYN SINH LP 10 THPT


TP. NNG

Nm hoc:

2012 2013
CHNH THC

MN: TON
Thi gian lm bi: 120 pht

Bi 1: (2,0 im)
1) Gii phng trnh:(x + 1)(x + 2) = 0
2 x y 1
2) Gii h phng trnh:
x 2y 7
Bi 2: (1,0 im)
y
Rt gn biu thc A ( 10 2) 3 5
y=ax
Bi 3: (1,5 im)
Bit rng ng cong trong hnh v bn l mt parabol y = ax2.
1) Tm h s a.
2) Gi M v N l cc giao im ca ng thng
2
y = x + 4 vi parabol. Tm ta ca cc im M v N.
x
Bi 4: (2,0 im)
0
1 2
2
2
Cho phng trnh x 2x 3m = 0, vi m l tham s.
1) Gii phng trnh khi m = 1.
2) Tm tt c cc gi tr ca m phng trnh c hai nghim x1, x2 khc 0 v tha iu kin
x1 x2 8
.
x2 x1 3
Bi 5: (3,5 im)
Cho hai ng trn (O) v (O) tip xc ngoi ti A. K tip tuyn chung ngoi BC,B (O),C(O).
ng thng BO ct (O) ti im th hai l D.
1) Ch`ng minh rng t gic COOB l mt hnh thang vung.
2) Chng minh rng ba im A, C, D thng hng.
3) T D k tip tuyn DE vi ng trn (O) (E l tip im). Chng minh rng DB = DE.
2

BI GII
Bi 1:
1)
(x + 1)(x + 2) = 0 x + 1 = 0 hay x + 2 = 0 x = -1 hay x = -2
2 x y 1 (1)
5y 15 ((1) 2(2))
y 3
2)

x 2 y 7 (2)
x 7 2y
x 1
Bi 2: A ( 10 2) 3 5 = ( 5 1) 6 2 5 =
( 5 1) ( 5 1) 2 = ( 5 1)( 5 1) = 4
Bi 3:
1)
Theo th ta c y(2) = 2 2 = a.22 a =
1 2
2)
Phng trnh honh giao im ca y = x v ng thng y = x + 4 l :
2
1 2
x + 4 = x x2 2x 8 = 0 x = -2 hay x = 4
2
y(-2) = 2 ; y(4) = 8. Vy ta cc im M v N l (-2 ; 2) v (4 ; 8).
Bi 4:
1)
Khi m = 1, phng trnh thnh : x2 2x 3 = 0 x = -1 hay x = 3 (c dng ab + c = 0)
9

2)

x1 x2 8
3( x12 x22 ) 8 x1 x2 3(x1 + x2)(x1 x2) = 8x1x2
x2 x1 3
2
Ta c : a.c = -3m 0 nn 0, m
b
c
2
Khi 0 ta c : x1 + x2 = 2 v x1.x2 = 3m 0
a
a
iu kin phng trnh c 2 nghim 0 m m 0 > 0 v x1.x2 < 0 x1 < x2
Vi a = 1 x1 = b ' ' v x2 = b ' ' x1 x2 = 2 ' 2 1 3m 2
Vi x1, x2 0, ta c :

Do , ycbt 3(2)(2 1 3m 2 ) 8(3m 2 ) v m 0


1 3m 2 2m 2 (hin nhin m = 0 khng l nghim)
4m4 3m2 1 = 0 m2 = 1 hay m2 = -1/4 (loi) m = 1
Bi 5:
B
C

O
E

1)
2)
3)

Theo tnh cht ca tip tuyn ta c OB, OC vung gc vi BC t gic COOB l hnh thang
vung.
Ta c gc ABC = gc BDC gc ABC + gc BCA = 900 gc BAC = 900
Mt khc, ta c gc BAD = 900 (ni tip na ng trn)
Vy ta c gc DAC = 1800 nn 3 im D, A, C thng hng.
Theo h thc lng trong tam gic vung DBC ta c DB2 = DA.DC
Mt khc, theo h thc lng trong ng trn (chng minh bng tam gic ng dng) ta c DE 2 =
DA.DC DB = DE.

10

S GD&T
VNH PHC

K THI TUYN SINH LP 10 THPT NM HC 2012-2013


THI MN : TON
Thi gian lm bi 120 pht (khng k thi gian giao )
Ngy thi: 21 thng 6 nm 2012

CHNH THC

Cu 1 (2,0 im). Cho biu thc :P=

x
3
6x 4

2
x 1 x 1 x 1

1. Tm iu kin xc nh ca biu thc P.


2. Rt gn P
2 x ay 4
ax 3 y 5

Cu 2 (2,0 im). Cho h phng trnh :


1. Gii h phng trnh vi a=1

2. Tm a h phng trnh c nghim duy nht.


Cu 3 (2,0 im). Mt hnh ch nht c chiu rng bng mt na chiu di. Bit rng nu gim mi chiu i
2m th din tch hnh ch nht cho gim i mt na. Tnh chiu di hnh ch nht cho.
Cu 4 (3,0 im). Cho ng trn (O;R) (im O c nh, gi tr R khng i) v im M nm bn ngoi
(O). K hai tip tuyn MB, MC (B,C l cc tip im ) ca (O) v tia Mx nm gia hai tia MO v MC. Qua
B k ng thng song song vi Mx, ng thng ny ct (O) ti im th hai l A. V ng knh BB ca
(O). Qua O k ng thng vung gc vi BB,ng thng ny ct MC v BC ln lt ti K v E. Chng
minh rng:
1. 4 im M,B,O,C cng nm trn mt ng trn.
2. on thng ME = R.
3. Khi im M di ng m OM = 2R th im K di ng trn mt ng trn c nh, ch r tm v bn
knh ca ng trn .
Cu 5 (1,0 im). Cho a,b,c l cc s dng tha mn a+ b + c =4. Chng minh rng :
4

S GD&T VNH PHC


Cu
C1.1
(0,75
im)

K THI TUYN SINH LP 10 THPT NM HC 2012-2013


P N THI MN : TON
Ngy thi: 21 thng 6 nm 2012
p n, gi
im

x 1 0
Biu thc P xc nh

a 3 4 b3 4 c3 2 2

x 1 0
x2 1 0

0,5
0,25

11

x 1
x 1

C1.2
(1,25
im)

x
3
6x 4
x( x 1) 3( x 1) (6 x 4)

x 1 x 1 ( x 1)( x 1)
( x 1)( x 1)
2
2
x x 3x 3 6 x 4
x 2x 1

( x 1)( x 1)
( x 1)( x 1)

P=

( x 1) 2
x 1

( x 1)( x 1) x 1

C2.1
(1,0
im)

0,25
0,5
0,5

(voi x 1)

2 x y 4
Vi a = 1, h phng trnh c dng:
x 3y 5

6x3y 12 7x 7

x 3y 5 x 3y 5

0,25
0,25
0,25
0,25

x 1 x 1

13y 5 y 2

x 1
Vy vi a = 1, h phng trnh c nghim duy nht l:
y 2
C2.2
(1,0
im)

x 2
2x 4
-Nu a = 0, h c dng:
5 => c nghim duy nht
3y 5 y
3
2
a
-Nu a 0 , h c nghim duy nht khi v ch khi:
a 3
2
2
a 6 (lun ng, v a 0 vi mi a)
Do , vi a 0 , h lun c nghim duy nht.

0,25

0,25
0,25
0,25

Vy h phng trnh cho c nghim duy nht vi mi a.


12

C3 (2,0
im)

Gi chiu di ca hnh ch nht cho l x (m), vi x > 4.


V chiu rng bng na chiu di nn chiu rng l:
=> din tch hnh ch nht cho l: x.

x
x2

2
2

x
(m)
2

(m2)

Nu gim mi chiu i 2 m th chiu di, chiu rng ca hnh ch nht ln lt


l: x 2 va

x
2 (m)
2

0,25
0,25
0,25

khi , din tch hnh ch nht gim i mt na nn ta c phng trnh:


x
1 x2
( x 2)( 2)
2
2 2
x2
x2

2x x 4
x 2 12 x 16 0
2
4
.=> x1 6 2 5 (tho mn x>4);
x 2 6 2 5 (loi v khng tho mn x>4)

C4.1
(1,0
im)

C4.2
(1,0
im)

C4.3
(1,0
im)

Vy chiu di ca hnh ch nht cho l 6 2 5 (m).


B
1) Chng minh M, B, O, C cng thuc 1 ng trn
Ta c: MOB 90 0 (v MB l tip tuyn)
MCO 90 0 (v MC l tip tuyn)
O
1
M
=> MBO + MCO =
2
1
= 900 + 900 = 1800
K
=> T gic MBOC ni tip
1
E
B
(v c tng 2 gc i =1800)
C
=>4 im M, B, O, C cng thuc 1 ng trn
2) Chng minh ME = R:
Ta c MB//EO (v cng vung gc vi BB)
=> O1 = M1 (so le trong)
M M1 = M2 (tnh cht 2 tip tuyn ct nhau) => M2 = O1 (1)
C/m c MO//EB (v cng vung gc vi BC)
=> O1 = E1 (so le trong) (2)
T (1), (2) => M2 = E1 => MOCE ni tip
=> MEO = MCO = 900
=> MEO = MBO = BOE = 900 => MBOE l hnh ch nht
=> ME = OB = R (iu phi chng minh)
3) Chng minh khi OM=2R th K di ng trn 1 ng trn c nh:
Chng minh c Tam gic MBC u => BMC = 600
=> BOC = 1200
=> KOC = 600 - O1 = 600 - M1 = 600 300 = 300
Trong tam gic KOC vung ti C, ta c:
CosKOC

OC
OC
3 2 3R
OK
R:

OK
2
3
Cos30 0

M O c nh, R khng i => K di ng trn ng trn tm O, bn knh =


2 3R
(iu phi chng minh)
3

0,25
0,25
0,5
0,25

0,25
0,25
0,25
0,25

0,25
0,25
0,25
0,25
0,25
0,25
0,25
0,25

13

C5 (1,0
im)

4a 3 4 4b3 4 4c 3
4

a b c a 3 4 a b c b3 4 a b c c 3

4 a 4 4 b4 4 c 4
abc
4
4

0,25
0,25

4
4

2 2
4
4
2
Ch : -Cu 4, tha gi thit tia Mx v im A gy ri.
-Mi cu u c cc cch lm khc
cu 5
Cach 2: t x = 4 a;y 4 b;z 4 c => x, y , z > 0 v x4 + y4 + z4 = 4.
Do ,

0,25

a 3 4 b3 4 c3

0,25

BT cn CM tng ng: x3 + y3 + z3 > 2 2


hay 2 (x3 + y3 + z3 ) > 4 = x4 + y4 + z4
x3( 2 -x) + y3( 2 -y)+ z3( 2 -z) > 0 (*).
Ta xt 2 trng hp:
- Nu trong 3 s x, y, z tn ti it nht mt s 2 , gi s x 2 th x3 2 2 .
Khi o: x3 + y3 + z3 > 2 2 ( do y, z > 0).
- Nu c 3 s x, y, z u nh 2 th BT(*) lun ung.
Vy x + y3 + z3 > 2 2 c CM.
Cach 3: C th dng BT thc Csi kt hp phng php lm tri v nh gi cng cho kt qu nhng hi
di, phc tp).
3

14

S GD V O TO
KLK
CHNH THC

K THI TUYN SINH VO 10 THPT NM HC 2012-2013


MN THI : TON
Thi gian lm bi: 120 pht,(khng k giao )
Ngy thi: 22/06/2012

Cu 1. (2,5)
1) Gii phng trnh:
a) 2x2 7x + 3 = 0.
b) 9x4 + 5x2 4 = 0.
2) Tm hm s y = ax + b, bit th hm s ca n i qua 2 im A(2;5) ; B(-2;-3).
Cu 2. (1,5)
1) Hai t i t A n B di 200km. Bit vn tc xe th nht nhanh hn vn tc xe th hai l 10km/h
nn xe th nht n B sm hn xe th hai 1 gi. Tnh vn tc mi xe.

1
2) Rt gn biu thc: A= 1
x x ; vi x 0.
x 1

Cu 3. (1,5 )
Cho phng trnh: x2 2(m+2)x + m2 + 4m +3 = 0.
1) Chng minh rng : Phng trnh trn lun c hai nghim phn bit x1, x2 vi mi gi tr ca m.
2) Tm gi tr ca m biu thc A = x12 x22 t gi tr nh nht.
Cu 4. (3,5)
Cho tam gic ABC c ba gc nhn ni tip ng trn tm O (AB < AC). Hai tip tuyn ti B v C ct nhau
ti M. AM ct ng trn (O) ti im th hai D. E l trung im on AD. EC ct ng trn (O) ti im
th hai F. Chng minh rng:
1) T gic OEBM ni tip.
2) MB2 = MA.MD.

3) BFC
.
MOC
4) BF // AM
Cu 5. (1)
1 2
Cho hai s dng x, y tha mn: x + 2y = 3. Chng minh rng: 3
x y
Bi gii s lc:
Cu 1. (2,5)
1) Gii phng trnh:
a) 2x2 7x + 3 = 0.
2
= (-7) 4.2.3 = 25 > 0
7 5
x1
3.
4
= 5. Phng trnh c hai nghim phn bit:
7 5 1
x2

4
2
4
2
2
b) 9x + 5x 4 = 0. t x = t , k : t 0.
Ta c pt: 9t2 + 5t 4 = 0.
a b + c = 0 t1 = - 1 (khng TMK, loi)
4
t2 = (TMK)
9
4
4
x2 = x = 4 2 .
t2 =
9
9
9
3

15

Vy phng trnh cho c hai nghim: x1,2 =

2
3
2a b 5
a 2

2a b 3 b 1

2) th hm s y = ax + b i qua hai im A(2;5) v B(-2;-3)


Vy hm s cn tm l : y = 2x + 1
Cu 2.
1) Gi vn tc xe th hai l x (km/h). k: x > 0
Vn tc xe th nht l x + 10 (km/h)
200
(gi)
x 10
200
Thi gian xe th hai i qung ng t A n B l :
(gi)
x
Thi gian xe th nht i qung ng t A n B l :

Xe th nht n B sm 1 gi so vi xe th hai nn ta c phng trnh:

200 200

1
x x 10

Gii phng trnh ta c x1 = 40 , x2 = -50 ( loi)


x1 = 40 (TMK). Vy vn tc xe th nht l 50km/h, vn tc xe th hai l 40km/h.
x 1 1

1
x x
2) Rt gn biu thc: A 1
x x
x 1
x 1

x
x x 1 = x, vi x 0.
=

Cu 3. (1,5 )
Cho phng trnh: x2 2(m+2)x + m2 + 4m +3 = 0.
1) Chng minh rng : Phng trnh trn lun c hai nghim phn bit x1, x2 vi mi gi tr ca m.

Ta c (m 2) m2 4m 3 1> 0 vi mi m.
Vy phng trnh cho lun c hai nghim phn bit x1, x2 vi mi gi tr ca m.
2) phng trnh cho lun c hai nghim phn bit x1, x2 vi mi gi tr ca m. Theo h thc Vi-t ta c
x1 x2 2(m 2)
:
2
x1.x2 m 4m 3
A = x12 x22 = (x1 + x2)2 2 x1x2 = 4(m + 2)2 2(m2 + 4m +3) = 2m2 + 8m+ 10
= 2(m2 + 4m) + 10
= 2(m + 2)2 + 2 2 vi mi m.
Suy ra minA = 2 m + 2 = 0 m = - 2
A
Vy vi m = - 2 th A t min = 2
Cu 4.
1) Ta c EA = ED (gt) OE AD ( Quan h gia ng knh v dy)
O
C
E

OEM
= 900; OBM
= 900 (Tnh cht tip tuyn)
F
E v B cng nhn OM di mt gc vung T gic OEBM ni tip.
B
1

2) Ta c MBD
( gc ni tip chn cung BD)
s BD
D
2
1

( gc to bi tia tip tuyn v dy cung chn cung BD)


MAB
s BD
2
. Xt tam gic MBD v tam gic MAB c:

MBD
MAB
M

16

MB MD

MBD ng dng vi MAB


Gc M chung, MBD

MAB
MA MB
MB2 = MA.MD
1
1
1

( Tnh cht hai tip tuyn ct nhau); BFC


3) Ta c: MOC
= s BC
(gc ni
BOC
s BC
2
2
2

tip) BFC
.
MOC
$ C
= 1800) MFC

4) T gic MFOC ni tip ( F


( hai gc ni tip cng chn cung MC), mt
MOC

BF // AM.
khc MOC
(theo cu 3) BFC
BFC
MFC

a 2 b2 a b
Cu 5.

x
y
x y
Ta c x + 2y = 3 x = 3 2y , v x dng nn 3 2y > 0
1 2
1
2
y 6 4y 3y(3 2y) 6(y 1)2
3

Xt hiu 3 =
0 ( v y > 0 v 3 2y > 0)
x y
3 2y y
y(3 2y)
y(3 2y)
2

x 0,y 0 x 0,y 0
x1
1 1

3 du = xy ra x 3 2y x 1

x 2y
y1
y 1 0
y1

17

S GIO DC VO O
TO HI DNG
---------------- CHNH THC

K THI TUYN SINH LP 10 THPT


NM HC 2012-2013
MN THI: TON
Thi gian lm bi 120 pht (khng k thi gian giao )
Ngy thi: Ngy 12 thng 7 nm 2012
( thi gm: 01 trang)

Cu 1 (2,0 im):
Gii cc phng trnh sau:
a) x(x-2)=12-x.
b)

x2 8
1
1

2
x 16 x 4 x 4

Cu 2 (2,0 im):

3 x y 2m 9
c nghim (x;y). Tm m biu thc (xy+x-1) t gii
x y 5

a) Cho h phng trnh


tr ln nht.

b) Tm m ng thng y = (2m-3)x-3 ct trc honh ti im c honh bng

2
.
3

Cu 3 (2,0 im):

3
1

.
x 1
x x 2

a) Rt gn biu thc P

x 2 vi x 0 v x 4 .

b) Nm ngoi, hai n v sn xut nng nghip thu hoch c 600 tn thc. Nm nay, n v
th nht lm vt mc 10%, n v th hai lm vt mc 20% so vi nm ngoi. Do c
hai n v thu hoch c 685 tn thc. Hi nm ngoi, mi n v thu hoch c bao nhiu
tn thc?
Cu 4 (3,0 im):
Cho tam gic ABC c ba gc nhn, ni tip ng trn (O). V cc ng cao BE, CF ca
tam gic y. Gi H l giao im ca BE v CF. K ng knh BK ca (O) .
a) Chng minh t gic BCEF l t gic ni tip.
b) Chng minh t gic AHCK l mnh bnh hnh.
c) ng trn ng knh AC ct BE M, ng trn ng knh AB ct CF N. Chng minh
AM = AN.
Cu 5 (1,0 im):
Cho a, b, c, d l cc s thc tha mn: b + d 0 v

ac
2 . Chng minh rng phng trnh
bd

(x2 + ax +b)(x2 + cx + d)=0 (x l n) lun c nghim.


---------------------Ht--------------------

18

Cu 1: a ) x = - 3 v x = 4.

HNG DN - P N
b) x = - 2; loi x = 4.

Cu 2: a) H => x = m + 2 v y = 3 - m => A = (xy+x-1) = = 8 - ( m -1)2


Amax = 8 khi m = 1.
b) Thay x = 2/3 v y = 0 vo pt ng thng => m = 15/4
Cu 3: a) A = 1
b) x + y = 600 v 0,1x + 0,2y = 85 hay x + 2y = 850.
T tnh c y = 250 tn, x = 350 tn
Cu 4 (3,0 im):
a) BFC BE C 90 0
b) AH//KC ( cng vung gc vi BC)
CH // KA ( cng vung gc vi AB)
c) C AN2 = AF.AB; AM2 = AE.AC
( H thc lng trong tam gic vung)
AEF : ABC

AM = AN

AE AF

AE. AC AF.AB
AB AC

Cu 5 (1,0 im)

Xt 2 phng trnh:
x2 + ax + b = 0

(1) v

x2 + cx + d = 0 (2)

1 2 (a 2 4b) (c 2 4d ) a 2 2ac c 2 2 ac 2(b d ) (a c) 2 2 ac 2(b d )

+ Vi b+d <0 b; d c t nht mt s nh hn 0

1 >0 hoc 2 >0 pt cho c nghim


ac
2 ac > 2(b + d) => 1 2 0
+ Vi b d 0 . T
bd
=> t nht mt trong hai biu gi tr 1 , 2 0 => t nht mt trong hai pt (1) v (2) c

nghim.
Vy vi a, b, c, d l cc s thc tha mn: b + d 0 v

ac
2,
bd

phng trnh (x2 + ax +b)(x2 + cx + d)=0 (x l n) lun c nghim.

19

S GIO DC VO O
TO HI DNG
---------------- CHNH THC

K THI TUYN SINH LP 10 THPT


NM HC 2012-2013
MN THI: TON
Thi gian lm bi 120 pht (khng k thi gian giao )
Ngy thi: Ngy 14 thng 7 nm 2012
( thi gm: 01 trang)

Cu 1 (2,0 im): Gii cc phng trnh sau:


2
4
x
3 0
a) x 5
3
5
b) | 2x 3 | = 1.

Cu 2 (2,0 im): Cho biu thc:

a
a
a
a

A=
:
vi a v b l cc s dng khc nhau.
a b a b 2 ab
a b b a

a b 2 ab
a) Rt gn biu thc A
.
ba
b) Tnh gi tr ca A khi a = 7 4 3 v b = 7 4 3 .
Cu 3 (2,0 im):
a) Tm m cc ng thng y = 2x + m v y = x 2m + 3 ct nhau ti mt im nm
trn trc tung.
b) Cho qung ng t a im A ti a im B di 90 km. Lc 6 gi mt xe my i t
A ti B Lc 6 gi 30 pht cng ngy, mt t cng i t A ti B vi vn tc ln hn vn tc xe
my 15 km/h (Hai xe chy trn cng mt con ng cho). Hai xe ni trn u n B cng lc.
Tnh vn tc mi xe.
Cu 4 (3,0 im): Cho na ng trn tm O ng knh AB = 2R (R l mt di cho trc). Gi C, D

l hai im trn na ng trn sao cho C thuc cung AD v COD


= 1200 . Gi giao im ca hai dy
AD v BC l E, giao im ca cc ng thng AC v BD l F.
a) Chng minh rng bn im C, D, E, F cng nm trn mt ng trn.
b) Tnh bn knh ca ng trn i qua C, E, D, F ni trn theo R.
c) Tm gi tr ln nht ca in tch tam gic FAB theo R khi C, D thay i nhung vn tha mn gi
thit bi ton
Cu 5 (1,0 im): Khng dng my tnh cm tay , tm s nguyn ln nht khng vt qu S, trong
S=

2 3

-------------------- Ht -------------------HNG DN GII .


Cu 1.

20

2
4
x 5
x
3 0
3
5
15
2

0
x

2 x 15
2

4 x 3 0 4 x 15 x 15
5

4
a)

2x 3 1
2x 4 x 2
15 15

;
} b) 2 x 3 1

2 4
2 x 3 1 2 x 2 x 1
Vy nghim ca phng trnh cho l S = {1;2}
Cu 2 .
Ta c :

a
a
a
a
A

a b a b 2 ab

a b b a


a
a
a
a

a b ( b a )( b a ) a b
a b

Vy nghim ca phng trnh cho l S = {

a( b a) a
a( a b) a
:
2
( b a )( b a )
a b

ab
( b a )( b a )

a b

ab

a b
b a
a) Ta c :
a b 2 ab
A
ba
a b ( a b )2

ba
b a
A

( a b ) 2 ( a b )2

0
ba
a b 2 ab
Vy A
=0
ba
b) Ta c :
a 74 3
a 44 3 3

a 2 3

a 2 3

21

b 74 3
b 44 3 3

b 2 3

b 2 3
Thay

a 2 3; b 2 3 vo biu thc A

a b
ta c :
b a

2 32 3
2 32 3
4
2 3
A
Vy vi a = 7 - 4 3 ; b = 7 + 4 3 th A =
.
2 3
3
A

2 3
3

Cu 3 .
a) hai ng thng y = 2x + m v y = x 2m + 3 ct nhau ti mt im trn trc tung th m = -2m + 3 =>
3m = 3 => m = 1.
Vy vi m = 1 th hai ng thng y = 2x + m v y = x 2m + 3 ct nhau ti mt im trn trc tung.
1
b) Xe my i trc t thi gian l : 6 gi 30 pht - 6 gi = 30 pht = h .
2
Gi vn tc ca xe my l x ( km/h ) ( x > 0 )
V vn tc t ln hn vn tc xe my 15 km/h nn vn tc ca t l x + 15 (km/h)
90
( h)
Thi gian xe my i ht qung ng AB l :
x
90
( h)
Thi gian t i ht qung ng AB l :
x 15
1
Do xe my i trc t
gi v hai xe u ti B cng mt lc nn ta c phng trnh :
2
90 1
90

x 2 x 15
90.2.( x 15) x ( x 15) 90.2 x
180 x 2700 x 2 15 x 180 x
x 2 15 x 2700 0
Ta c :
152 4.(2700) 11025 0
11025 105
15 105
x1
60 ( khng tha mn iu kin )
2
15 105
x2
45 ( tha mn iu kin )
2
Vy vn tc ca xe my l 45 ( km/h ) , vn tc ca t l 45 + 15 = 60 ( km/h ).
Cu 4.
a) Ta c : C, D thuc ng trn nn :
22

ACB ADB 900 ( gc ni tip chn na ng trn )

=> FCE
900 ; FDE
900 ( gc k b )
Hai im C v D cng nhn on thng FE di mt gc bng nhau
bng 900 nn 4 im C,D,E,F cng thuc ng trn ng knh EF.
b) Gi I l trung im EF th ID = IC l bn knh ng trn i qua
4 im C, D, E, F ni trn.
Ta c : IC = ID ; OC = OD ( bn knh ng trn tm O )

suy ra IO l trung trc ca CD => OI l phn gic ca COD


1200

=> IOD

600
2
Do O l trung im AB v tam gic ADB vung ti D nn tam gic ODB cn ti O

=> ODB
(1)
OBD

Do ID = IF nn tam gic IFD cn ti I => IFD


(2)
IDF
Tam gic AFB c hai ng cao AD, BC ct nhau ti E nn E l trc tm tam gic => FE l ng cao th

D 900 (3)
ba => FE vung gc AB ti H => OBD
IF

T (1) , (2) , (3) suy ra IDF


ODB
900 => IDO
900 .

Xt tam gic vung IDO c IOD


600 .

Ta c : ID = OD.tan IOD
= R.tan600 = R 3 .
Vy bn knh ng trn i qua 4 im C,D,E,F l R 3 .
c) Theo phn b) : OI =

ID 2 OD 2 3R 2 R 2 2 R .

t OH = x th 0 x R => IH =

4R 2 x 2 .

=> FH = R 3 + 4R 2 x 2 .
1
1
S FAB . AB.FH .2 R.( R 3 4 R 2 x 2 )
2
2
S FAB R 2 3 R 4 R 2 x 2
Ta c : 4R2 - x2 4R2 . Du bng xy ra khi x = 0.

Khi : SFAB = R2 3 + 2R2 v H O => O, I, F thng hng => CD // AB => ADO DAO
150 => BD =
0
AC = 2RSin15 .
Vy din tch ln nht t c ca tam gic AFB l R2 3 + 2R2 khi AC = BD = 2Rsin150 .
Cu 5
Xt hai s a = 2 + 3 v b = 2 - 3 .
Ta c : a + b = 4 v ab = 1, 0< b < 1.
(a+b)3 = 43 = 64 => a3 + b3 = 64 - 3ab(a + b) = 64 - 3.1.4 = 52
(a3+b3)(a3 + b3) = 52.52 => a6 + b6 = 2704 - 2(ab)3 = 2704 - 2 = 2702
=> a6 = S = 2702 - b6 (*).
Do 0<b<1 nn 0 < b6 < 1
Kt hp (*) th s nguyn ln nht khng vt qu S l 2701.

23

S GIO DC V O TO
HI DNG
CHNH THC

K THI TUYN SINH LP 10 THPT CHUYN


NGUYN TRI NM HC 2012- 2013
Mn thi: TON (khng chuyn)
Thi gian lm bi: 120 pht
Ngy thi 19 thng 6 nm 2012
thi gm : 01 trang

Cu I (2,0 im)

x 1
x 1 .
3
x 3 3 3 0
2) Gii h phng trnh
.
3 x 2 y 11
Cu II ( 1,0 im)
1
1
a +1

+
Rt gn biu thc P =
:
2 - a a - 2 a
2 a -a
Cu III (1,0 im)
1) Gii phng trnh

vi a > 0 v a 4 .

Mt tam gic vung c chu vi l 30 cm, di hai cnh gc vung hn km nhau 7cm. Tnh di
cc cnh ca tam gic vung .
Cu IV (2,0 im)
1 2
Trong mt phng ta Oxy, cho ng thng (d): y = 2x - m +1 v parabol (P): y = x .
2
1) Tm m ng thng (d) i qua im A(-1; 3).
2) Tm m (d) ct (P) ti hai im phn bit c ta (x1; y1) v (x2; y2) sao cho
x1x 2 y1 + y 2 48 0 .
Cu V (3,0 im)
Cho ng trn tm O ng knh AB. Trn ng trn ly im C sao cho AC < BC (C A). Cc
tip tuyn ti B v C ca (O) ct nhau im D, AD ct (O) ti E (E A) .
1) Chng minh BE2 = AE.DE.
2) Qua C k ng thng song song vi BD ct AB ti H, DO ct BC ti F. Chng minh t gic
CHOF ni tip .
3) Gi I l giao im ca AD v CH. Chng minh I l trung im ca CH.
Cu VI ( 1,0 im)
1 1
Cho 2 s dng a, b tha mn 2 . Tm gi tr ln nht ca biu thc
a b
1
1
Q 4
4
.
2
2
2
a b 2ab b a 2ba 2

24

S GIO DC V O TO
TUYN QUANG

THI TUYN SINH VO LP 10 THPT


Nm hoc 2011 - 2012
MN THI: TON
Thi gian: 120 pht (khng k thi gian giao )

CHNH THC

Cu 1 (3,0 im)
a) Gii phng trnh:

x2 6x 9 0

b) Gii h phng trnh:

4x 3 y 6

3 y 4 x 10

c) Gii phng trnh:

x 2 6 x 9 x 2011

Cu 2 (2,5 im)
Mt ca n chy xui dng t A n B ri chy ngc dng t B n A ht tt c 4 gi. Tnh vn tc ca
n khi nc yn lng, bit rng qung sng AB di 30 km v vn tc dng nc l 4 km/gi.
Cu 3 (2,5 im)
Trn ng trn (O) ly hai im M, N sao cho M, O, N khng thng hng. Hai tip tuyn ti M , N vi
ng trn (O) ct nhau ti A. T O k ng vung gc vi OM ct AN ti S. T A k ng vung
gc vi AM ct ON ti I. Chng minh:
a) SO = SA
b) Tam gic OIA cn
Cu 4 (2,0 im).
a) Tm nghim nguyn ca phng trnh: x2 + 2y2 + 2xy + 3y 4 = 0
b) Cho tam gic ABC vung ti A. Gi I l giao im cc ng phn gic trong. Bit AB = 5 cm,
IC = 6 cm. Tnh BC.
Hng dn chm, biu im
MN THI: TON CHUNG
Ni dung

im

Cu 1 (3,0 im)
a) Gii phng trnh:

x2 6x 9 0

1,0

Bi gii: Ta c ' (3) 2 9 0


6
3
Phng trnh c nghim: x
2
b) Gii h phng trnh:

4x 3 y 6

3 y 4 x 10

0,5
0,5

(1)
(2)

1,0

25

Bi gii: Cng (1) v (2) ta c: 4x - 3y + 3y + 4x = 16 8x = 16 x = 2


x2
2

Thay x = 2 vo (1): 4. 2 3y = 6 y = . Tp nghim:


2
3
y 3
c) Gii phng trnh:
Bi gii: Ta c
Mt khc:

0,5
0,5

1,0

x 2 6 x 9 x 2011 (3)

x2 6x 9

x 3

x 3

0,5

x 2 6 x 9 0 x 2011 0 x 2011 x 3 x 3

0,5

Vy: (3) x 3 x 2011 3 2011 . Phng trnh v nghim


Cu 2 (2,5 im )
Bi gii: Gi vn tc ca ca n khi nc yn lng l x km/gi ( x > 4)

2,5
0,5

Vn tc ca ca n khi xui dng l x +4 (km/gi), khi ngc dng l x - 4 (km/gi). Thi gian
30
ca n xui dng t A n B l
gi, i ngc dng
0,5
x4
30
t B n A l
gi.
x4
30
30

4
Theo bi ra ta c phng trnh:
(4)
0,5
x4 x4
(4) 30( x 4) 30( x 4) 4( x 4)( x 4) x 2 15 x 16 0 x 1
hoc x = 16. Nghim x = -1 <0 nn b loi

0,5

Vy vn tc ca ca n khi nc yn lng l 16km/gi.

0,5

Cu 3 (2,5 im)
A

S
I

0,5

1,0

a) Chng minh: SA = SO

V AM, AN l cc tip tuyn nn: MAO

(1)

(2)
SOA
SAO cn SA = SO (.p.c.m)
T (1) v (2) ta c: SAO
V MA//SO nn:

SAO

MAO SOA (so le trong)

b) Chng minh tam gic OIA cn

0,5

0,5
1,0

26

V AM, AN l cc tip tuyn nn: MOA


(3)
NOA
V MO // AI nn: gc MOA bng gc OAI (so le trong) (4)

0,5

0,5

T (3) v (4) ta c: IOA IAO OIA cn (.p.c.m)


Cu 4 (2,0 im).
a) Tm nghim nguyn ca phng trnh: x2 + 2y2 + 2xy + 3y 4 = 0 (1)

1,0

Bi gii: (1) (x2 + 2xy + y2) + (y2 + 3y 4) = 0


(x + y)2 + (y - 1)(y + 4) = 0

0,5

(y - 1)(y + 4) = - (x + y)2 (2)


V - (x + y)2 0 vi mi x, y nn: (y - 1)(y + 4) 0 -4 y 1
V y nguyn nn y 4; 3; 2; 1; 0; 1

0,5

Thay cc gi tr nguyn ca y vo (2) ta tm c cc cp nghim nguyn (x; y) ca PT cho


l: (4; -4), (1; -3), (5; -3), ( -2; 0), (-1; 1).
b) Cho tam gic ABC vung ti A. Goi I l giao im cc ng phn gic trong. Bit AB
= 5 cm, IC = 6 cm. Tnh BC.
Bi gii:
Gi D l hnh chiu vung gc ca C
trn ng thng BI, E l giao im ca

AB v CD. BIC c DIC


l gc ngoi
=
nn:
DIC
ICB

IBC

1
2

$ C) 90 : 2 45
(B
0

0,5

DIC vung cn DC = 6 : 2
Mt khc BD l ng phn gic v
ng cao nn tam gic BEC cn ti B
EC = 2 DC = 12: 2 v BC = BE

I
6

Gi x = BC = BE. (x > 0). p dng nh l Pi-ta-go vo cc tam gic vung ABC v ACE ta
c: AC2 = BC2 AB2 = x2 52= x2 -25
EC2 = AC2 + AE2 = x2 -25 + (x 5)2 = 2x2 10x
(12: 2 )2 = 2x2 10x
x2 - 5x 36 = 0
Gii phng trnh ta c nghim x = 9 tho mn. Vy BC = 9 (cm)

O,5

27

CHNH THC

28

29

30

31

32

33

34

S GIO DC V O TO
THANH HA

K THI TUYN SINH LP 10 THPT


NM HC 2012-2013
Mn thi : Ton

THI CHNH THC


A

Thi gian : 120 pht khng k thi gian giao

Ngy thi 29 thng 6 nm 2012

Bi 1: (2.0 im)
1- Gii cc phng trnh sau :
a) x - 1 = 0 .
b) x2 - 3x + 2 = 0
2- Gii h phng trnh :

2x y 7

x y 2

Bi 2: (2.0 im) Cho biu thc : A =

1
22 a

1
22 a

1- Tm iu kin xc nh v rt gn biu thc A


2- Tm gi tr ca a ; bit A <

a2 1
1 a2

1
3

Bi 3: (2.0 im)
1- Cho ng thng (d) : y = ax + b .Tm a; b ng thng (d) i qua im A( -1 ;
3) v song song vi ng thng (d) : y = 5x + 3
2- Cho phng trnh ax2 + 3(a + 1)x + 2a + 4 = 0 ( x l n s ) .Tm a phmg trnh
cho c hai nghim phn bit x1 ; x2 tho mn x12 + x 22 = 4
Bi 4: (3.0 im) Cho tam tam gic u ABC c ng cao AH . Trn cnh BC ly im M
bt k ( M khng trng B ; C; H ) T M k MP ; MQ ln lt vung gc vi cc cnh AB ;
AC ( P thuc AB ; Q thuc AC)
1- Chng minh :T gic APMQ ni tip ng trn
2- Gi O l tm ng trn ngoi tip t gic APMQ .Chng minh OH PQ
3- Chng minh rng : MP +MQ = AH
Bi 5: (1.0 im) Cho hai s thc a; b thay i , tho mn iu kin a + b 1 v a > 0
Tm gi tr nh nht ca biu thc A =

8a 2 b
b2
4a

---------------------------------------HT ----------------------------------

35

36

37

38

39

40

S GIO DC O TO
THANH HO
CHNH THC

K THI VO LP 10 CHUYN LAM SN


NM HC 2012 - 2013
Mn thi : TON

( gm c 01 trang)

(Mn chung cho tt cc th sinh)


Thi gian lm bi :120 pht (Khng k thi gian giao
)
Ngy thi : 17 thng 6 nm 2012

Cu 1: (2.0 im ) Cho biu thc :


a 1

a 1
1
P

4 a
2a a , (Vi a > 0 , a 1)
a 1
a 1

1. Chng minh rng : P

2
a 1

2. Tm gi tr ca a P = a
Cu 2 (2,0 im ) : Trong mt phng to Oxy, cho Parabol (P) : y = x2 v ng thng (d) : y = 2x + 3
1. Chng minh rng (d) v (P) c hai im chung phn bit
2. Gi A v B l cc im chung ca (d) v (P) . Tnh din tch tam gic OAB ( O l gc to )
Cu 3 (2.0 im) : Cho phng trnh : x2 + 2mx + m2 2m + 4 = 0
1. Gii phng trnh khi m = 4
2. Tm m phng trnh c hai nghim phn bit
Cu 4 (3.0 im) : Cho ng trn (O) c ng knh AB c nh, M l mt im thuc (O) ( M khc A v B )
. Cc tip tuyn ca (O) ti A v M ct nhau C. ng trn (I) i qua M v tip xc vi ng thng AC ti
C. CD l ng knh ca (I). Chng minh rng:
1. Ba im O, M, D thng hng
2. Tam gic COD l tam gic cn
3. ng thng i qua D v vung gc vi BC lun i qua mt im c nh khi M di ng trn ng
trn (O)
Cu 5 (1.0 im) : Cho a,b,c l cc s dng khng m tho mn : a 2 b 2 c 2 3
Chng minh rng :

a
b
c
1
2
2

a 2b 3 b 2c 3 c 2a 3 2
2

41

HNG DN GII:
CU

NI DUNG
2
a 1

a 1
a 1
1

4 a

a 1
a 1
2a a

IM

1. Chng minh rng : P

P
P
1

a 1

a 1 4 a

a 1

a 1

a 1

a 2 a 1 a 2 a 1 4a a 4 a

a 1

a 1

a 1

1
2a a

1.0

1
2a a

4a a
1
2
.

(PCM)
a 1 2a a a 1

2. Tm gi tr ca a P = a. P = a
2
a a 2 a 2 0
a

1
=>
.
Ta c 1 + 1 + (-2) = 0, nn phng trnh c 2 nghim
a1 = -1 < 0 (khng tho mn iu kin) - Loi
c 2
2
a2 = a 1
(Tho mn iu kin)
Vy a = 2 th P = a
1. Chng minh rng (d) v (P) c hai im chung phn bit
Honh giao im ng thng (d) v Parabol (P) l nghim ca phng trnh
x2 = 2x + 3 => x2 2x 3 = 0 c a b + c = 0
Nn phng trnh c hai nghim phn bit
c 3
3
x1 = -1 v x2 = a 1
Vi x1 = -1 => y1 = (-1)2 = 1 => A (-1; 1)
Vi x2 = 3 => y2 = 32 = 9 => B (3; 9)
Vy (d) v (P) c hai im chung phn bit A v B
2. Gi A v B l cc im chung ca (d) v (P) . Tnh din tch tam gic OAB ( O l
gc to )
Ta biu din cc im A v B trn mt phng to Oxy nh hnh v

1.0

1.0

1.0

A
D
-1

1
0

C
3

42

AD BC
1 9
.DC
.4 20
2
2
BC.CO 9.3
S BOC

13,5
2
2
AD.DO 1.1
S AOD

0,5
2
2
Theo cng thc cng din tch ta c:
S(ABC) = S(ABCD) - S(BCO) - S(ADO)
= 20 13,5 0,5 = 6 (vdt)
1. Khi m = 4, ta c phng trnh
x2 + 8x + 12 = 0 c = 16 12 = 4 > 0
Vy phng trnh c hai nghim phn bit
x1 = - 4 + 2 = - 2 v x2 = - 4 - 2 = - 6
2. Tm m phng trnh c hai nghim phn bit
x2 + 2mx + m2 2m + 4 = 0
C D = m2 (m2 2m + 4) = 2m 4
phng trnh c hai nghim phn bit th D > 0
=> 2m 4 > 0 => 2(m 2) > 0 => m 2 > 0 => m > 2
Vy vi m > 2 th phng trnh c hai nghim phn bit
S ABCD

1.0

1.0

H M

N
A

1. Ba im O, M, D thng hng:
Ta c MC l tip tuyn ca ng trn (O) MC MO (1)

Xt ng trn (I) : Ta c CMD


900 MC MD (2)
T (1) v (2) => MO // MD MO v MD trng nhau
O, M, D thng hng
2. Tam gic COD l tam gic cn
CA l tip tuyn ca ng trn (O) CA AB(3)
ng trn (I) tip xc vi AC ti C CA CD(4)

T (3) v (4) CD // AB => DCO


(*)
COA
( Hai gc so le trong)

CA, CM l hai tip tuyn ct nhau ca (O) COA


(**)
COD

T (*) v (**) DOC


Tam gic COD cn ti D
DCO
3. ng thng i qua D v vung gc vi BC lun i qua mt im c nh khi M di

1.0

1.0

1.0
43

ng trn ng trn (O)

* Gi chn ng vung gc h t D ti BC l H. CHD


900 H (I) (Bi ton
qu tch)
DH ko di ct AB ti K.
Gi N l giao im ca CO v ng trn (I)

CND
900
NC NO
=>
COD can tai D
Ta c t gic NHOK ni tip
O
DCO

V c H
( Cng b vi gc DHN) NHO
NKO
1800 (5)
2
1

* Ta c : NDH
(Cng chn cung NH ca ng trn (I))
NCH

CBO
HND
HCD
DHN

COB (g.g)

HN OB

HD OC

OB OA
HN ON

...

M ONH

CDH
OC OC
HD CD

OA CN ON
...

OC CD CD
NHO DHC (c.g.c)
0

NHO 90 M NHO
NKO
1800 (5) NKO
900 , NK AB NK // AC
K l trung im ca OA c nh (PCM)
2
2
2
Cu 5 (1.0 im) : Cho a,b,c l cc s dng khng m tho mn : a b c 3
a
b
c
1
2
2

Chng minh rng : 2


a 2b 3 b 2c 3 c 2a 3 2
2
2
2
a b2 a b
a 2 b2 c2 a b c
* C/M b :
v
.


x
y
x y
x
y x
x yz
Tht vy
2
a 2 b2 a b
2
2

a 2 y b 2 x x y xy a b ay bx 0
x
y
x y
(ng) PCM

1.0

a 2 b2 c2 a b c
p dng 2 ln , ta c:

x
y x
x yz
2
2
* Ta c : a 2b 3 a 2b 1 2 2a 2b 2 , tng t Ta c:
a
b
c
a
b
c
A 2
2
2

a 2b 3 b 2c 3 c 2a 3 2a 2b 2 2b 2c 2 2c 2a 2
1
a
b
c
A

(1)

2 1a4 4
b 41 4 b42c 4
14 4c 4a43
1
2

Ta chng minh

a
b
c

1
a b 1 b c 1 c a 1

44

a
b
c
1
1
1 2
a b 1
b c 1
c a 1
b 1
c 1
a 1

2
a b 1 b c 1 c a 1
b 1
c 1
a 1

2
a b 1 b c 1 c a 1

b 1
c 1
a 1

a b 1 b 1 b c 1 c 1 c a 1 a 1
1 4 4 4 4 4 4 4 4 44 2 4 4 4 4 4 4 4 4 4 43
2

(2)

3 B

* p dng B trn ta c:

a b c 3
3 B
a b 1 b 1 b c 1 c 1 c a 1 a 1
2
a b c 3
2

3 B

(3)

a 2 b 2 c 2 ab bc ca 3(a b c ) 3

* M:
2 a 2 b 2 c 2 ab bc ca 3(a b c) 3
2a 2 2b 2 2c 2 2ab 2bc 2ca 6a 6b 6c 6
2a 2 2b 2 2c 2 2ab 2bc 2ca 6a 6b 6c 6 ( Do : a 2 b 2 c 2 3)
a 2 b 2 c 2 2ab 2bc 2ca 6a 6b 6c 9
a b c 3

a b c 3

a 2 b 2 c 2 ab bc ca 3(a b c) 3
T (3) v (4) (2)
Kt hp (2) v (1) ta c iu phi chng minh.
Du = xy ra khi a = b = c = 1

(4)

45

S GIO DC V O TO
THNH PH CN TH
CHNH THC

K THI TUYN SINH LP 10 THPT


NM HC 2012-2013
Kha ngy:21/6/2012
MN: TON
Thi gian lm bi: 120 pht (khng k thi gian pht )

Cu 1: (2,0 im)
Gii h phng trnh , cc phng trnh sau y:

x y 43
3 x 2 y 19

1.

2. x 5 2 x 18
3. x 2 12 x 36 0
4. x 2011 4 x 8044 3
Cu 2: (1,5 im)

Cho biu thc: K 2

1
1 a 1
: 2 (vi a 0, a 1 )
a 1
a a a

1. Rt gn biu thc K.
2. Tm a K 2012 .
Cu 3: (1,5 im)
2
2
Cho phng trnh (n s x): x 4 x m 3 0 * .
1. Chng minh phng trnh (*) lun c hai nghim phn bit vi mi m.
2. Tm gi tr ca m phng trnh (*) c hai nghim x1 , x2 tha x2 5 x1 .
Cu 4: (1,5 im)
Mt t d nh i t A n B cch nhau 120 km trong mt thi gian quy nh. Sau khi i c 1 gi
th t b chn bi xe cu ha 10 pht. Do n B ng hn xe phi tng vn tc thm 6 km/h.
Tnh vn tc lc u ca t.
Cu 5: (3,5 im)
Cho ng trn O , t im A ngoi ng trn v hai tip tuyn AB v AC ( B, C l cc tip
im). OA ct BC ti E.
1. Chng minh t gic ABOC ni tip.
2. Chng minh BC vung gc vi OA v BA.BE AE.BO .
3. Gi I l trung im ca BE , ng thng qua I v vung gc OI ct cc tia AB, AC theo th t

ti D v F . Chng minh IDO


v DOF cn ti O .
BCO
4. Chng minh F l trung im ca AC .
GI GII:
Cu 1: (2,0 im)
Gii h phng trnh , cc phng trnh sau y:

x 21
x y 43
2 x 2 y 86 5 x 105

x y 43
y 22
3 x 2 y 19 3 x 2 y 19

1.

2. x 5 2 x 18 ; K : x 9

46

x 23(TMK )
x 5 2 x 18

x 13 ( KTMK )
x 5 2 x 18
3

2
2
3. x 12 x 36 0 ( x 6) 0 x 6
4.

x 2011 4 x 8044 3; K : x 2011


3 x 2011 3 x 2012(TMK )

Cu 2: (1,5 im)

1
1 a 1
: 2 (vi a 0, a 1 )
a a a
a 1

1
1 a 1
a a 1
a 1

K 2
: 2 2
:

a (a 1)
a a a
a ( a 1)
a 1

1
1
1
2
:
2
: a( a 1) 2 a
a
(
a

1)
a
(
a

1)
a
(
a

1)

a = 503 (TMK)
K 2012
2 a = 2012

Cho biu thc: K 2

Cu 3: (1,5 im)
Cho phng trnh (n s x):.
1.

x 2 4 x m 2 3 0 *

16 4m 2 12 4m 2 4 4 0; m

V y (*) lun c hai nghim phn bit vi mi m.


2. Tm gi tr ca m phng trnh (*) c hai nghim x1 , x2 tha x2 5 x1 .
Theo h thc VI-ET c :x1.x2 = - m2 + 3 ;x1+ x2 = 4; m x2 5 x1 => x1 = - 1 ; x2 = 5
Thay x1 = - 1 ; x2 = 5 vo x1.x2 = - m2 + 3 => m = 2 2
Cu 4: (1,5 im)
120
( h)
x
Sau 1 h t i c x km => qung ng cn li 120 x ( km)
Vt lc sau: x + 6 ( km/h)
1 120 x 120

Pt 1
=> x = 48 (TMK) => KL
6
x6
x
HD C3
Tam gic BOC cn ti O => gc OBC = gc OCB
T gic OIBD c gc OID = gc OBD = 900 nn OIBD n i tip => gc ODI = gc OBI

Do IDO
BCO
Li c FIOC n i tip ; nn gc IFO = gc ICO
Suy ra gc OPF = gc OFP ; v y DOF cn ti O .
HD C4
Xt t gic BPFE c IB = IE ; IP = IF ( Tam gic OPF cn c OI l ng cao=> )
Nn BPEF l Hnh bnh hnh => BP // FE
Tam gic ABC c EB = EC ; BA // FE; nn EF l TB ca tam gic ABC => FA = FC

Gi x (km/h) l vt d nh; x > 0 => Thi gian d nh :

47

thi vo THPT nm hc 2012 -

S GD T NGH AN
2013

Mn thi: Ton
Thi gian 120 pht
Ngy thi 24/ 06/ 2012

CHNH THC
Cu 1: 2,5 im:

Cho biu thc A =

1
1
x 2

.
x 2
x 2
x

a) Tm iu kin xc nh v t gn A.
b) Tm tt c cc gi tr ca x A

1
2

c) Tm tt c cc gi tr ca x B

7
A t gi tr nguyn.
3

Cu 2: 1,5 im:
Qung ng AB di 156 km. Mt ngi i xe my t A, mt ngi i xe p t B. Hai xe
xut pht cng mt lc v sau 3 gi gp nhau. Bit rng vn tc ca ngi I xe my
nhanh hn vn tc ca ngi I xe p l 28 km/h. Tnh vn tc ca mi xe?
Cu 3: 2 im:
Chjo phng trnh: x2 2(m-1)x + m2 6 =0 ( m l tham s).
a) GiI phng trnh khi m = 3
2
2
b) Tm m phng trnh c hai nghim x1, x2 tha mn x1 x2 16

Cu 4: 4 im
Cho im M nm ngoi ng trn tm O. V tip tuyn MA, MB vi ng trn (A, B l
cc tip im). V ct tuyn MCD khng I qua tm O ( C nm gia M v D), OM ct
AB v (O) ln lt ti H v I. Chng minh.
a) T gic MAOB ni tip.
b) MC.MD = MA2
c) OH.OM + MC.MD = MO2
d) CI l tia phn gic gc MCH.

48

HNG DN GII
Cu 1: (2,5 im)
a, Vi x > 0 v x 4, ta c:
1
1
x 2
x 2 x 2
x 2

.
=
= ... =
.
x 2
x 2
x
( x 2)( x 2)
x

A=

x 2

b, A =
c, B =

7
.
3

2
1
... x > 4.
>
x 2
2

14
2
=
l mt s nguyn ...
3( x 2)
x 2

x 2 = 7,

2
x 2

x 2 l c ca 14 hay

x 2 = 1,

x 2 = 14.

(Gii cc pt trn v tm x)
Cu 2: (1,5 im)
Gi vn tc ca xe p l x (km/h), iu kin x > 0
Th vn tc ca xe my l x + 28 (km/h)
Trong 3 gi:
+ Xe p i c qung ng 3x (km),
+ Xe my i c qung ng 3(x + 28) (km), theo bi ra ta c phng trnh:
3x + 3(x + 28) = 156
Gii tm x = 12 (TMK)
Tr li: Vn tc ca xe p l 12 km/h v vn tc ca xe my l 12 + 28 = 40 (km/h)
Cu 3: (2,0 im)
a, Thay x = 3 vo phng trnh x2 - 2(m - 1)x + m2 - 6 = 0 v gii phng trnh:
x2 - 4x + 3 = 0 bng nhiu cch v tm c nghim x1 = 1, x2 = 3.
b, Theo h thc Vit, gi x1, x2 l hai nghim ca phng trnh
x2 - 2(m - 1)x + m2 - 6 = 0 , ta c:
x1 x2 2(m 1)

2
x1.x2 m 6

49

v x12 + x22 = (x1 + x2)2 - 2x1.x2 = 16


Thay vo gii v tm c m = 0, m = -4
Cu 4: (4,0 im).
T vit GT-KL
A
D
C

M
I

H H

B
a, V MA, MB l cc tip tuyn ca ng trn (O) ti A v B nn cc gc ca t gic MAOB vung ti A v
B, nn ni tip c ng trn.

), nn ng dng. T suy ra
v MAC

b, MAC v MDA c chung M


= MDA
(cng chn AC
MA MD

MC.MD MA2 (fcm)


MC MA

c, MAO v AHO ng dng v c chung gc O v AMO HAO


(cng chn hai cung bng nhau ca
ng trn ni tip t gic MAOB). Suy ra OH.OM = OA2
p dng nh l Pitago vo tam gic vung MAO v cc h thc OH.OM = OA 2 MC.MD = MA2 suy ra
iu phi chng minh.
d, T MH.OM = MA2, MC.MD = MA2 suy ra MH.OM = MC.MD

MH MC

(*)
MD MO

Trong MHC v MDO c (*) v DMO


chung nn ng dng.

MC MO MO
MC MO

hay
(1)
HC MD OA
CH OA

Ta li c MAI
(cng chn hai cung bng nhau) AI l phn gic ca MAH
.
IAH
Theo t/c ng phn gic ca tam gic, ta c:

MI MA

(2)
IH AH

chung v MHA
MHA v MAO c OMA
MAO
900 do ng dng (g.g)

MO MA

(3)
OA AH

50

T (1), (2), (3) suy ra

MC MI

suy ra CI l tia phn gic ca gc MCH


CH IH

S GIO DC V O TO
H NAM
CHNH THC

K THI TUYN SINH LP 10 THPT


NM HC 2012 2013
Mn: Ton
Thi gian lm bi: 120 pht
Ngy thi : 22/06/2012

Cu 1 (1,5 im)
Rt gn cc biu thc sau:
a) A 2 5 3 45 500
b) B

8 2 12
8
3 1

Cu 2: (2 im)
a) Gii phng trnh: x2 5x + 4 = 0
3x y 1
b) Gii h phng trnh:
x 2y 5
Cu 3: (2 im)
Trong mt phng to Oxy cho Parabol (P) c phng trnh: y = x2 v ng thng (d) c phng trnh: y =
2mx 2m + 3 (m l tham s)
a) Tm to cc im thuc (P) bit tung ca chng bng 2
b) Chng minh rng (P) v (d) ct nhau ti hai im phn bit vi mi m.
Gi y1 , y 2 l cc tung giao im ca (P) v (d), tm m y1 y 2 9
Cu 4: (3,5 im)
Cho ng trn tm O, ng knh AB. Trn tip tuyn ca ng trn (O) ti A ly im M ( M khc A). T
M v tip tuyn th hai MC vi (O) (C l tip im). K CH vung gc vi AB ( H AB ), MB ct (O) ti
im th hai l K v ct CH ti N. Chng minh rng:
a) T gic AKNH l t gic ni tip.
b) AM2 = MK.MB
c) Gc KAC bng gc OMB
d) N l trung im ca CH.
Cu 5(1 im)
Cho ba s thc a, b, c tho mn a 1; b 4;c 9
Tm gi tr ln nht ca biu thc :
bc a 1 ca b 4 ab c 9
P
abc

51

52

53

S GIO DC V OTO

K THI TUYN SINH LP 10 THPT NM HC 2012-2013


KHA NGY : 19/6/2012
MN : TON
Thi gian lm bi: 120 pht (khng k thi gian giao )

QUNG TR
CHNH THC

Cu 1:(2 im)
1.Rt gn cc biu thc (khng dng my tnh cm tay):
a) 2 50 - 18

b) P

a 1

a 1

1
, vi a 0,a 1
a 1

2.Gii h phng trnh (khng dng my tnh cm tay):

x y 4

2x y 5

Cu 2:(1,5 im)
Gi x1, x2 l hai nghim ca phng trnh x 2 5 x 3 0 .Khng gii phng trnh, tnh gi tr cc
biu thc sau:
a, x1 + x2

b,

1
x1 x 2

c, x12 x 22

Cu 3:(1,5 im)
Trn mt phng ta , gi (P) l th hm s y x 2
a, V (P)
b, Tm ta giao im ca (P) v ng thng d: y = -2x+3
Cu 4:(1,5 im)
Hai xe khi hnh cng mt lc i t a im A n a im B cch nhau 100km. Xe th nht chy
nhanh hn xe th hai 10km/h nn n B sm hm 30 pht, Tnh vn tc mi xe.
Cu 5:(3,5 im)
Cho ng trn (O). ng thng (d) khng i qua tm (O) ct ng trn ti hai im A v B theo
th t, C l im thuc (d) ngoi ng trn (O). V ng knh PQ vung gc vi dy AB ti D ( P thuc
cung ln AB), Tia CP ct ng trn (O) ti im th hai l I, AB ct IQ ti K.
a) Chng minh t gic PDKI ni tip ng trn.
b) Chng minh CI.CP = CK.CD
c) Chng minh IC l phn gic ca gc ngoi nh I ca tam gic AIB.
d) Cho ba im A, B, C c nh. ng trn (O) thay i nhng vn i qua A v B. Chng minh
rng IQ lun i qua mt im c nh.

54

S GIO DC O TO
NINH THUN

K THI TUYN SINH VO LP 10 THPT


NM HC 2012 2013
Kha ngy: 24 6 2012
Mn thi: TON
Thi gian lm bi: 120 pht

CHNH THC

Bi 1: (2,0 im)
2x y 3
x 3y 4
b) Xc nh cc gi tr ca m h phng trnh sau v nghim:
(m 2) x (m 1) y 3
( m l tham s)

x 3y 4
Bi 2: (3,0 im)
Cho hai hm s y = x2 v y = x + 2.
a) V th hai hm s cho trn cng mt h trc ta Oxy.
b) Bng php tnh hy xc nh ta cc giao im A, B ca hai th trn (im A c
honh m).
c) Tnh din tch ca tam gic OAB (O l gc ta )
Bi 3: (1,0 im)
Tnh gi tr ca biu thc H = ( 10 2) 3 5
Bi 4: (3,0 im)
Cho ng trn tm O, ng knh AC = 2R. T mt im E trn on OA (E khng trng vi A
v O). K dy BD vung gc vi AC. K ng knh DI ca ng trn (O).
a) Chng minh rng: AB = CI.
b) Chng minh rng: EA2 + EB2 + EC2 + ED2 = 4R2
2R
c) Tnh din tch ca a gic ABICD theo R khi OE =
3
Bi 5: (1,0 im)
Cho tam gic ABC v cc trung tuyn AM, BN, CP. Chng minh rng:
3
(AB + BC + CA) < AM + BN + CP < AB + BC + CA
4
P N:
Bi 1: (2,0 im)
x 1
2x y 3 2x y 3 5 y 5

a) Gii h phng trnh:

x 3 y 4
y 1
x 3y 4 2x 6 y 8
b) H phng trnh v nghim khi:
m 2 m 1
1 3
3m 6 m 1
m 2 m 1 3
5

m
1
3
4
2
4m 4 9
m 1 3
3
4
Bi 2: (3,0 im)
a) V (d) v (P) trn cng mt h trc ta .
x
-2
-1
0
1
2
2
4
1
0
1
4
y =x (P)
a) Gii h phng trnh:

-2

0
55

y = x + 2(d)

1
-2
-10

A
1

-5

10

-2

-4

-6

b) Ta giao im ca (P) v (d) l nghim ca h phng trnh:


y x2
x2 x 2
x1 1; x2 2
x2 x 2 0

y x2
y1 1; y2 4
y x2 y x2
Ta cc giao im ca (d) v (P): A (-1;1) v B (2;4)
1
1
1
c) SOAB = .(1+4).3 .1.1 - .2.4 = 3
2
2
2
Bi 3: (1,0 im)
H = ( 10 2) 3 5

5 1

62 5

5 1

5 1 5 1 4

Bi 4: (3,0 im)
a) Chng minh rng: AB = CI.
Ta c: BD AC (gt)

= 900 ( gc ni tip chn na ng trn) BD BI


DBI
AB = CI
Do : AC // BI AB CI
b) Chng minh rng: EA2 + EB2 + EC2 + ED2 = 4R2
V BD AC AB AD nn AB = AD

E
O

Ta c: EA2 + EB2 + EC2 + ED2 = AB2 + CD2 = AD2 + CD2 = AC2 = (2R)2 = 4R2
2R
c) Tnh din tch ca a gic ABICD theo R khi OE =
3
1
1
SABICD = SABD + SABIC = .DE.AC + .EB.(BI + AC)
2
2
2R
R
2R
5R
AE =
* OE =
v EC =
+R=
3
3
3
3

56

R 5R
5R 2
R 5
R 5
.
=
DE =
. Do : EB =
3 3
9
3
3
R 4R
* BI = AC 2AE = 2R 2.
=
3
3
1 R 5
1 R 5 4R
R 5 16 R
8R 2 5
Vy: SABICD = .
.2R +
.(
+ 2R) =
.
=
(vdt)
2 3
2 3
3
3
6
9
Bi 5: (1,0 im)
* DE2 = AE.EC =

Cho tam gic ABC v cc trung tuyn AM, BN, CP. Chng minh rng:
3
(AB + BC + CA) < AM + BN + CP < AB + BC + CA
4

1
1
1
AM; GN = BN; GP = CP
3
3
3
V AM, BN, CP cc trung tuyn, nn: M, N, P ln lt l trung im ca BC, AC, AB
Do : MN, NP, MP l cc ng trung bnh ca ABC
1
1
1
Nn: MN = AB; NP = BC; MP = AC
2
2
2
p dng bt ng thc tam gic, ta c:
1
1
* AM < MN + AN hay AM <
AB + AC (1)
2
2
1
1
Tng t: BN <
AB + BC (2)
2
2
1
1
CP < BC + AC (3)
2
2
T (1), (2), (3) suy ra:
AM + BN + CP < AB + BC + CA
(*)
1
1
1
* GN + GM > MN hay BN + AM > AB (4)
3
3
2
1
1
1
Tng t: BN + CP > BC (5)
3
3
2
1
1
1
CP + AM > AC (6)
3
3
2
T (4), (5), (6) suy ra:
Gi G l trng tm ca ABC, ta c: GM =

1
1
1
1
1
1
1
1
1
BN + AM + BN + CP + CP + AM > AB + BC+ AC
3
3
3
3
3
3
2
2
2
2
1
(AM + BN + CP) > (AB + AC + BC)
3
2
3
(AB + BC + CA) < AM + BN + CP
(**)
4
3
T (*), (**) suy ra: (AB + BC + CA) < AM + BN + CP < AB + BC + CA
4

57

58

CHNH THC

59

60

61

S GIO DC V OTO

K THI TUYN SINH LP 10 THPT NM HC 2012-2013


Kha ngy : 24/6/2012
Mn thi : TON
Thi gian lm bi: 120 pht (khng k thi gian giao )

THA THIN HU

CHNH THC

Bi 1:(2,0 im)
a).Cho biu thc: C =

53 5
5

3 3
3 1

5 3 . Chng t C = 3

b) Gii phng trnh : 3 x 2 x 2 4 = 0


Bi 2:(2,0 im)
Cho hm s y = x2 c th (P) v ng thng (d) i qua im M (1;2) c h s gc k 0.
a/ Chng minh rng vi mi gi tr k 0. ng thng (d) lun ct (P) ti hai im phn bit A v B.
b/ Gi xA v xB l honh ca hai im A v B.Chng minh rng x A + xB x A .xB 2 = 0
Bi 3:(2,0 im)
a/ Mt xe la i t ga A n ga B.Sau 1 gi 40 pht, mt xe la khc i t ga A n ga B vi vn
tc ln hn vn tc ca xe la th nht l 5 km/h.Hai xe la gp nhau ti mt ga cch ga B 300
km.Tm vn tc ca mi xe, bit rng qung ng st t ga A n ga B di 645 km.
2 x y 5 x y

b/ Gii h phng trnh :

20
20
x y x y 7

Bi 4:(3,0 im)
Cho na ng trn (O) ng knh BC.Ly im A trn tia i ca tia CB.K tip tuyn AF vi na
ng trn (O) ( F l tip im), tia AF ct tia tip tuyn Bx ca na ng trn (O) ti D ( tia tip
tuyn Bx nm trong na mt phng b BC cha na ng trn (O)) .Gi H l giao im ca BF vi
DO ; K l giao im th hai ca DC vi na ng trn (O).
a/ Chng minh rng : AO.AB=AF.AD.
b/ Chng minh t gic KHOC ni tip.
c/ K OM BC ( M thuc on thng AD).Chng minh

BD DM

=1
DM AM

Bi 5:(1,0 im)

Cho hnh ch nht OABC, COB


= 300 .Gi CH l ng cao ca tam gic
COB, CH=20 cm.Khi hnh ch nht OABC quay mt vng quanh cnh OC c
nh ta c mt hnh tr, khi tam gic OHC to thnh hnh (H).Tnh th
tch ca phn hnh tr nm bn ngoi hnh (H).
(Cho 3,1416 )

30 0

K
12 cm

62

63

64

65

66

S GIO DC V O TO
PH TH

K THI TUYN SINH


VO LP 10 TRUNG HC PH THNG
NM HC 2012-2013
Mn ton
Thi gian lm bi: 120 pht, khng k thi gian giao
thi c 01 trang
-------------------------------------------

Cu 1 (2)
a) Gii phng trnh 2x 5 =1
b) Gii bt phng trnh 3x 1 > 5
Cu 2 (2)

3x y 3

2x y 7

a) Gii h phng trnh

b) Chng minh rng

1
1
6

3 2 3 2 7

Cu 3 (2)
Cho phng trnh x2 2(m 3)x 1 = 0
a) Gii phng trnh khi m = 1
b) Tm m phng trnh c nghi m x1 ; x2 m biu thc
2
A = x1 x1x2 + x22 t gi tr nh nht? Tm gi tr nh nht .
Cu 4 (3)
Cho tam gic ABC vung ti A. Ly B lm tm v ng trn tm B bn knh AB.Ly C lm tm v ng
trn tm C bn knh AC, hai ng trn ny ct nhau ti im th 2 l D.V AM, AN ln lt l cc dy cung
ca ng trn (B) v (C) sao cho AM vung gc vi AN v D nm gia M; N.
a) CMR: ABC=DBC
b) CMR: ABDC l t gic n i tip.
c) CMR: ba im M, D, N thng hng
d) Xc nh v tr ca cc dy AM; AN ca ng trn (B) v (C) sao cho on MN c di ln nht.

x 2 5 y 2 8 y 3

Cu 5 (1) Gii H PT

(2 x 4 y 1) 2 x y 1 (4 x 2 y 3) x 2 y

---------------------------Ht-------------------------GI GII
Cu 1 (2) a) Gii phng trnh 2x 5 = 1
b) Gii bt phng trnh 3x 1 > 5
p n a) x = 3 ; b) x > 2

3x y 3
Cu 2 (2) a) Gii h phng trnh
2x y 7
67

b) Chng minh rng

1
1
6

3 2 3 2 7

p n a) x = 2 ; y = 3
b) VT =

3 2 3 2 6
=VP (pcm)
92
7

Cu 3 (2) Cho phng trnh x2 2(m 3)x 1 = 0


c) Gii phng trnh khi m = 1
d) Tm m phng trnh c nghi m x1 ; x2 m biu thc
2
A = x1 x1x2 + x22 t gi tr nh nht? Tm gi tr nh nht .
p n a) x1 = 2 5 ; x2 = 2 5
e) Thy h s ca pt : a = 1 ; c = A 1 pt lun c 2 nghi m
Theo vi- t ta c x1 + x2 =2(m 3) ; x1x2 = 1
M A=x12 x1x2 + x22 = (x1 + x2 )2 3x1x2 = 4(m 3)2 + 3 3
GTNN ca A = 3 m = 3
Cu 4 (3)
Hng dn
a) C AB = DB; AC = DC; BC chung ABC = DBC (c-c-c)
b) ABC = DBC gc BAC =BDC = 900 ABDC l t gic n i tip
c) C gcA1 = gcM1 ( ABM cn ti B)
A
gcA4 = gcN2 ( ACN cn ti C)
gcA1 = gcA4 ( cng ph A2;3 )
M
gcA1 = gcM1 =gcA4= gcN2
B
gcA2 = gcN1 ( cng chn cung AD ca (C) )
Li c A1+A2 + A3 = 900 => M1 + N1 + A3 = 900
M AMN vung ti A => M1 + N1 + M2 = 900
=> A3 = M2 => A3 = D1
D
CDN cn ti C => N1;2 = D4
D2;3 + D1 + D4 =D2;3 + D1 + N1;2 = D2;3 + M2 + N1 + N2
= 900 + M2 + N1 + M1 ( M1 = N2)
= 900 + 900 = 1800
M; D; N thng hng.
d) AMN ng dng ABC (g-g)
Ta c NM2 = AN2 +AM2 NM ln nht th AN ; AM ln nht
M AM; AN ln nht khi AM; AN ln lt l ng knh ca (B) v (C)
V y khi AM; AN ln lt l ng knh ca (B) v (C) th NM ln nht.
1

1
2

1 2

x 2 5 y 2 8 y 3

Cu 5 (1): Gii H PT

(2 x 4 y 1) 2 x y 1 (4 x 2 y 3) x 2 y

Hng dn

x 2 5 y 2 8 y 3

(2 x 4 y 1) 2 x y 1 (4 x 2 y 3) x 2 y

x 2 5 y 2 8 y 3(1)

(2 x 2 y 1) 2 x y 1 (2 2 x y 1 1) x 2 y (2)
68

T (2) t x +2y = a ; 2xy 1 = b (a:b 0)


Ta dc (2a-1) b =(2b 1) a ( a b )(2 ab 1) = 0 a = b
x = 3y + 1 thay vo (1) ta dc
2y2 y 1= 0 => y1 = 1 ; y2 = 1/2
=> x1 = 4 ; x2 = 1/2
Thy x2 + 2y2 = 1 < 0 (loi)
V y h c nghi m (x; y) = (4 ; 1)

S gio dc v o to

k thi tuyn sinh vo lp 10 thpt chuyn

Hng yn

Nm hc 2012 - 2013

Mn thi: Ton

CHNH THC
( thi c 01 trang)

(Dnh cho th sinh d thi cc lp chuyn: Ton,


Tin)

Thi gian lm bi: 150 pht

Bi 1: (2 im)
a) Cho A = 20122 2012 2.20132 20132 . Chng minh A l mt s t nhin.

1 x
2
x

y2 y

b) Gii h phng trnh


x 1 x 3

y y
Bi 2: (2 im)
a) Cho Parbol (P): y = x2 v ng thng (d): y = (m +2)x m + 6. Tm m ng thng (d) ct
Parabol (P) ti hai im phn bit c honh dng.
b) Gii phng trnh: 5 + x + 2 (4 x)(2x 2) 4( 4 x 2x 2)
Bi 3: (2 im)
a) Tm tt c cc s hu t x sao cho A = x2 + x+ 6 l mt s chnh phng.
b) Cho x > 1 v y > 1. Chng minh rng :

(x 3 y3 ) (x 2 y 2 )
8
(x 1)(y 1)

Bi 4 (3 im)
Cho tam gic ABC nhn ni tip ng trn tm O, ng cao BE v CF. Tip tuyn ti B v C ct nhau ti
S, gi BC v OS ct nhau ti M
a) Chng minh AB. MB = AE.BS
b) Hai tam gic AEM v ABS ng dng
69

c) Gi AM ct EF ti N, AS ct BC ti P. CMR NP vung gc vi BC
Bi 5: (1 im)
Trong mt gii bng c 12 i tham d, thi u vng trn mt lt (hai i bt k thi u vi nhau ng
mt trn).
a) Chng minh rng sau 4 vng u (mi i thi u ng 4 trn) lun tm c ba i bng i mt cha thi
u vi nhau.
b) Khng nh trn cn ng khng nu cc i thi u 5 trn?

HNG DN GII
Bi 1: (2 im)
a) Cho A = 20122 2012 2.20132 20132

t 2012 = a, ta c

2
2
2
2
20122 20122.20132 20132 a a (a 1) (a 1)

(a 2 a 1)2 a 2 a 1
x
y a

b) t
Ta c
x 1 b

2

1 x

1
x
x 2 3
x 3
y
y

y
y

1 x
x 1 x 3
x 3

y y
y y

b2 a 3 b2 b 6 0 a 6 a 1
v

nn

b 3 b 2
ba 3 ba 3
Bi 2:
a) ycbt tng ng vi PT x2 = (m +2)x m + 6 hay x2 - (m +2)x + m 6 = 0 c hai nghim dng
phn bit.
b) t t =

4 x 2x 2

Bi 3:
a) x = 0, x = 1, x= -1 khng tha mn. Vi x khc cc gi tr ny, trc ht ta chng minh x phi l s
nguyn.
+) x2 + x+ 6 l mt s chnh phng nn x2 + x phi l s nguyn.

m
vi m v n c c nguyn ln nht l 1.
n
m 2 m m 2 mn
Ta c x2 + x = 2
l s nguyn khi m 2 mn chia ht cho n2
2
n
n
n
nn m 2 mn chia ht cho n, v mn chia ht cho n nn m2 chia ht cho n v do m v n c c
nguyn ln nht l 1, suy ra m chia ht cho n( mu thun vi m v n c c nguyn ln nht l
1). Do x phi l s nguyn.
t x2 + x+ 6 = k2
Ta c 4x2 + 4x+ 24 = 4 k2 hay (2x+1)2 + 23 = 4 k2 tng ng vi 4 k2 - (2x+1)2 = 23
+) Gi s x

70

(x 3 y3 ) (x 2 y 2 ) x 2 (x 1) y 2 (y 1)
x2
y2

=
(x 1)(y 1)
(x 1)(y 1)
y 1 x 1
2
2
(x 1) 2(x 1) 1 (y 1) 2(y 1) 1

y 1
x 1
2
2
(x 1) (y 1)
2(y 1) 2(x 1)
1
1

x 1
y 1 y 1 x
x 1
1
y 1
Theo BT Csi

(x 1) 2 (y 1) 2
(x 1) 2 (y 1) 2

2
.
2 (x 1)(y 1)
y 1
x 1
y 1
x 1
2(y 1) 2(x 1)
2(y 1) 2(x 1)

.
4
x 1
y 1
x 1
y 1
1
1
1
1

2
.
y 1 x 1
y 1 x 1

1
1
.
(x 1)(y 1) 2.2
y 1 x 1

1
1
.
. (x 1)(y 1) 4
y 1 x 1

Bi 4
C

P
M

Q
N

O
F
B

a) Suy ra t hai tam gic ng dng l ABE v BSM


b) T cu a) ta c

AE MB

(1)
AB BS

M MB = EM( do tam gic BEC vung ti E c M l trung im ca BC


AE EM

Nn
AB BS

C MOB
BAE,
EBA
BAE
900 , MBO
MOB
900
71

Nn MBO
do MEB
OBA(
MBE)
EBA

Suy ra MEA
(2)
SBA

T (1) v (2) suy ra hai tam gic AEM v ABS ng dng(pcm.)


c) D thy SM vung gc vi BC nn chng minh bi ton ta chng minh NP //SM.
+ Xt hai tam gic ANE v APB:

T cu b) ta c hai tam gic AEM v ABS ng dng nn NAE


,
PAB

M AEN
( do t gic BCEF ni tip)
ABP

Do hai tam gic ANE v APB ng dng nn

AN AE

AP AB

AM AE

( hai tam gic AEM v ABS ng dng)


AS AB
AM AN

Suy ra
nn trong tam gic AMS c NP//SM( nh l Talet o)
AS AP
Li c

Do bi ton c chng minh.


Bi 5
a. Gi s kt lu n ca bi ton l sai, tc l trong ba i bt k th c hai i u vi nhau ri. Gi
s i g p cc i2, 3, 4, 5. Xt cc b (1; 6; i) vi i {7; 8; 9;;12}, trong cc b ny phi c t nht
m t c p u vi nhau, tuy nhin1 khng g p 6 hay i nn 6 g p i vi mi i {7; 8; 9;;12} , v l v
i 6 nh th u hn 4 tr n. V y c pcm.
b. Kt lu n khng ng. Chia 12 i thnh 2 nhm, mi nhm 6 i. Trong mi nhm ny, cho tt c
cc i i m t thi u vi nhau. Lc ny r rng mi i u5 tr n. Khi xt 3 i bt k, phi c 2 i
thu c cng m t nhm, do 2 i ny u vi nhau. Ta c phn v d.
Co th giai quyt n gian hn cho cu a. nh sau:
Do mi i u 4 tr n nn tn ti hai iA, B cha u vi nhau. Trong cc i cn li, v A v B ch
u 3 tr n vi h nn tng s tr n caA, B vi cc i ny nhiu nht l 6 v do , tn ti i C trong s cc
i cn li cha u vi c A v B. Ta c A, B, C l b ba i i m t cha u vi nhau.

72

S GIO DC V O TO

K THI TUYN SINH VO LP 10 THPT

HNG YN
CHNH THC

NM HC 2012 - 2013
Thi gian lm bi: 120 pht (khng k thi gian giao )

PHN A: TRC NGHIM KHCH QUAN (2 im)


T cu 1 n cu 8, hy chn phng n ng v vit ch ci ng trc phng n vo bi lm
Cu 1: gi tr ca biu thc
A.

B. 3 2

10

Cu 2: Biu thc

2 8 bng:
C.

D.

2 4

x 1 x 2 c ngha khi:
B. x 2

A. x < 2

C. x 1

D. x 1

Cu 3: ng thng y = (2m 1)x + 3 song song vi ng thng y = 3x 2 khi:


A. m = 2

B. m = - 2

C. m 2

D. m 2

2x y 3
c nghim (x;y) l:
x y 3

Cu 4: H phng trnh
A. (-2;5)

B. (0;-3)

C. (1;2)

D. (2;1)

Cu 5: Phng trnh x2 6x 5 = 0 c tng hai nghim l S v tch hai nghim l P th:


A. S = 6; P = -5

B. S = -6; P = 5

C. S = -5; P = 6

D. S = 6; P = 5

C. (2;-4)

D. ( 2 ;-1)

Cu 6: th hm s y = -x2 i qua im:


A. (1;1)

B. (-2;4)

Cu 7: Tam gic ABC vung ti A c AB = 4cm; AC = 3cm th di ng cao AH l:


A.

3
cm
4

B.

12
cm
5

C.

5
cm
12

D.

4
cm
3

Cu 8: Hnh tr c bn knh y v chiu cao cng bng R th th tch l


A. 2 R 3

B. R 2

C. R 3

D. 2 R 2
73

PHN B: T LUN ( 8,0 im)


Bi 1: (1 im)

a) Tm x bit 3 x 2 2 x 2
b) Rt gn biu thc: A

1 3

Bi 2: (1,5 im)
Cho ng thng (d): y = 2x + m 1
a) Khi m = 3, tm a im A(a; -4) thuc ng thng (d).
b) Tm m ng thng (d) ct cc trc ta Ox, Oy ln lt ti M v N sao cho tam gic OMN c
din tch bng 1.
Bi 3: (1,5 im) Cho phng trnh x2 2(m + 1)x + 4m = 0

(1)

a) Gii phng trnh (1) vi m = 2.


b) Tm m phng trnh (1) c nghim x1, x2 tha mn (x1 + m)(x2 + m) = 3m2 + 12
Bi 4: (3 im) T im A bn ngoi ng trn (O), k cc tip tuyn Am, AN vi ng trn (M, N l
cc tip im). ng thng d i qua A ct ng trn (O) ti hai im phn bit B,C (O khng thuc (d), B
nm gia A v C). Gi H l trung im ca BC.
a) Chng minh cc im O, H, M, A, N cng nm trn mt ng trn,

b) Chng minh HA l tia phn gic ca MHN


.
c) Ly im E trn MN sao cho BE song song vi AM. Chng minh HE//CM.
Bi 5 (1,0 im) Cho cc s thc dng x, y , z tha mn x + y + z = 4.
Chng minh rng

1 1
1
xy xz
HNG DN GII:

Phn trc nghim:


Cu

Cu
B

Cu

Cu

Cu
A

Cu
B

Cu
B

Cu
C

Phn t lun:
Bi 1:

a) Tm x bit 3 x 2 2 x 2 3 x 2 2 x 2 2 x 2 . Vy x 2
b) Rt gn biu thc: A

1 3

3 1 3 3 3 1 3 1 . Vy A 1

Bi 2:
a) Thay m = 3 vo phng trnh ng thng ta c: y = 2x + 2.
im A(a; -4) thuc ng thng (d) khi v ch khi: -4 = 2a + 2 suy ra a = -3.

74

b) Cho x = 0 suy ra y = m 1 suy ra: ON m 1 , cho y = 0 suy ra x


suy ra OM

1 m
2

1 m
m 1
hayOM
2
2

din tch tam gic OMN = 1 khi v ch khi: OM.ON = 2 khi v ch khi m 1 .

m 1
2
2

Khi v ch khi (m 1)2 = 4 khi v ch khi: m 1 = 2 hoc m 1 = -2 suy ra m = 3 hoc m = -1


Vy din tch tam gic OMN = 1 khi v ch khi m = 3 hoc m = -1.
Bi 3: Cho phng trnh x2 2(m + 1)x + 4m = 0

(1)

a) Gii phng trnh (1) vi m = 2.


b) Tm m phng trnh (1) c nghim x1, x2 tha mn (x1 + m)(x2 + m) = 3m2 + 12
HD:
a) Thay m = 2 vo phng trnh (1) ta c phng trnh:
x2 6x + 8 = 0 Khi v ch khi (x 2)(x 4) = 0 khi v ch khi x = 2 hoc x = 4
Vy vi m = 2 th phng trnh c 2 nghim x1 = 2 , x2 = 4.
b) Ta c ' m 1 4m m 1 0 vy phng trnh lun c nghim vi mi m.
2

S 2 m 1

p dng nh l Vi-et ta c:

P 4m

(x1 + m)(x2 + m) = 3m2 + 12 khi v ch khi x1x2 + (x1 + x2) m - 2 m2 12 = 0. S khi v ch khi : 4m
+ m.2(m + 1) 2m2 12 = 0 khi v ch khi 6m = 12 khi v ch khi m= 2
Bi 5 :
a) Theo tnh cht tip tuyn ct nhau ta c :
AMO ANO 900
Do H l trung im ca BC nn ta c:
AHO 900
Do 3 im A, M, H, N, O thuc ng trn ng knh AO
b) Theo tnh cht hai tip tuyn ct nhau ta c: AM = AN
Do 5 im A, M, H, O, N cng thuc mt ng trn nn:
AHM AHN (gc ni tip chn hai cung bng nhau)

Do HA l tia phn gic ca MHN

c) Theo gi thit AM//BE nn MAC


( ng v) (1)
EBH
Do 5 im A, M, H, O, N cng thuc mt ng trn nn:

(gc ni tip chn cung MH)


MAH
MNH

(2)

T (1) v (2) suy ra ENH


EBH
75

Suy ra t gic EBNH ni tip

Suy ra EHB
ENB

M ENB
(gc ni tip chn cung MB)
MCB

Suy ra: EHB


MCB
Suy ra EH//MC.
Bi 5 (1,0 im) Cho cc s thc dng x, y , z tha mn x + y + z = 4.
Chng minh rng

1 1
1
xy xz

Hng dn:
V x + y + z = 4 nn suy ra x = 4 (y + z)
Mt khc:

1 1
1 1 1
1 1
1 1 x do x dng. (*)
xy xz
x y z
y z

Thay x = 4 (y + z) vo (*) ta c :
2

2
1

1 1
1
1
1

4 y z 2 y 2 z 0
y
z 0
y

y z
y
z
z

Lun ng vi mi x, y, z dng, du bng xy ra khi v ch khi : y = z = 1, x = 2.

76

S GIO DC V O TO
NG NAI
CHNH THC

K THI TUYN SINH VO LP 10 THPT NM HC 2012


Kha ngy : 29 , 30 / 6 / 2012
Mn thi : TON HC
Thi gian lm bi : 120 pht
( ny c 1 trang , 5 cu )

Cu 1 : ( 1,5 im )
1 / Gii phng trnh : 7x2 8x 9 = 0 .
3x + 2y =1
2 / Gii h phng trnh :
4x + 5y = 6
Cu 2 : ( 2,0 im )
1 / Rt gn cc biu thc : M

12 +3
3 2 2
; N
3
2 1

2 / Cho x1 ; x2 l hai nghim ca phng trnh : x2 x 1 = 0 .


Tnh :

1 1
+
.
x1 x 2

Cu 3 : ( 1,5 im )
Trong mt phng vi h trc ta Oxy cho cc hm s :
2
y = 3x c th ( P ) ; y = 2x 3 c th l ( d ) ; y = kx + n c th l ( d1 ) vi k v n l nhng s
thc .
1 / V th ( P ) .
2 / Tm k v n bit ( d1 ) i qua im T( 1 ; 2 ) v ( d1 ) // ( d ) .
Cu 4 : ( 1,5 im )
Mt tha t hnh ch nht c chu vi bng 198 m , din tch bng 2430 m2 . Tnh chiu di v chiu
rng ca tha t hnh ch nht cho .
Cu 5 : ( 3,5 im )
Cho hnh vung ABCD . Ly im E thuc cnh BC , vi E khng trng B v E khng trng C . V
EF vung gc vi AE , vi F thuc CD . ng thng AF ct ng thng BC ti G . V ng thng a i
qua im A v vung gc vi AE , ng thng a ct ng thng DE ti im H .
1 / Chng minh

AE CD

.
AF DE

2 / Chng minh rng t gic AEGH l t gic ni tip c ng trn .


3 / Gi b l tip tuyn ca ng trn ngoi tip tam gic AHE ti E , bit b ct ng trung trc ca
on thng EG ti im K . Chng minh rng KG l tip tuyn ca ng trn ngoi tip tam gic AHE .
77

HNG DN GII:
Cu 1 : ( 1,5 im )
1 / Gii phng trnh : 7x2 8x 9 = 0 ( x1,2 =

3x + 2y =1

2 / Gii h phng trnh :

4 79
)
7

( x ; y ) = (1 ; 2 )

4x + 5y = 6

Cu 2 : ( 2,0 im )
1 / Rt gn cc biu thc : M

12 +3 2 3 3

2 3
3
3

3 2 2
N

2 1

2 1

2 1

2 1

2 / Cho x1 ; x2 l hai nghim ca phng trnh : x2 x 1 = 0 .

c
b
1
a
a
1 1 x1 x 2 1
+
1
Nn :
x1 x 2 x1x 2 1
S = 1 ; P =

Cu 3 : ( 1,5 im )
1 / V th ( P ) .
2 / ( d1 ) // ( d ) nn k = 2 ; n 3 v i qua im T( 1 ; 2 ) nn x = 1 ; y = 2 . Ta c phng trnh : 2 =
1.2 + n n = 0
Cu 4 : ( 1,5 im )
Gi x ( m ) l chiu di tha t hnh ch nht ( 49,5 < x < 99 )
Chiu rng ca tha t hnh ch nht l : 99 x ( m )
Theo bi ta c phng trnh : x ( x 99 ) = 2430
Gii c : x1 = 54 ( nhn ) ; x2 = 45 ( loi )
Vy chiu di tha t hnh ch nht l 54 ( m )
Chiu rng ca tha t hnh ch nht l : 99 54 = 45 ( m )
Cu 5 : ( 3,5 im )
1 / Chng minh t gic AEFD ni tip
D

A
1

AEF
DCE ( g g )
AE AF

=
DC DE
AE DC

=
AF DE
ph vi A

2 / Ta c A
2
1

Ta c E1 ph vi D1
D

M A
1
1

A 2 E1

B
1
2

E
1

I
1

D
H

K
F

Suy ra t gic AEFD ni tip ng trn ng knh HE


78

Gi I trung im ca HE I l tm ng trn ngoi tip t gic AEFD cng l ng trn ngoi


tip AHE
I nm trn ng trung trc EG IE = IG
V K nm trn ng trung trc EG KE = KG
Suy ra IEK = IGK ( c-c-c )

IGK
IEK
900
KG IG ti G ca ng trn ngoi tip AHE
KG l tip tuyn ca ng trn ngoi tip AHE

THI TUYN SINH VO LP 10 CHUYN TNH NG NAI


NM HC 2012 - 2013
CHNH THC
Mn thi: Ton chung
Thi gian lm bi: 120 pht ( khng k thi gian giao )
( thi ny gm mt trang, c bn cu)
Cu 1: ( 2,5 im) .
1/ Gii cc phng trnh :
a/ x 4 x 2 20 0
b/
x 1 x 1
x y 3 1
2/ Gii h phng trnh :
y x 3
Cu 2 : ( 2,0 im) .
Cho parabol y = x2 (P) v ng thng y = mx (d), vi m l tham s.
1/ Tm cc gi tr ca m (P) v (d) ct nhau ti im c tung bng 9.
2/ Tm cc gi tr ca m (P) v (d) ct nhau ti 2 im, m khong cch gia hai im ny bng
6
Cu 3 : ( 2,0 im)
1
1
3 1

).
2 3 2 3 3 3
2/ Chng minh : a 5 b5 a 3b 2 a 2b3 , bit rng a b 0 .
1/ Tnh : P (

Cu 4 : (3,5 im)
Cho tam gic ABC vung A, ng cao AH. V ng trn tm O, ng knh AH, ng trn ny
ct cc cnh AB, AC theo th t ti D v E .
1/ Chng minh t gic BDEC l t gic ni tip c ng trn.
2/ Chng minh 3 im D, O, E thng hng.
3/ Cho bit AB = 3 cm, BC = 5 cm. Tnh din tch t gic BDEC.
--------HT------

79

CHNH THC

THI TUYN SINH VO LP 10 CHUYN TNH NG NAI


NM HC 2012 - 2013
Mn thi: Ton ( mn chuyn)
Thi gian lm bi: 150 pht ( khng k thi gian giao )
( thi ny gm mt trang, c nm cu)

Cu 1. (1,5 im)
Cho phng trnh

x 4 16 x 2 32 0 ( vi x R )

Chng minh rng x 6 3 2 3 2 2 3 l mt nghim ca phng trnh cho.


Cu 2. (2,5 im)
2 x( x 1)( y 1) xy 6
( vi x R, y R ).
2 y ( y 1)( x 1) yx 6

Gii h phng trnh

Cu 3.(1,5 im)
Cho tam gic u MNP c cnh bng 2 cm. Ly n im thuc cc cnh hoc pha trong tam gic u
MNP sao cho khong cch gia hai im tu ln hn 1 cm ( vi n l s nguyn dng). Tm n ln
nht tho mn iu kin cho.
Cu 4. (1 im)
Chng minh rng trong 10 s nguyn dng lin tip khng tn ti hai s c c chung ln hn 9.
Cu 5. (3,5 im)
Cho tam gic ABC khng l tam gic cn, bit tam gic ABC ngoi tip ng trn (I). Gi D,E,F ln
lt l cc tip im ca BC, CA, AB vi ng trn (I). Gi M l giao im ca ng thng EF v
ng thng BC, bit AD ct ng trn (I) ti im N (N khng trng vi D), gii K l giao im
ca AI v EF.
1) Chng minh rng cc im I, D, N, K cng thuc mt ng trn.
2) Chng minh MN l tip tuyn ca ng trn (I).
----------HT-----------

80

GII THI VO LP 10
CHUYN LNG TH VINH NG NAI
NM 2012 2013
Mn: Ton chung
----------------Cu 1: ( 2,5 im) .
1/ Gii cc phng trnh :
a/ x 4 x 2 20 0 (*)
t x 2 t ;(t 0)
(*) t2 t 20 = 0 (t1 = 5 (nhn) v t2 = - 4 ( loi)); Vi t = 5 => x2 = 5 x = 5
Vy phng trnh c hai nghim x = 5 v x = - 5
b/
x 1 x 1 ( iu kin x 1 )
( x 1) 2 ( x 1) 2 x 1 x 2 2 x 1 x 2 3 x 0 x(x-3) = 0
x = 0 ( loi) v x = 3 ( nhn).
Vy phng trnh c mt nghim x = 3.
x y 3 1
2/ Gii h phng trnh :
y x 3
T y x 3 y 3 x y 3 0 y 3 y 3
1

x y 3 1 x y 3 1
x y 4 2 x 1
2

(nhn)


y x 3
y x 3
y x 3 y x 3 y 7

2
1 7
1 7
Vy h phng trnh c 2 nghim (x; y): ( ; ), ( ; )
2 2
2 2
Cu 2 : ( 2,0 im) .
1/ P.trnh honh giao im (P) v (d) :

x1 0
x2 m

x 2 mx 0 x( x m) 0

V giao im ( P ) : y x 2 y m 2 . Vi y = 9 => m2 = 9 (m = 3 v m = -3)


Vy vi m 3 th (P) v (d) ct nhau ti im c tung bng 9.
2/ T cu 1 => (P) v (d) lun ct nhau ti hai im phn bit khi m 0 .
Khi giao im th nht l gc to O ( x = 0; y = 0), giao im th 2 l im A c ( x = m; y = m2).
81

Khong cch gia hai giao im : AO =

(1)

m2 m4 6 m4 m2 6 0
t t m 2 ;(t 0) (1) t 2 t 6 0 (t1 = 3 ( nhn ) v t2 = - 2 ( loi))
Vi t1 = 3 m2 = 3 , m 3 ( nhn)
Vy vi m 3 th (P) ct (d) ti hai im c khong cch bng

6.

Cu 3 : ( 2,0 im)
1/ Tnh:
1
1
3 1 2 3 2 3
3 1
P(

).

.
2
43
2 3 2 3 3 3
3( 3 1)
2/ Ta c:
a 5 b5 a 3b 2 a 2b3 a 5 b5 a 3b 2 a 2b3 0 a 3 (a 2 b 2 ) b 3 (a 2 b 2 ) 0 (a 3 b3 )(a 2 b 2 ) 0
(a b) 2 (a b)(a 2 b 2 ab) 0
( a b) 2 0
(vi mi a, b R ).
a b 0
( theo gi thit)
2
2
a b ab 0 ( vi mi a, b R )
Nn bt ng thc cui ng. Vy a 5 b5 a3b2 a 2b3 vi a b 0 (pcm)
V :

Cu 4 : (3,5 im)
A

O
D

B
H

1/ Ni H vi E .
+ HEA 900 ( v AH l ng knh), AHC 900 ( AH l ng cao)
=> AHE ACB (cng ph vi EHC )
(1)
+ ADE AHE ( gc ni tip cng chn cung AE)
(2)
T (1) v (2) => ADE = ACB =>T gic BDEC ni tip ng trn ( c gc i bng gc k
b gc i)
2/ V DAE 900 => DE l ng knh => D, O, E thng hng (pcm).
3/ Ta c S BDEC S ABC SADE
+ ABC vung c AH l ng cao:
AB. AC
6 (cm2)
AC BC 2 AB 2 4cm => sABC
2
AB. AC 12
DE AH

(cm) ( cng l ng knh t O).


BC
5
+ ADE v ABC c : A chung , ADE = ACB ( cu 1)
=> ADE ~ ABC (g.g) => t s din tch bng bnh phng t ng dng :
2
S ABC .DE 2
S
DE
AED

AED

S ABC BC
BC 2
82

+ S BDEC S ABC S ADE S ABC (1

DE 2
122
)

6(1

) = 4,6176 (cm2)
BC 2
52.52
---------HT---------

GII THI VO LP 10
CHUYN LNG TH VINH NG NAI
NM 2012 2013
Mn: Ton chuyn
----------------4
2
Cu 1: Phng trnh cho : x 16 x 32 0 ( vi x R ) ( x 2 8) 2 32 0 (1)
Vi x 6 3 2 3 2 2 3 x 3 2 2 3 2 2 3
=> x 2 8 2 2 3 2 3 2 3
Th x vo v phi ca (1) ta c:
( x 2 8) 2 32 (8 2 2 3 2 3 2 3 8) 2 32 4(2 3) 4 3 12(2 3) 32
= 8 4 3 8 3 24 12 3 32 0 ( v phi bng v tri)
Vy x 6 3 2 3 2 2 3 l mt nghim ca phng trnh cho ( pcm)
2 x( x 1)( y 1) xy 6 (1)
2 x( x 1)( y 1) 6 xy

2 y ( y 1)( x 1) yx 6 (2)
2 y ( y 1)( x 1) 6 xy
Thay x = 0, y = 0 th h khng tho . Thay x = -1 v y = -1 vo, h khng tho
=>
( x; y ) (0;0); xy 0; x 1 0; y 1 0 6 xy 0
(*)
x 6 xy
xy ( x y ) 6( x y )
- Chia tng v ca hai phng trnh cho nhau : =>
y 6 xy
Thay x = y, h pt c v phi bng nhau, v tri khc nhau (khng tho) => x y 0 ) (**)
6( x y )
=> xy
(3)
x y
- Cng tng v (1) v (2) ca h ta c pt: 2(x+y)(x+1)(y+1) + 2xy = 0
(4)
6( x y ) 6( x y )
)
0
(x + y) ( x + y + xy + 1) + xy = 0 ( x y )( x y 1
x y
x y
x y 0
x y 1 0
6( x y 1)
6
) 0 ( x y )( x y 1)(1
) 0
( x y )( x y 1
x y
x y

6
1 x y 0

Cu 2: H pt cho

83

- Vi x + y = 0 x = - y. Th vo h => -2y2 = 0 (y = 0 v x = 0) khng tho (*)


- Vi x + y +1 =0 x = -y - 1 th vo phng trnh (1) ca h ta c :
y 2 0 y 2
2 y 3 3 y 2 y 6 0 ( y 2)(2 y 2 y 3) 0
2
2 y y 3 0(vn)
Vi y = - 2 => x = 1.Th vo h tho, vy c nghim 1: (x; y) = (1; - 2)
6
0 x y6 0 x y6
- Vi 1
x y
Th x = y -6 vo pt (2) ca h :
2 y 1 0
2
(2) 2 y 3 7 y 2 16 y 6 0 (2 y 1)( y 4 y 6) 0 2
y 4y 6 0
y2 - 4y - 6 = 0

y1 2 10

y2 2 10
1
2y +1 = 0
y3 =
2

x1 4 10

T ba gi tr ca y trn ta tm c ba gi tr x tng ng: x2 4 10

13
x3

2
Th cc gi tr (x; y) tm c vo h (tho).
Vy h phng trnh cho c 4 nghim ( x;y):
(1; -2), ( 4 10; 2 10), (4 10; 2 10), (

13 1
; ).
2 2

Cu 3. (Cch 1)
Tam gic u c cnh bng 2 cm th din tch bng 3 cm2 , tam gic u c cnh bng 1 cm th din
3
3
cm2 . Nu tam gic u c cnh > 1cm th din tch >
cm2
4
4
Gi t l s tam gic u c cnh bng > 1cm cha c trong tam gic u c cnh 2 cm:
1 t p 4 ( vi t l s nguyn dng) => tmax = 3.
Theo nguyn l Drichen s c 1 trong t tam gic u c cnh > 1cm cha ti a 2 im tho mn khong
cch gia hai im bt k lun > 1 cm.
Vy s im tho yu cu bi ton l : 2 n 4 Vy nmax = 4
tch bng

(Cch 2): Gii theo kin thc hnh hc

84

Nu ta chn 3 im 3 nh ca tam gic u cnh bng 2 cm v 3 ng trn ng knh 1 cm, cc


ng trn ny tip xc vi nhau trung im mi cnh tam gic. => Cc im khc trong tam gic cch 3
nh > 1cm ch c th nm trong phn din tch cn li ca tam gic (ngoi phn din tch b ba hinh trn
che ph), c gii hn bi 3 cung trn bn kinh 1 cm.
V 3 dy cung l 3 ng trung bnh ca tam gic c di 1 cm => khong cch gia hai im bt
k nm trong phn din tch cn li ca tam gic lun 1 cm.
=> trong phn din tch ch ly c 1 im m khong cch n 3 nh ca tam gic lun > 1 cm.
Vy s im ln nht tho mn khong cch gia hai im bt k > 1cm l :
nmax = 3 + 1 = 4 im.
Cu 4. Gi a v b l hai s bt k trong 10 s nguyn dng lin tip vi a > b ( a; b nguyn dng)
1 a b 9 .
Gi n l c chung ca a v b, khi : a = n.x v b = n.y ( n, x, y l s nguyn dng).
1
9
9
V a > b => x > y => x y 1 1 n.x n. y 9 x y 1 n 9
n
n
n
Vy trong 10 s nguyn dng lin tip khng tn ti hai s c c chung ln hn 9.
Cu 5.
A

N
K

F
I

1)Ni N v F, D v F.
- Xt ANF v AFD c: AFN = ADF ( v AF l tt) v FAD chung => ANF AFD (g.g) =>
AN AF

AF2 AN . AD
(1)
AF AD
- Xt AFI c: AF IF ( v AF tip tuyn, FI l bn knh) v FK AI ( v AF v AE tt chung v AI ni
tm) => AFI vung ti F c FK l ng cao) => AK.AI = AF2 (2)
- Xt ANK v AID c:
85

+ IAD chung.
+ T (1) v (2) => AN.AD = AK.AI =>

AN
AI

AK AD

=> ANK AID (c.g.c) => NKA = IDN


(3)
- T (3) => t gic DIKN ni tip t (v c gc i bng gc k b gc i)
=> cc im I,D,N,K cng thuc mt ng trn. (pcm).
2) Ta c ID DM ( DM l tip tuyn, DI l bn knh) v IK KM ( cu 1) => t gic DIKM ni tip
ng trn ng knh MI. V 4 im D, I, K, N cng thuc mt ng trn ( cu 1) => hai ng trn
ny cng ngoi tip DIK => hai ng trn trng nhau => N cng nm trn ng trn ng knh MI
=> INM = 900 .
V IN l bn knh ng trn (I), MN IN => MN l tip tuyn ca ng trn (I) ti tip im N.
(pcm).
-----------HT----------

CHNH THC

86

GI GII:
Cu 1c C = 1
Cu 2a ( 2;1) ; Cu 2b b = - 1
Cu 3a a = 1
Cu 3b A ( -1 ; 1 ) ; B (2 ; 4 )
Cu 4a1 12 0 ; nn pt lun c 2 nghi m phn bi t vi mi x
Cu 4 a2 => x1 + x2 = - 5 ; x1x2 = 3
Cu 4b
Gi x ( km/h) l vt xe II => vt xe I l x + 10 ( km/h ) ; x> 0
100
(h)
x
100
Th gian xe II i ht qg :
(h)
x 10
100
100
1
PT
=
=> x = 40
x
x 10
2
KL
Th gian xe I i ht qg :

Cu 5 : a
1. MH = 20 ( cm ) ; ME = 12 ( cm)
2. NPFE l h thang cn
b)
b1
b2
Tam gic ABC vung ti A c AH l g cao => AB2 = BH.BC (1)
87

Tam gic BHE g dng vi tam gic BDC =>

BH BE

BH .BC BD.BE (2)


BD BC

T (1) v (2) => AB2 = BD . BE

K THI TUYN SINH VO LP 10 THPT


NM HC 2012 2013
Mn thi: TON
Thi gian lm bi: 120 pht (khng k thi gian giao )
thi gm 05 cu trong 01 trang

S GIO DC - O TO
TNH NINH BNH

CHNH THC

Cu 1: (2,0 im)
1. Cho biu thc P = x + 5. Tnh gi tr biu thc P ti x = 1.
2. Hm s bc nht y = 2x + 1 ng bin hay nghch bin trn R? V sao?
3. Gii phng trnh x2 + 5x + 4 = 0
Cu 2: (2,5 im)

2x y 1
3x 2 y 5

1. Gii h phng trnh:

2. Cho biu thc Q =

1
1

:
x 1 x x

1
2

vi x > 0 v x 1.
x 1 x 1

a) Rt gn Q.
b) Tnh gi tr ca Q vi x = 7 4 3 .
Cu 3: (1,5 im)
Khong cch gia hai bn sng A v b l 30 km. Mt ca n i xui dng t bn A n bn B ri li
ngc dng t bn B v bn A. Tng thi gian ca n i xui dng v ngc dng l 4 gi . Tm vn tc ca
ca n khi nc yn lng, bit vn tc ca dng nc l 4 km/h.
Cu 4: (3,0 im)
Cho ng trn tm O bn knh R. Mt ng thng d khng i qua O v ct ng trn ti hai im
phn bit A v B. Trn d ly im M sao cho A nm gia M v B. T M k hai tip tuyn MC v MD vi
ng trn (C, D l cc tip im).
88

1. Chng minh rng MCOD l t gic ni tip.


2. Gi I l trung im ca AB. ng thng IO ct tia MD ti K. Chng minh rng KD. KM = KO.
KI
3. Mt ng thng i qua O v song song vi CD ct cc tia MC v MD ln lt ti E v F. Xc
nh v tr ca M trn d sao cho din tch tam gic MEF t gi tr nh nht.
Cu 5: (1,0 im)
Cho a, b, c l cc s thc dng. Chng minh rng:

a
bc ca ab
b
c

a
b
c
b c c a a b
------------------- Ht ---------------------HNG DN GII:
Cu 1:
1) Thay x = 1 vo biu thc P c: P = x + 5 = 1 + 5 = 6.
2) Hm s ng bin trn R v a = 2 > 0
3)Ta thy a b + c = 1 5 + 4 = 0 nn pt c 2 nghim: x1 = 1; x2 = 4
Cu 2:
1.

Vy h pt c nghim : x = 1 v y = 1.
2. Vi x > 0 v x 1, ta c:

a) Q

b) Vi
Suy ra :
Cu 3:
Gi vn tc ca ca n khi nc yn lng l x(km/h) (k:

Vn tc ca ca n khi xui dng: x + 4 (km/h)


Vn tc ca ca n khi ngc dng: x 4 (km/h)
89

Thi gian ca n i xui dng:

30
(h)
x4

Thi gian ca n i ngc dng:

30
(h)
x4

Tng thi gian ca n i xui dng v ngc dng l 4h nn ta c phng trnh:

30
30
+
= 4 x2 15x 16 = 0
x4 x4
ng tho
a K )
x1 1(kho
a K )
x2 16( tho

Gii phng trnh trn ta c:

Vy vn tc ca ca n khi nc yn lng l 16km/h

90

Cu 5: (cch 2)
p dng bt ng thc Csi ta c::

Tng t ta c :

du bng xy ra a = b

du = xy ra a = b = c

91

S GIO DC - O TO
TNH NINH BNH

CHNH THC

K THI TUYN SINH VO LP 10 THPT CHUYN


Mn thi: TON
Ngy thi: 26 / 6 / 2012
Thi gian lm bi: 120 pht

Cu 1 (2 im). Cho phng trnh bc hai n x, tham s m: x2 + 2mx 2m 3 = 0 (1)


a) Gii phng trnh (1) vi m = -1.
b) Xc nh gi tr ca m phng trnh (1) c hai nghim x1, x2 sao cho x12 x 22 nh nht. Tm
nghim ca phng trnh (1) ng vi m va tm c.
Cu 2 (2,5 im).

6x 4

3x

1 3 3x 3

1. Cho biu thc A=


1 3x 3x
3
3
x

2
3
x

4
3
3
x

a) Rt gn biu thc A.
b) Tm cc gi tr nguyn ca x biu thc A nhn gi tr nguyn.
2. Gii phng trnh: x 1 x x1 x 1
Cu 3 (1,5 im). Mt ngi i xe p t A ti B, qung ng AB di 24 km. Khi i t B tr v A ngi
tng vn tc thm 4 km/h so vi lc i, v vy thi gian v t hn thi gian i l 30 pht. Tnh vn tc ca xe
p khi i t A ti B.
Cu 4 (3 im). Cho ABC nhn ni tip (O). Gi s M l im thuc on thng AB (M A, B); N l
im thuc tia i ca tia CA sao cho khi MN ct BC ti I th I l trung im ca MN. ng trn ngoi tip
AMN ct (O) ti im P khc A.
1. C MR cc t gic BMIP v CNPI ni tip c.

92

2. Gi s PB = PC. Chng minh rng ABC cn.

Cu 5 (1 im). Cho x; y R , tha mn x2 + y2 = 1. Tm GTLN ca : P y 2


HNG DN GII:
2) Gii pt :
t

x 1 x

x (1 x ) 1

K : 0 x 1

x a 0; 1 x b 0

Ta c

a b ab 1(*)

2
2
a b 1(**)

T tm c nghim ca pt l x = 0
Cu 5 :
T x 2 y 2 1 1 x, y 1 2 1

2 1

V P y 2 x P( y 2 ) thay vo x 2 y 2 1
a v pt: ( P 2 1) y 2 2 2 P 2 y 2 P 2 1 0

Dng iu kin c nghim ca pt bc hai P 1 PMax 1

2
2

y 2

S GIO DC V O TO
TNH LO CAI
CHNH THC

K THI TUYN SINH VO 10 - THPT


NM HC: 2012 2013
MN: TON
Thi gian: 120 pht (khng k thi gian giao )

Cu I: (2,5 im)
1. Thc hin php tnh: a) 3 2 10 36 64
2. Cho biu thc: P =

b)

2 3

2 5 .

2a 2 4
1
1

3
1 a
1 a 1 a

a) Tm iu kin ca a P xc nh
b) Rt gn biu thc P.
Cu II: (1,5 im)
1. Cho hai hm s bc nht y = -x + 2 v y = (m+3)x + 4. Tm cc gi tr ca m th ca hm s cho
l:
a) Hai ng thng ct nhau
b) Hai ng thng song song.
2. Tm cc gi tr ca a th hm s y = ax2 (a 0) i qua im M(-1; 2).
Cu III: (1,5 im)
1. Gii phng trnh x 2 7x 8 = 0
2. Cho phng trnh x2 2x + m 3 = 0 vi m l tham s. Tm cc gi tr ca m phng trnh c hai
3
3
nghim x1; x2 tha mn iu kin x1 x 2 x1x 2 6
Cu IV: (1,5 im)

93

3x 2y 1
.
x 3y 2
2x y m 1
2. Tm m h phng trnh
c nghim (x; y) tha mn iu kin x + y > 1.
3x y 4m 1
1. Gii h phng trnh

Cu V: (3,0 im) Cho na ng trn tm O ng knh AB = 2R v tip tuyn Ax cng pha vi na


ng trn i vi AB. T im M trn Ax k tip tuyn th hai MC vi na ng trn (C l tip im). AC
ct OM ti E; MB ct na ng trn (O) ti D (D khc B).
a) Chng minh AMOC l t gic ni tip ng trn.
b) Chng minh AMDE l t gic ni tip ng trn.

c) Chng mnh ADE


ACO
-------- Ht --------HNG DN GII:
Cu I: (2,5 im)
1. Thc hin php tnh:

a) 3 2 10 36 64 3 8 100 2 10 12

b)

2 3

2 5

2 3 2 5 3 2 2 5 2

2a 2 4
1
1

2. Cho biu thc: P =


3
1 a
1 a 1 a
a) Tm iu kin ca a P xc nh:
b) Rt gn biu thc P.

P xc nh khi a 0 v a 1

2a 2 4 1 a a 2 a 1 1 a a 2 a 1
2a 2 4
1
1

P=
=
1 a3 1 a 1 a
1 a a 2 a 1
2a 2 4 a 2 a 1 a 2 a a a a a 1 a 2 a a a a
=
1 a a 2 a 1
=

2 2a
2
= 2
2
1 a a a 1 a a 1

Vy vi a 0 v a 1 th P =

2
a2 a 1

Cu II: (1,5 im)


1. Cho hai hm s bc nht y = -x + 2 v y = (m+3)x + 4. Tm cc gi tr ca m th ca hm s cho
l:
a) hm s y = (m+3)x + 4 l hm s bc nht th m + 3 0 suy ra m -3.
th ca hai hm s cho l hai ng thng ct nhau a a
-1 m+3 m -4
Vy vi m -3 v m -4 th th ca hai hm s cho l hai ng thng ct nhau.
b) th ca hm s cho l Hai ng thng song song

a a ' 1 m 3

m 4 tha mn iu kin m -3
b b' 2 4

Vy vi m = -4 th th ca hai hm s cho l hai ng thng song song.


2. Tm cc gi tr ca a th hm s y = ax2 (a 0) i qua im M(-1; 2).
94

V th hm s y = ax2 (a 0) i qua im M(-1; 2) nn ta thay x = -1 v y = 2 vo hm s ta c phng


trnh 2 = a.(-1)2 suy ra a = 2 (tha mn iu kin a 0)
Vy vi a = 2 th th hm s y = ax2 (a 0) i qua im M(-1; 2).
Cu III: (1,5 im)
1. Gii phng trnh x 2 7x 8 = 0 c a b + c = 1 + 7 8 = 0 suy ra x1= -1 v x2= 8
2. Cho phng trnh x2 2x + m 3 = 0 vi m l tham s. Tm cc gi tr ca m phng trnh c hai
3
3
nghim x1; x2 tha mn iu kin x1 x 2 x1x 2 6 .
phng trnh c hai nghim x1; x2 th 0 1 m + 3 0 m 4
Theo viet ta c: x1+ x2 =2 (1) v x1. x2 = m 3 (2)
Theo u bi:

x13 x 2 x1x 32 6 x1x 2 x1 x 2 2x1x 2 = 6


2

(3)

Th (1) v (2) vo (3) ta c: (m - 3)(2) 2 2(m-3)=6 2m =12 m = 6 Khng tha mn iu kin m 4


3
3
vy khng c gi tr no ca m phng trnh c hai nghim x1; x2 tha mn iu kin x1 x 2 x1x 2 6 .
Cu IV: (1,5 im)

3 3y 2 2y 1 7y 7
3x 2y 1
y 1
.

x 3y 2
x 3y 2 x 1
x 3y 2
2x y m 1
2. Tm m h phng trnh
c nghim (x; y) tha mn iu kin x + y > 1.
3x y 4m 1
xm
xm
2x y m 1 5x 5m

2m y m 1
y m 1
3x y 4m 1 2x y m 1
M x + y > 1 suy ra m + m + 1 > 1 2m > 0 m > 0.
1. Gii h phng trnh

Vy vi m > 0 th h phng trnh c nghim (x; y) tha mn iu kin x + y > 1.


Cu V: (3,0 im) Cho na ng trn tm O ng knh AB = 2R v tip tuyn Ax cng pha vi na
ng trn i vi AB. T im M trn Ax k tip tuyn th hai MC vi na ng trn (C l tip im). AC
ct OM ti E; MB ct na ng trn (O) ti D (D khc B).
a) Chng minh AMCO l t gic ni tip ng trn.
b) Chng minh AMDE l t gic ni tip ng trn.

c) Chng mnh ADE


ACO
M
Gii.
D

C
a) MAO
MCO
900 nn t gic AMCO ni tip

b) MEA
MDA
900 . T gic AMDE c
D, E cng nhn AM di cng mt gc 900
Nn AMDE ni tip

c) V AMDE ni tip nn ADE


AME
cng chan cung AE

V AMCO ni tip nn ACO


AME
cng chan cung AO

E
A

Suy ra ADE
ACO

95

S GIO DC V O TO
GIA LAI

CHNH
chnh THC
thc
Ngy thi: 26/6/2012

K THI TUYN SINH VO LP 10 CHUYN


Nm hoc 2012 2013
Mn thi: Ton (khng chuyn)
Thi gian lm bi: 120 pht

Cu 1. (2,0 im)

x2
x 2

x x , vi x 0, x 1
Cho biu thc Q

1
x

2
x

a. Rt gn biu thc Q
b. Tm cc gi tr nguyn ca x Q nhn gi tr nguyn.
Cu 2. (1,5 im)
Cho phng trnh x2 2(m 1)x m 2 0, vi x l n s, m R
a. Gii phng trnh cho khi m 2
b. Gi s phng trnh cho c hai nghim phn bit x1 v x2 . Tm h thc lin h gia x1 v x2
m khng ph thuc vo m.
Cu 3. (2,0 im)
(m 1)x (m 1)y 4m
Cho h phng trnh
, vi m R
x (m 2)y 2
a. Gii h cho khi m 3
b. Tm iu kin ca m phng trnh c nghim duy nht. Tm nghim duy nht .
96

Cu 4. (2,0 im)
Cho hm s y x2 c th (P). Gi d l ng thng i qua im M(0;1) v c h s gc k.
a. Vit phng trnh ca ng thng d
b. Tm iu kin ca k t d ct th (P) ti hai im phn bit.
Cu 5. (2,5 im)
Cho tam gic nhn ABC (AB < AC < BC) ni tip trong ng trn (O). Gi H l giao im ca hai
ng cao BD v CE ca tam gic ABC (D AC, E AB)
a. Chng minh t gic BCDE ni tip trong mt ng trn
b. Gi I l im i xng vi A qua O v J l trung im ca BC. Chng minh rng ba im H, J, I
thng hng
1
1
1

c. Gi K, M ln lt l giao im ca AI vi ED v BD. Chng minh rng


2
2
DK
DA
DM 2
HNG DN GII:
Cu 1.

x 2

x x
x 2 x 1 x 1

a. Q

x2

x 1

x2

x 1

Vy Q

x 1 1

x 1

x2

x 1

x 1

x 2

x 1

x 1

2x
x x 1 x 1. x 2 x . x
x 1
x1
x 1

x 1 1

x 1

x2

x 1

x 1

1
1

x 1
1

x 1
x 1

2x
x 1

b.
Q nhn gi tr nguyn
2x 2x 2 2
2
Q

2
x 1
x 1
x 1
2
Q khi
khi 2 chia ht cho x 1
x 1
x0

x 1 1 x 2

i chiu iu kin th
x 1 2 x 1

x3

x2
x 3

Cu 2. Cho pt x2 2(m 1)x m 2 0, vi x l n s, m R


a.
Gii phng trnh cho khi m 2
Ta c phng trnh x 2 2x 4 0
x 2 2x 4 0 x 2 2x 1 5 x 1 5
2

x 1 5

x 1 5

x 1 5
x 1 5
Vy phng trinh c hai nghim x 1 5 v x 1 5
b.
x 1 5

97

x x 2 2 x1x 2 2 2
x1 x 2 2m 2 (1)
x x 2 2m 2
1
1
(2)
m x1x 2 2
x1x 2 m 2
m x1 x 2 2
Suy ra x1 x 2 2 x1x 2 2 2 x1 x 2 2x1x 2 6 0
Theo Vi-et, ta c

(m 1)x (m 1)y 4m
, vi m R
x (m 2)y 2

Cho h phng trnh

Cu 3.

a. Gii h cho khi m 3

2x 2y 12
x y 6
x7

x 5y 2
x 5y 2
y 1
Vy h phng trnh c nghim x; y vi 7;1
b. iu kin c nghim ca phng trnh
m 1 m 1
m 1 m 2 m 1

1
m2
m 1 m 2 m 1 0 m 1 m 1 0
Ta c h phng trnh

m 1 0
m 1

m 1 0
m 1
Vy phng trnh c nghim khi m 1 v m 1
(m 1)x (m 1)y 4m
m 1
Gii h phng trnh
khi
x (m 2)y 2
m 1

4m

4m
x y

x
(m

1)x

(m

1)y

4m
x

m 1

x (m 2)y 2
x (m 2)y 2 y 2
y
m 1

4m 2
m 1 .
2
m 1

4m 2 2
;
Vy h c nghim (x; y) vi

m 1 m 1
Cu 4.
a. Vit phng trnh ca ng thng d
ng thng d vi h s gc k c dng y kx b
ng thng d i qua im M(0; 1) nn 1 k.0 b b 1
d
:
y kx 1
Vy
b.
Phng trnh honh giao im ca (P) v d
x 2 kx 1 x 2 kx 1 0 , c k 2 4
d ct (P) ti hai im phn bit khi 0
k 2
k 2 4 0 k 2 4 k 2 22 k 2
k2
Cu 5.
a.
BCDE ni tip

BEC
BDC
900
Suy ra BCDE ni tip ng trn
ng knh BC
b.
H, J, I thng hng
IB AB; CE AB (CH AB)
Suy ra IB // CH
98

IC AC; BD AC (BH AC)


Suy ra BH // IC
Nh vy t gic BHCI l hnh bnh hnh
J trung im BC J trung im IH
Vy H, J, I thng hng
1

ACB
AIB
AB
c.
2
ACB DEA

cng b vi gc DEB
ca t gic ni tip BCDE

BAI
AIB
900 v ABI vung ti B

Suy ra BAI
AED
900 , hay EAK
AEK
900
Suy ra AEK vung ti K
Xt ADM vung ti M (suy t gi thit)
DK AM (suy t chng minh trn)www.VNMATH.
1
1
1

Nh vy
2
2
DK
DA
DM 2

S GIO DC V O TO
QUNG NINH
CHNH THC

K THI TUYN SINH LP 10 THPT


NM HC 2012 2013

MN: TON(Dng cho mi th sinh d thi)


Ngy thi: 28/6/2012
Thi gian lm bi: 120 pht (Khng k thi gian giao )
( thi ny c 01 trang)
Cu I. (2,0 im)
1) Rt gn cc biu thc sau:
a) A = 2

1
18
2

b) B =

x 1

1
2

vi x 0, x 1
x 1 x 1

2x y 5
x 2y 4

2. Gii h phng trnh:

Cu II. (2,0 im)


Cho phng trnh (n x): x2 ax 2 = 0 (*)
1. Gii phng trnh (*) vi a = 1.
2. Chng minh rng phng trnh (*) c hai nghim phn bit vi mi gi tr ca a.
3. Gi x1, x2 l hai nghim ca phng trnh (*). Tm gi tr ca a biu thc:
2
2
N= x1 ( x1 2)( x2 2) x2 c gi tr nh nht.
99

Cu III. (2,0 im)Gii bi ton bng cch lp phng trnh hoc h phng trnh.
Qung ng sng AB di 78 km. Mt chic thuyn my i t A v pha B. Sau 1 gi, mt chic
ca n i t B v pha A. Thuyn v ca n gp nhau ti C cch B 36 km. Tnh thi gian ca thuyn, thi gian
ca ca n i t lc khi hnh n khi gp nhau, bit vn
tc ca ca n ln hn vn tc ca thuyn l 4 km/h.
Cu IV. (3,5 im)
Cho tam gic ABC vung ti A, trn cnh AC ly im D (D A, D C). ng trn (O)
ng knh DC ct BC ti E (E C).
1. Chng minh t gic ABED ni tip.
2. ng thng BD ct ng trn (O) ti im th hai I. Chng minh ED l tia phn gic ca gc AEI.
3. Gi s tg ABC 2 Tm v tr ca D trn AC EA l tip tuyn ca ng trn ng knh DC.
CuV. (0.5 im) Gii phng trnh:
7 2 x x (2 x ) 7 x
HNG DN GII:
Cu IV :
c. EA l tip tuyn ca .Trn, . knh CD th gc E1 = gc C1 (1)
M t gic ABED ni tip nn gc E1 = gc B1 (2)
T (1) v (2) gc C1 = gc B1 ta li c gc BAD chung nn
AB
AD
AB 2

AB2 = AC.AD AD =
(I)
AC
AB
AC
AB 1
AC
Theo bi ra ta c : tan (ABC) =
= 2 nn
( II )
AC 2
AB
AB
T (I) v (II) AD =
.
2
AB
Vy AD =
th EA l tip tuyn ca T, knh CD
2

ABD ACB

Cu V:
Gii phng trnh: 7 2 x x (2 x ) 7 x
t 7 x t ; x v K v, t 0
t 2 2v (2 v).t ... (t v)(t 2) 0 t v hoc t=2
Nu t= 2 th
Nu t = v th

7x 2
7x

x = 3 (TM)
x x = 3,5

100

S GIO DC V O TO
KHNH HA
CHNH THC

K THI TUYN SINH VO LP 10 THPT


NM HC 2011 - 2012

Mn thi: TON
Ngy thi : 21/06/2011
Thi gian lm bi: 120 pht

Bi 1( 2 im)
1)

2 3 6 84
2 3 4
1
1
P a(

);(a 1)
a a 1
a a 1

n gin biu thc: A

2) Cho biu thc:

Rt gn P v chng t P 0
101

Bi 2( 2 im)
1) Cho phng trnh bc hai x2 + 5x + 3 = 0 c hai nghim x1; x2. Hy lp mt phng trnh bc
hai c hai nghim (x12 + 1 ) v ( x22 + 1).
2
x

2) Gii h phng trnh 4

3
4
y2
1
1
y2

Bi 3( 2 im)
Qung ng t A n B di 50km.Mt ngi d nh i xe p t A n B vi vn tc khng
i.Khi i c 2 gi,ngi y dng li 30 pht ngh.Mun n B ng thi gian nh,ngi
phi tng vn tc thm 2 km/h trn qung ng cn li.Tnh vn tc ban u ca ngi i xe
p.
Bi 4( 4 im)
Cho tam gic ABC c ba gc nhn v H l trc tm.V hnh bnh hnh BHCD.ng thng i qua
D v song song BC ct ng thng AH ti E.
1) Chng minh A,B,C,D,E cng thuc mt ng trn
2) Chng minh BAE DAC
3) Gi O l tm ng trn ngoi tip tam gic ABC v M l trung im ca BC,ng thng
AM ct OH ti G.Chng minh G l trng tm ca tam gicABC.
4) Gi s OD = a.Hy tnh di ng trn ngoi tip tam gic BHC theo a

HNG DN GII:
Bi 1
2 3 2 6 8 2 ( 2 3 4)(1 2)

1 2
2 3 4
2 3 4
a a 1 a a 1
P a(
); a 1
a a 1
a 2 a 1 a 1 2 a 1 1; vi : a 1

3) A

4)

Bi 2

P ( a 1 1) 2 0; a 1
x2 + 5x + 3 = 0
102

1) C 25 12 13 0
Nn pt lun c 2 nghim phn bit
x1+ x2 = - 5 ; x1x2 = 3
Do S = x12 + 1 + x22 + 1 = (x1+ x2)2 - 2 x1x2 + 2 = 25 6 + 2 = 21
V P = (x12 + 1) (x22 + 1) = (x1x2)2 + (x1+ x2)2 - 2 x1x2 + 1 = 9 + 20 = 29
Vy phng trnh cn lp l x2 21x + 29 = 0
2) K x 0; y 2
3
2
14
7
x2
x y2 4
x2

y3
12 3 3 2 3 4 1 y 2 4

x y 2
x y2
Vy HPT c nghim duy nht ( x ;y) = ( 2 ;3)
Bi 3 :
Gi x(km/h) l vtc d nh; x > 0 ; c 30 pht = (h)
50
( h)
Th gian d nh :
x
Qung ng i c sau 2h : 2x (km)
Qung ng cn li : 50 2x (km)
Vn tc i trn qung ng cn li : x + 2 ( km/h)
50 2 x
( h)
Th gian i qung ng cn li :
x2
1 50 2 x 50
2

Theo bi ta c PT:
2
x2
x
Gii ra ta c : x = 10 (tha K bi ton)
Vy Vn tc d nh : 10 km/h
Bi 4 :
A
Gii cu c)
V BHCD l HBH nn H,M,D thng hng
Tam gic AHD c OM l TBnh => AH = 2 OM
V AH // OM
H

HAG

OMG
slt
G

2 tam gic AHG v MOG c
O
AGH MGO ( )
B
AHGMOG (G G )
M
AH AG

2
MO MG
Hay AG = 2MG
E
Tam gic ABC c AM l trung tuyn; G AM
Do G l trng tm ca tam gic ABC
d) BHC BDC ( v BHCD l HBH)
c B ;D ;C ni tip (O) bn knh l a
Nn tam gic BHC cng ni tip (K) c bn knh a
Do C (K) = 2 a ( VD)
S GIO DC-O TO
K THI TUYN SINH VO 10 THPT NM 2012
BNH NH
Kha ngy 29 thng 6 nm 2012
CHNH THC

Mn thi: TON
103

Ngy thi: 30/6/2012


Thi gian lm bi: 120 pht (khng k thi gian giao )
Bi 1: (3, 0 im)
Hc sinh khng s dng my tnh b ti
a) Gii phng trnh: 2x 5 = 0
yx 2
b) Gii h phng trnh:
5x 3y 10
c) Rt gn biu thc A

5 a 3
a 2

3 a 1 a2 2 a 8

vi a 0, a 4
a4
a 2

d) Tnh gi tr ca biu thc B 4 2 3 7 4 3


Bi 2: (2, 0 im)
Cho parabol (P) v ng thng (d) c phng trnh ln lt l y mx 2 v
y m 2 x m 1 (m l tham s, m 0).
a) Vi m = 1 , tm ta giao im ca (d) v (P).
b) Chng minh rng vi mi m 0 ng thng (d) lun ct parabol (P) ti hai im phn bit.
Bi 3: (2, 0 im)
Qung ng t Quy Nhn n Bng Sn di 100 km. Cng mt lc, mt xe my khi hnh t Quy
Nhn i Bng Sn v mt xe t khi hnh t Bng Sn i Quy Nhn. Sau khi hai xe gp nhau, xe my i 1
gi 30 pht na mi n Bng Sn. Bit vn tc hai xe khng thay i trn sut qung ng i v vn tc
ca xe my km vn tc xe t l 20 km/h. Tnh vn tc mi xe.
Bi 4: (3, 0 im)
Cho ng trn tm O ng knh AB = 2R. Gi C l trung im ca OA, qua C k dy MN vung
gc vi OA ti C. Gi K l im ty trn cung nh BM, H l giao im ca AK v MN.
a) Chng minh t gic BCHK l t gic ni tip.
b) Chng minh AK.AH = R2
c) Trn KN ly im I sao cho KI = KM, chng minh NI = KB.
HNG DN GII:
Bi 1:
5
2
5x 5y 10
2y 20
y 10

5x 3y 10
y x 2
x 8

a) 2x 5 = 0 2 x 5 0 2 x 5 x
yx 2

5x 3y 10

b)
c)

5 a 3 3 a 1 a2 2 a 8 5 a 3
A

a4
a 2
a 2

5a 10 a 3 a 6 3a 6 a a 2 a 2 2 a 8

a 2

a 2


a 2 a
a 2 a 2

a 2 3 a 1

a 2 8a 16
a 2

a 2

2 a 8

a 2 8a 16
a 2

a 2

a 4

a 4 4 a
a4
2

104

d) B 4 2 3 7 4 3

3 1

2 3

3 1 2 3 3 1 2 3 3

Bi 2:
a) Vi m 1 P v d ln lt tr thnh y x 2 ; y x 2 .

Lc phng trnh honh giao im ca P v d l: x 2 x 2 x 2 x 2 0 c


a b c 1 1 2 0 nn c hai nghim l x1 1; x2 2 .
Vi x1 1 y1 1
Vi x2 2 y2 4
Vy ta giao im ca P v d l 1; 1 v 2; 4 .
b) Phng trnh honh giao im ca P v d l:

mx 2 m 2 x m 1 mx 2 m 2 x m 1 0 * .

Vi m 0 th * l phng trnh bc hai n x c

m 2 4m m 1 m 2 4m 4 4m 2 4m 5m 2 4 0 vi mi m. Suy ra * lun c hai


nghim phn bit vi mi m. Hay vi mi m 0 ng thng (d) lun ct parabol (P) ti hai im phn bit.
Bi 3:
i 1h30' 1,5h
t a im :
1,5x
- Quy Nhn l A
100-1,5x
- Hai xe gp nhau l C
A
C
B
- Bng Sn l B
Gi vn tc ca xe my l x km / h . K : x 0 .
Suy ra :
Vn tc ca t l x 20 km / h .
2

Qung ng BC l : 1,5x km

Qung ng AC l : 100 1,5x km


100 1,5x
Thi gian xe my i t A n C l :
h
x
1,5 x
Thi gian t my i t B n C l :
h
x 20
V hai xe khi hnh cng lc, nn ta c phng trnh :

100 1,5 x
1,5 x

x
x 20

Gii pt :
100 1,5 x
1,5 x

100 1,5 x x 20 1,5 x 2 100 x 2000 1,5 x 2 30 x 1,5 x 2


x
x 20
2
3x 70 x 2000 0
2
' 35 3.2000 1225 6000 7225 0 ' 7225 85
M
35 85
40 (tha mn K)
Phng trnh c hai nghim phn bit : x1
E
3
H
35 85
50
I
x2

(khng tha mn K)
3
3
A
C
O
40
km
/
h
Vy vn tc ca xe my l
.
Vn tc ca t l 40 20 60 km / h .

105
N

Bi 4:
a) T gic BCHK l t gic ni tip.
Ta c : AKB 900 (gc ni tip chn na ng trn)

900 ; HCB
900 gt
hay HKB

T gic BCHK c HKB


HCB
900 900 1800
t gic BCHK l t gic ni tip.
b) AK . AH R 2
AC AH
R

AK . AH AC. AB 2 R R 2
D thy ACH AKB g .g
AK
AB
2
c) NI KB
OAM c OA OM R gt OAM cn ti O 1

OAM c MC l ng cao ng thi l ng trung tuyn (gt) OAM cn ti M 2

1 & 2 OAM l tam gic u MOA


600 MON
1200 MKI
600

KMI l tam gic cn (KI = KM) c MKI


600 nn l tam gic u MI MK 3 .
1
1

MON
1200 600 nn l tam gic u MN MB 4
D thy BMK cn ti B c MBN
2
2
Gi E l giao im ca AK v MI.

NKB
NMB
600

D thy
NKB MIK KB // MI (v c cp gc v tr so le trong bng nhau)
MIK 600

mt khc AK KB cmt nn AK MI ti E HME


.
900 MHE

HAC
900 AHC

)
900 MHE
HME
cmt HAC
Ta c : HME
mt khc HAC
(cng chn KB
KMB

AHC MHE

dd

KMB
hay NMI
5 T 3 , 4 & 5 IMN KMB c.g.c NI KB (pcm)
HME
KMB

S GIO DC V O TO
BC GIANG
CHNH THC

THI TUYN SINH LP 10 THPT


NM HC 2012 2013
Mn thi : Ton
Thi gian : 120 pht khng k thi gian giao
106

Ngy thi 30 thng 6 nm 2012


Cu 1. (2 im)
1
- 2
1.Tnh
2- 1
2 .Xc nh gi tr ca a,bit th hm s y = ax - 1 i qua im M(1;5)
Cu 2: (3 im)
1.Rt gn biu thc: A = (

1
2
a - 3 a +2
).(
+1) vi a>0,a 4
a - 2 a- 2 a
a- 2

2 x - 5 y = 9
2.Gii h pt:
3x + y = 5
3. Chng minh rng pt: x 2 + mx + m - 1 = 0 lun c nghim vi mi gi tr ca m.
Gi s x1,x2 l 2 nghim ca pt cho,tm gi tr nh nht ca biu thc

B = x 21 + x 2 2 - 4.( x1 + x2 )
Cu 3: (1,5 im)
Mt t ti i t A n B vi vn tc 40km/h. Sau 2 gi 30 pht th mt t taxi cng xut pht i t A
n B vi vn tc 60 km/h v n B cng lc vi xe t ti.Tnh di qung ng AB.
Cu 4: (3 im)
Cho ng trn (O) v mt im A sao cho OA=3R. Qua A k 2 tip tuyn AP v AQ ca ng trn
(O),vi P v Q l 2 tip im.Ly M thuc ng trn (O) sao cho PM song song vi AQ.Gi N l giao im
th 2 ca ng thng AM v ng trn (O).Tia PN ct ng thng AQ ti K.
1.Chng minh APOQ l t gic ni tip.
2.Chng minh KA2=KN.KP

3.K ng knh QS ca ng trn (O).Chng minh tia NS l tia phn gic ca gc PNM
.
4. Gi G l giao im ca 2 ng thng AO v PK .Tnh di on thng AG theo bn knh R.
Cu 5: (0,5im)
Cho a,b,c l 3 s thc khc khng v tho mn:
a 2 (b + c ) + b 2 (c + a ) + c 2 (a + b) + 2abc = 0

a 2013 + b 2013 + c 2013 = 1

1
1
1
Hy tnh gi tr ca biu thc Q = 2013 + 2013 + 2013
a
b
c
Cu
1

HNG DN CHM
(tham kho)
Ni dung
1
2 +1
2 +1
- 2=
- 2=
2- 1
( 2 - 1).( 2 +1)
( 2) 2 - 1)
KL:

2 = 2 +1-

Do th hm s y = ax-1 i qua M(1;5) nn ta c a.1-1=5 a=6


KL:

2 =1

im
1

107

A=(
=(

a
a ( a - 2)

2
( a - 1).( a - 2)
).(
+1) =
a ( a - 2)
a- 2

a- 2
1
).( a - 1 +1) =
. a =1
a ( a - 2)
a

0,5
0,5

KL:
2

2 x - 5 y = 9

3x + y = 5

KL:
3

Xt Pt:

2 x - 5 y = 9

15 x + 5 y = 25

2 x - 5 y = 9

17 x = 34

y =- 1

x = 2

x 2 + mx + m - 1 = 0

= m 2 - 4(m - 1) = m 2 - 4m + 4 = ( m - 2) 2 0
Vy pt lun c nghim vi mi m
x1 + x2 =- m
Theo h thc Viet ta c
x1 x2 = m - 1
Theo bi

0,25

0,25

B = x 21 + x 2 2 - 4.( x1 + x2 ) = ( x1 + x2 ) 2 - 2 x1 x2 - 4.( x1 + x2 )
= m 2 - 2( m - 1) - 4(- m) = m 2 - 2m + 2 + 4m = m 2 + 2m +1 +1
= (m +1) 2 +1 1

0,5

Vy minB=1 khi v ch khi m = -1


KL:
3

Gi di qumg ng AB l x (km) x>0


x
Thi gian xe ti i t A n B l
h
40
x
Thi gian xe Taxi i t A n B l : h
60
5
Do xe ti xut pht trc 2h30pht =
nn ta c pt
2
x
x
5
=
40 60 2
3 x - 2 x = 300
x = 300
Gi tr x = 300 c tho mn K

0,25
0,25
0,25
0,25
0,25
0,25

Vy di qung ng AB l 300 km.


4

1
Xt t gic APOQ c
APO = 900 (Do AP l tip tuyn ca (O) P)
AQO = 900 (Do AQ l tip tuyn ca (O) Q)
APO + AQO = 1800 ,m hai gc ny l 2 gc i nn t gic APOQ l t gic ni

0,75

tip
108

M
N
A

Xt AKN v PAK c AKP l gc chung


APN = AMP ( Gc ntcng chn cung NP)

M NAK
= AMP (so le trong ca PM //AQ

AKN ~ PKA (gg)


3

0,75

AK NK
=
AK 2 = NK .KP (pcm)
PK
AK

K ng knh QS ca ng trn (O)


Ta c AQ ^ QS (AQ l tt ca (O) Q)
M PM//AQ (gt) nn PM ^ QS
ng knh QS ^ PM nn QS i qua im chnh gia ca cung PM nh
= sd SM
PNS

(hai gc nt chn 2 cung bng nhau)


sd PS
= SNM
Hay NS l tia phn gic ca gc PNM
Chng minh c AQO vung Q, c QG ^ AO(theo Tnh cht 2 tip tuyn ct
nhau)
Theo h thc lng trong tam gic vung ta c
OQ 2 R 2 1
2
OQ = OI .OA OI =
=
= R
OA
3R 3
1
8
AI = OA - OI = 3R - R = R
3
3
Do KNQ ~ KQP (gg) KQ 2 = KN .KP m AK 2 = NK .KP nn AK=KQ
Vy APQ c cc trung tuyn AI v PK ct nhau G nn G l trng tm
2
2 8
16
AG = AI = . R = R
3
3 3
9
Ta c:
a 2 (b + c ) + b 2 (c + a ) + c 2 (a + b ) + 2abc = 0

0,75

0,75

a 2b + a 2c + b 2c + b 2 a + c 2 a + c 2b + 2abc = 0
( a 2b + b 2 a ) + (c 2 a + c 2b ) + (2abc + b 2c + a 2c ) = 0
ab(a + b) + c 2 (a + b) + c (a + b) 2 = 0
( a + b)(ab + c 2 + ac + bc ) = 0
( a + b).( a + c ).(b + c) = 0
*TH1: nu a+ b=0
a =- b
a =- b ta c Q = 1 + 1 + 1 = 1

Ta c 2013
a + b 2013 + c 2013 = 1 c = 1
a 2013 b 2013 c 2013
Cc trng hp cn li xt tng t
1
1
1
Vy Q = 2013 + 2013 + 2013 = 1
a
b
c

0,25

0,25

109

S GIO DC V O TO
YN BI
CHNH THC

THI TUYN SINH LP 10 THPT


NM HC 2012 2013
Mn thi : TON
Thi gian : 120 pht (khng k thi gian giao )
Kha ngy 23 thng 6 nm 2012
( thi c 01 trang, gm 05 cu)

Cu 1: (2,0 im)
1. Cho hm s y = x + 3 (1)
a. Tnh gi tr ca y khi x = 1
b. V th ca hm s (1)
2. Gii phng trnh: 4x 7x + 3 = 0
Cu 2: (2,0 im)
Cho biu thc M = +
1. Tm iu kin ca x biu thc M c ngha. Rt gn biu thc M.
2. Tm cc gi tr ca x M > 1
Cu 3: (2,0 im)
Mt i th m phi khai thc 260 tn than trong mt thi hn nht nh. Trn thc t, mi ngy i
u khai thc vt nh mc 3 tn, do h khai thc c 261 tn than v xong trc thi hn mt
ngy.
Hi theo k hoch mi ngy i th phi khai thc bao nhiu tn than?
Cu 4: (3,0 im)
Cho na ng trn tm O, ng knh AB = 12 cm. Trn na mt phng b AB cha na ng trn
(O) v cc tia tip tuyn Ax, By. M l mt im thuc na ng trn (O), M khng trng vi A v B. AM
ct By ti D, BM ct Ax ti C. E l trung im ca on thng BD.
1. Chng minh: AC . BD = AB.
2. Chng minh: EM l tip tuyn ca na ng trn tm O.
3. Ko di EM ct Ax ti F. Xc nh v tr ca im M trn na ng trn tm O sao cho din tch t gic
AFEB t gi tr nh nht? Tm gi tr nh nht .
Cu 5: (1,0 im)
Tnh gi tr ca biu thc T = x + y + z 7 bit:
x + y + z = 2 + 4 + 6 + 45

110

S GIO DC V O TO
LM NG
CHNH THC
( thi gm 01 trang)

K THI TUYN SINH L 10 THPT


MN THI : TON
Kha ngy : 26 thng 6 nm 2012
Thi gian lm bi : 120 pht

18 2 2 32
2x 3y 1
Cu 2: (0,75) Gii h phng trnh :
4 x 3 y 11
Cu 3: (0,75) Cho tam gic ABC vung ti A, ng cao AH. Bit BH = 9cm, Ch = 16cm.
Tnh di cc on thng AH, BH, AC.
Cu 4: (0,75) Cho hai ng thng (d) : y = (m-3)x + 16 (m 3) v (d): y = x + m2.
Tm m (d) v (d) ct nhau ti mt im trn trc tung
Cu 5: (0,75) Cho AB l dy cung ca ng trn tm O bn knh 12cm. Bit AB = 12cm . Tnh
din tch hnh qut to bi hai bn knh OA, OB v cung nh AB.
Cu 6: (1)
Cho hm s y = ax2 (a 0) c th (P).
a) Tm a bit (P) i qua im A(2;4)
b) Tm k ng thng (d) : y = 2x + k lun ct (P) ti 2 im phn bit.
Cu 7: (0,75) Hnh nn c th th tch l 320 cm3, bn knh ng trn l 8cm. Tnh din tch ton
phn ca hnh nn .
Cu 8: (1)
Cho ng trn (O) ng knh AB, M l trung im ca OA. Qua M v dy cung CD
vung gc vi OA.
a) Chng minh t gic ACOD l hnh thoi .
b) Tia CO ct BD ti I. Chng minh t gic DIOM ni tip.
Cu 9: (1) Hai i cng nhn cng o mt con mng . Nu h cng lm th trong 8 gi xong
vic. Nu h lm ring th i A hon thnh cng vic nhanh hn i B 12 gi. Hi nu
lm ring th mi i phi lm trong bao nhiu gi mi xong vic.
Cu 10: (0,75) Rt gn : 37 20 3 37 20 3
Cu 11: (1) Cho phng trnh : x2 2(m-2)x - 3m2 +2 = 0 (x l n, m l tham s )
Tm m phng trnh c 2 nghim x1; x2 tha : x1(2-x2) +x2(2-x1) = -2
Cu 12: (0,75) Cho na ng trn (O) ng knh AB, v cc tip tuyn Ax v By cng pha vi
na ng trn , M l im chnh gia cung AB, N l mt im thuc on OA
N O, N A . ng thng vung gc vi MN ti M ct Ax v By ln lt ti C v D.
Chng minh : AC = BN
Cu 1: (0,75) Tnh :

111

112

113

S GIO DC V O TO
QUNG NGI

K THI TUYN SINH VO LP 10 NM HC 2012-2013


Mn thi: Ton (khng chuyn)
Thi gian lm bi: 120 pht (khng k thi gian giao )

CHNH THC
Bi 1: (1,5 im)
1/ Thc hin php tnh:

2 1

2 1

x y 1
2x 3y 7

2/ Gii h phng trnh:

3/ Gii phng trnh: 9 x 2 8 x 1 0


Bi 2: (2,0 im)
2
2
Cho parapol P : y x v ng thng d : y 2 x m 1 (m l tham s).

2
2
1/ Xc nh tt c cc gi tr ca m d song song vi ng thng d ' : y 2m x m m .

2/ Chng minh rng vi mi m, d lun ct P ti hai im phn bit A v B.

2
2
3/ K hiu x A ; xB l honh ca im A v im B. Tm m sao cho x A xB 14 .
Bi 3: (2,0 im)
Hai xe t cng i t cng Dung Qut n khu du lch Sa Hunh, xe th hai n sm hn xe th nht l 1
gi. Lc tr v xe th nht tng vn tc thm 5 km mi gi, xe th hai vn gi nguyn vn tc nhng dng
li ngh mt im trn ng ht 40 pht, sau v n cng Dung Qut cng lc vi xe th nht. Tm vn
tc ban u ca mi xe, bit chiu di qung ng t cng Dung Qut n khu du lch Sa Hunh l 120 km
v khi i hay v hai xe u xut pht cng mt lc.
Bi 4: (3,5 im)
Cho ng trn tm O ng knh AB = 2R v C l mt im nm trn ng trn sao cho CA > CB. Gi
I l trung im ca OA. V ng thng d vung gc vi AB ti I, ct tia BC ti M v ct on AC ti P; AM
ct ng trn (O) ti im th hai K.
1/ Chng minh t gic BCPI ni tip c trong mt ng trn.
2/ Chng minh ba im B, P, K thng hng.
3/ Cc tip tuyn ti A v C ca ng trn (O) ct nhau ti Q. Tnh din tch ca t gic QAIM theo R
khi BC = R.
Bi 5: (1,0 im)
2 xy
Cho x 0, y 0 tha mn x 2 y 2 1 . Tm gi tr nh nht ca biu thc A
.
1 xy
-------------- HT --------------

HNG DN GII:
Bi 1:
1/

2 1

2 1

12 2 1 1

x2
x y 1
3 x 3 y 3 5 x 10

x y 1
y 1
2x 3y 7 2x 3 y 7

2/

3/ Phng trnh 9 x 2 8 x 1 0 c a b c 9 8 1 0 nn c hai nghim l: x1 1; x2

1
.
9

Bi 2:
2
2
2
1/ ng thng d : y 2 x m 1 song song vi ng thng d ' : y 2m x m m khi

114

m 1
m2 1

m 1 m 1
2
m

1
m 1 m m

m 1

2/ Phng trnh honh giao im ca d v P l x 2 2 x m 2 1 x 2 2 x m 2 1 0 l phng


trnh bc hai c ac m 2 1 0 vi mi m nn lun c hai nghim phn bit vi mi m. Do d lun ct
2 2m 2

ti hai im phn bit A v B vi mi m.


3/ Cch 1: K hiu x A ; xB l honh ca im A v im B th x A ; xB l nghim ca phng trnh
2
x 2 x m2 1 0 .
Gii phng trnh x 2 2 x m 2 1 0 .
' 1 m2 1 m2 2 0 ' m2 2
Phng trnh c hai nghim l x A 1 m 2 2; xB 1 m 2 2 .
Do

x A 2 xB 2 14 1 m 2 2

1
2

m2 2

14 1 2 m 2 2 m 2 2 1 2 m 2 2 m 2 2 14

2m 2 6 14 2m 2 8 m 2 4 m 2
Cch 2: K hiu x A ; xB l honh ca im A v im B th x A ; xB l nghim ca phng trnh
S x A xB 2
do
x 2 2 x m 2 1 0 . p dng h thc Viet ta c:
2
P x A .xB m 1

x A 2 xB 2 14 x A xB 2 x A .xB 14 22 2 m 2 1 14 4 2m 2 2 14 m 2
2

Bi 3:
Gi vn tc ban u ca xe th nht l x (km/h), xe th hai l y (km/h). K: x > 0; y > 0.
120
Thi gian xe th nht i t cng Dung Qut n khu du lch Sa Hunh l
h .
x
120
h .
Thi gian xe th hai i t cng Dung Qut n khu du lch Sa Hunh l
y
120 120

1 1
V xe th hai n sm hn xe th nht l 1 gi nn ta c phng trnh:
x
y
Vn tc lc v ca xe th nht l x+ 5 (km/h).
120
Thi gian xe th nht v t khu du lch Sa Hunh n cng Dung Qut
h .
x5
120
h .
Thi gian xe th hai v t khu du lch Sa Hunh n cng Dung Qut
y
2
V xe th hai dng li ngh ht 40 ph h , sau v n cng Dung Qut cng lc vi xe th nht nn
3
120 120 2

2 .
ta c phng trnh:
x5
y
3
120 120
x y 1

T (1) v (2) ta c hpt:


120 120 2
x 5
y
3

115

120 120
x y 1
120 120 1

360 x 5 360 x x x 5 x 2 5 x 1800 0


Gii hpt:
120
120
2
x
x

5
3

x 5
y
3
25 4.1800 7225 0 85 .

5 85
40 (tha mn K)
2
5 85
x2
45 (khng tha mn K)
2
120 120
120

1
2 y 60 (tha mn K).
Thay x 40 vo pt (1) ta c:
40
y
y
Vy vn tc ban u ca xe th nht l 40 km/h, xe th hai l 60 km/h.
Q
Bi 4:(Bi gii vn tt)
a) T gic BCPI ni tip (hs t cm).
b) D thy MI v AC l hai ng cao ca MAB P l trc tm
ca MAB BP l ng cao th ba BP MA 1 .
K
Mt khc AKB 900 (gc ni tip chn na ng trn) BK MA 2 .
Phng trnh c hai nghim phn bit: x1

T (1) v (2) suy ra ba im B, P, Q thng hng.


c) AC AB 2 BC 2 4 R 2 R 2 R 3

Khi BC = R d thy tam gic OBC l tam gic u suy ra CBA


600

M QAC
(gc to bi tia tip tuyn v gc ni tip cng chn AC ) do QAC
CBA
600 .

D thy tam gic QAC cn ti Q (QA = QC) c QAC


600 nn l tam gic u AQ AC R 3 .
R
3R
D thy AI ; IB
2
2
3R
3 3R
0
Trong tam gic vung IBM I$ 90 ta c IM IB.tan B IB.tan 600
.
3
2
2
0
Ta chng minh c t gic QAIM l hnh thang vung AQ / / IM ; I$ 90 .

Do SQAIM

1
1
3 3R R R 5 R 3 5 3R 2
AQ

IM
AI

R
3

(vdt).

.
2
2
2
2
4
2
8

Bi 5:

2 xy
2 xy
1
1 xy
1
1
A

1 xy
1 xy
A
2 xy
2 xy 2
1
1
0 do Amin Amax
min .
V x 0, y 0 A 0 A 0
A
A
1
2
2
2
1 (v 2 xy 0 )
Mt khc x y 0 x y 2 xy 2 xy 1
2 xy
1
1 3
1 . Du = xy ra khi x y .
Do
A
2 2
x 0, y 0

2
x y
T x y
2
2
2
x y 1
Cch 1: Ta c A

116

1
2
2
2
2 2
Lc A
. Vy min A khi x y
.
1
3
3
2
1
2
x2 y 2
1
3
1
2
2
4
xy xy 1 xy

Cch 2: Vi x 0, y 0 ta c
2
2
2
1 xy 3
1 xy 3
2 xy
2
4
2
2
2 .
Do A
1 xy
1 xy
3
3
x

y
Du = xy ra khi
.
x 0, y 0

2
x y
T x y
2
2
2
x y 1
Vy min A

2
2
khi x y
.
3
2

Cch 3:
Vi x 0, y 0 v x 2 y 2 1

2
2
2
2 2 2 xy 2 2 xy 6 xy 2 x y 4 xy 2 x y
2
Ta c A

0 A
3 3 1 xy
3 1 xy
3 1 xy
3 1 xy
3

Du = xy ra khi x y
A

2
2
2
. Vy min A khi x y
.
3
2
2

a
a 2 xy
0; b 0
0 a axy 2bxy 0 a x 2 y 2 2b a xy 0
b
b 1 xy

a0
2b a
a 2

a x y
xy 0 2b a

a
b 3
2

117

UBND tnh bc ninh


S gio dc v o to
chnh thc

thi tuyn sinh vo lp 10 thpt


Nm hc 2012 - 2013
Mn thi: Ton (Dnh cho tt c th sinh)
Thi gian: 120 pht (Khng k thi gian giao )
Ngy thi: 30 thng 06 nm 2012

Bi 1 (2,0im)
1) Tm gi tr ca x cc biu thc c ngha:
4
3x 2 ;
2x 1
2) Rt gn biu thc:

(2 3) 2 3
2 3

Bi 2 (2,0 im)
Cho phng trnh: mx2 (4m -2)x + 3m 2 = 0 (1) ( m l tham s).
1) Gii phng trnh (1) khi m = 2.
2) Chng minh rng phng trnh (1) lun c nghim vi mi gi tr ca m.
3) Tm gi tr ca m phng trnh (1) c cc nghim l nghim nguyn.
Bi 3 (2,0 im)
Gii bi ton sau bng cch lp phng trnh hoc h phng trnh:
Mt mnh vn hnh ch nht c chu vi 34m. Nu tng thm chiu di 3m v chiu rng 2m th din
tch tng thm 45m2. Hy tnh chiu di, chiu rng ca mnh vn.
Bi 4 (3,0 im)
Cho ng trn O. T A l mt im nm ngoi (O) k cc tip tuyn AM v AN vi (O) ( M; N l
cc tip im ).
1) Chng minh rng t gic AMON ni tip ng trn ng knh AO.
2) ng thng qua A ct ng trn (O) ti B v C (B nm gia A v C ). Gi I l trung im ca
BC. Chng minh I cng thuc ng trn ng knh AO.
3) Gi K l giao im ca MN v BC . Chng minh rng AK.AI = AB.AC.
Bi 5 (1,0 im)
Cho cc s x,y tha mn x 0; y 0 v x + y = 1.
Tm gi tr ln nht v nh nht ca A = x2 + y2.

--------------------- Ht -------------------Cu 1:
a)

3x 2 c ngha 3x 2 0 3 x 2 x

2
3

4
1
c ngha 2 x 1 0 2 x 1 x
2x 1
2
b) A

(2 3) 2 3
2 3

(2 3) (2 3) 2
(2 3)(2 3)

(2 3)(2 3)
22 32

2 2 32

1
1

Cu 2: mx 2 (4m 2) x 3m 2 0 (1)
118

1.Thay m = 2 vo pt ta c:
(1) 2 x 2 6 x 4 0 x 2 3x 2 0
Ta thy: 1 3 +2 = 0 nn pt c 2 nghim: x1 0; x2 2
2. * Nu m = 0 th (1) 2 x 2 0 x 1 .
Suy ra: Pt lun c nghim vi m=0
*Nu m 0 th ph (1) l pt bc 2 n x.
Ta c: ' (2m 1) 2 m(3m 2) 4m 2 4m 1 3m2 2m (m 1) 2 0 m 0
Kt lun: Kt hp 2 trng hp ta c: pt lun c nghim vi mi m (pcm)
3. * Nu m = 0 th (1) 2 x 2 0 x 1 nguyn
Suy ra: Vi m = 0 pt c nghim nguyn
2m 1 m 1

1
x1
m
* Nu m # 0 th ph (1) l pt bc 2 n x. T 2 ta c: pt c 2 nghim:
x 2m 1 m 1 3m 2
2
m
m
3m 2
2
Z 3 Z (m 0) 2Mm hay m l
pt (1) c nghim nguyn th nghim x2 phi nguyn
m
m
c ca 2 m = {-2; -1; 1; 2}
Kt lun: Vi m = { 1; 2;0 } th pt c nghim nguyn
Cu 3:
Gi chiu di hcn l x (m); chiu rng l y (m) (0 < x, y < 17)
x y 34 : 2 17
x 12

Theo bi ra ta c hpt :
(tha mn k)
( x 3)( y 2) xy 45 y 5
Vy : chiu di = 12m, chiu rng = 5m
Cu 4 :
1. Theo tnh cht tip tuyn vung gc vi bn knh
ti tip im ta c : AMO ANO 90O
VAMO vung ti M A, M , O thuc ng trn
ng knh AO ( V AO l cnh huyn)
VANO vung ti N A, N, O thuc ng trn
ng knh AO (V AO l cnh huyn)
Vy: A, M, N, O cng thuc ng trn ng knh AO
Hay t gic AMNO ni tip ng trn ng knh AO
2. V I l trung im ca BC (theo gt) OI BC (tc)
VAIO vung ti I A, I, O thuc ng trn
ng knh AO (V AO l cnh huyn)
Vy I cng thuc ng trn ng knh AO (pcm)
3. Ni M vi B, C.

Xt VAMB &VAMC c MAC


chung
1

MCB
AMB s MB
2
AB AM

AB. AC AM 2 (1)
VAMB ~VACM (g.g)
AM
AC

Xt VAKM &VAIM c MAK


chung
AIM AMK (V: AIM ANM cng chn
AM
v AMK ANM )
119

AK AM

AK . AI AM 2
(2)
AM
AI
T (1) v (2) ta c: AK.AI = AB.AC (pcm)
Cu 5:
* Tm Min A
Cch 1:
2
x y x 2 2 xy y 2 1
Ta c:
2
x y x 2 2 xy y 2 0
1
1
2
2
2
2
Cng v vi v ta c: 2 x y 1 x y A
2
2
1
1
Vy Min A = . Du = xy ra khi x = y =
2
2
Cch 2
T x y 1 x 1 y Thay vo A ta c :
1
1 1
2
A 1 y y 2 2 y 2 2 y 1 2( y ) 2 y
2
2 2
1
Du = xy ra khi : x = y =
2
1
1
Vy Min A = Du = xy ra khi x = y =
2
2
* Tm Max A
x 2 x
0 x 1

x2 y2 x y 1
T gi thit suy ra
2
0 y 1 y y
VAMK ~VAIM (g.g)

Vy : Max A = 1 khi x = 0, y
GII CU 05
THI VO LP 10 MN TON BC NINH
2012-2013
=====================================
CU 05 :
Cho cc s x ; y tho mn x 0; y 0 v x+ y = 1
.Tm gi tr ln nht v gi tr nh nht ca biu thc A = x2 + y2
I- TM GI TR NH NHT
CCH 01 :
a) Tm gi tr nh nht ca biu thc A .
Ta c x + y = 1 nn y = - x + 1 thay vo A = x2 + y2 ta c :
x2 + ( -x + 1)2 - A = 0 hay 2x2 - 2x + ( 1- A) = 0 (*)
do biu thc A tn ti gi tr nh nht v gi tr ln nht khi v ch khi phng trnh (*) c nghim hay
1
.Vy gi tr nh nht ca biu thc A l 1 khi phng trnh
2
2
1
1
(*) c nghim kp hay x =
m x + y = 1 th y =
. Vy Min A = 1/2 khi x = y = 1/2 ( t/m)
2
2
' 0 1 21 A 0 2 A 1 0 A

b) Tm gi tr ln nht ca biu thc A .


CCH 02 :
a) Tm gi tr nh nht ca biu thc A .
Theo Bt ng thc Bunhia ta c 1 = x + y hay
120

1= (x + y)2 2 x 2 y 2 x 2 y 2

1
. Vy gi tr nh nht ca biu thc A l 1/2 khi x = y m x + y =1 hay x
2

=y = 1/2 ( t/m)
b) Tm gi tr ln nht ca biu thc A .
CCH 03 :
a) Tm gi tr nh nht ca biu thc A .
Khng mt tnh tng qut ta t

x 1 m

ym

vi 0 m 1

M A= x2 + y2 . Do A = ( 1- m)2 + m2 hay A= 2m2 - 2m +1


hay 2A = (4m2 - 4m + 1) + 1 hay 2A = (2m- 1)2 + 1 hay A

2m 1 2
2

1 1
.

2 2

Vy gi tr nh nht ca biu thc A l 1/2 khi m= 1/2 hay x = y = 1/2.


b) Tm gi tr ln nht ca biu thc A.
CCH 04 :
a) Tm gi tr nh nht ca biu thc A .
Ta c A = x2 + y2 = ( x+ y)2 - 2xy = 1 -2xy ( v x + y =1 )
m xy

x y 2
4

xy

1
1
1
1
2 xy
1 2 xy A .
4
2
2
2

Vy gi tr nh nht ca biu thc A l 1/2 khi x = y = 1/2.


b) Tm gi tr ln nht ca biu thc A.
CCH 05 :
a)Tm gi tr nh nht ca biu thc A .
Xt bi ton ph sau : Vi a , b bt k v c ; d > 0 ta lun c :
2
a b
a 2 b 2 a b

(*) , du = xy ra khi

c d
c
d
cd

Tht vy : c

x
2

b
y

a b 2 a b a b

.P DNG
Cho a = x v b = y ,t (*) c : A= x2 + y2 =
Nn A

x 2 y 2 x y

1
1
2

x y

(PCM)

m x+ y =1

1
.Vy gi tr nh nht ca biu thc A l 1/2 khi x = y = 1/2.
2

b) Tm gi tr ln nht ca biu thc A .


CCH 06 :
a)Tm gi tr nh nht ca biu thc A .
Ta c A = x2 + y2 hay xy =

1 A
(*) m x + y =1 (**)
2

121

x y 1

Vy t (*) ;(**) c h phng trnh


1 A
xy

2
x 0; y 0 1 21 A 0 A

,h ny c nghim

1
. Vy gi tr nh nht ca biu thc A l 1/2 khi x+ y =1 v x2 + y2
2

1
hay x = y = 1/2.
2

b) Tm gi tr ln nht ca biu thc A .


CCH 07 :
a)Tm gi tr nh nht ca biu thc A .
Ta c A = x2 + y2 = x2 + y2 + 1 - 1 m x + y =1 nn A = x2 + y2 - x - y -1

1
4

1
4

1
2

2
2
Hay A = x x y y

1
. Vy gi tr nh nht ca biu thc A l 1/2 khi x = y = 1/2.
2

b) Tm gi tr ln nht ca biu thc A .


CCH 08 :
a)Tm gi tr nh nht ca biu thc A .

x2 y2 x2 y2
x2
y2
x y x y

Ta c A= x + y =
1
x y
x y x y 2 x y
2
1
M x + y =1 nn A . Vy gi tr nh nht ca biu thc A l 1/2. khi x = y = 1/2.
2
2

b)Tm gi tr ln nht ca biu thc A .


CCH 09 :
a)Tm gi tr nh nht ca biu thc A .
Ta c x + y = 1 l mt ng thng , cn x2 + y2 = A l mt ng trn c tm l gc to O bn kn
A m x 0; y 0 thuc gc phn t th nht ca ng trn trn . Do tn ti cc tr th
khong cch t O n ng thng x + y =1 phi nh hn hay bng bn kn ng trn hay A

1
. Vy
2

gi tr nh nht ca biu thc A l 1/2 khi x =y = 1/2.


b)Tm gi tr ln nht ca biu thc A .
CCH 10 :
a)Tm gi tr nh nht ca biu thc A .
1 1
1
. Vy chng minh A
2 2
2
1
2
2
vi A = x2 + y2 th ta ch cn chng minh x y x y .
2

Ta c x + y =1 x y

Tht vy :
2
2
Ta c x y x y

1
Hay x
2

y
2

1
0
2
2

0 ( lun ng ) Vy A

1
. Vy gi tr nh nht ca biu thc A l 1/2 khi x =
2

y =1/2.
b)Tm gi tr ln nht ca biu thc A .
CCH 11 :
a)Tm gi tr nh nht ca biu thc A .
122

Khng mt tnh tng qut ta t

x 2m
1 m 2
y m 1

.Do A = x2 + y2 hay (2-m)2 + (m-1)2 - A =0 hay 2m2 - 6m +5 = A


Hay A

2 m 3 2
2

1 1
.
2 2

Vy gi tr nh nht ca A l 1/2 khi x = y = 1/2.


b)Tm gi tr ln nht ca biu thc A .
CCH 12 :
a)Tm gi tr nh nht ca biu thc A .
Khng mt tnh tng qut ta t

x 3 m
2m3
y m2

.Do A = x2 + y2 hay (3-m)2 + (m-2)2 - A =0 hay 2m2 - 10m +13 = A


Hay A

2m 5 2
2

1 1
.
2 2

Vy gi tr nh nht ca A l 1/2 khi x = y = 1/2.


b)Tm gi tr ln nht ca biu thc A .
CCH 13 :
a)Tm gi tr nh nht ca biu thc A .
Ta c x + y =1 hay (x+1) + (y +1) = 3 m A = x2 + y2 hay
A = (x2 + 2x + 1) + ( y2 + 2y +1) - 4 hay A = (x+1)2 + ( y+1)2 - 4

,do ta t

a x 1 a 1

b y 1 b 1

. Khi ta c bi ton mi sau :

Cho hai s a , b tho mn a 1; b 1 v a + b =3 . Tm gi tr nh nht ca biu thc A = a2 + b2 - 4


Tht vy : Ta c A = a2 + b2 - 4 = (a+b)2 - 2ab - 4 = 5 - 2ab ( v a+b=3)
Mt khc theo csi c : ab

a b 2
4

1
9
do A . Vy gi tr nh nht ca biu thc A l 1/2 khi x = y =
2
4

1/2.
b)Tm gi tr ln nht ca biu thc A .
CCH 14 :
a)Tm gi tr nh nht ca biu thc A .
Khng mt tnh tng qut ta t

x am
bma
y mb

( vi a > b v a - b =1 hay a = b+ 1 hay a > b )


.Do A = x2 + y2 hay (a-m)2 + (m-b)2 - A =0 hay
2m2 - 2m (a+b) +(a2 + b2) = A hay
123

Hay 2 A 2m a b 2 2 a 2 b 2 a b 2 A

2m a b 2 1 1
2

(V a - b= 1)
Vy gi tr nh nht ca A l 1/2 khi x = y = 1/2.
b)Tm gi tr ln nht ca biu thc A .
CCH 15 :
a)Tm gi tr nh nht ca biu thc A .
Ta c x + y =1 hay y = 1 - x m y 0 0 x 1
Do x2 + y2 - A = 0 hay 2 x2 - 2x +( 1 - A ) = 0 .
Khi ta c bi ton mi sau :
Tm A phng trnh 2 x2 - 2x +( 1 - A ) = 0 (*) c nghim 0 x1 x 2 1
Vi x1 ; x2 l nghim ca phng trnh (*)
Tht vy phng trnh (*) c nghim

124

' 0

x10 S0 S0

x 12 0 x20 P0 P0 1
0x 21 1 A 1
x 21 1 x11 S2 '0 2

x 21 P1 S2

P 1

Vy gi tr nh nht ca biu thc A l 1/2 khi x =y = 1/2.


125

b)Tm gi tr ln nht ca biu thc A .


Vy theo trn ta c gi tr ln nht ca biu thc A l 1
khi x = 0 v y = 1 hoc x= 1 v y = 0 .
II- TM GI TR LN NHT
CCH 01 :
Vy theo trn ta c gi tr ln nht ca biu thc A l 1
khi x = 0 v y = 1 hoc x= 1 v y = 0
CCH 02 :
Ta c A = x2 + y2 hay xy =

1 A
(*) v x + y =1 m x 0; y 0 xy 0
2

Do theo (*) c A 1 . Vy gi tr ln nht ca biu thc A l 1


khi x = 0 v y = 1 hoc x= 1 v y = 0
CCH 03 :
2
Khng mt tnh tng qut ta t

x sin 0

y cos 2 0

Do A = sin 4 cos 4 1 2 sin . cos 2 1 .


Vy gi tr ln nht ca biu thc A l 1
khi x = 0 v y = 1 hoc x= 1 v y = 0

S GD & T H TNH

K THI TUYN SINH VO LP 10 THPT


NM HC 2012 2013
Mn thi: TON
Ngy thi : 28/6/2012
Thi gian lm bi : 120 pht

CHNH THC
( thi c 1 trang)
M 01
Cu 1 (2im)
a) Trc cn thc mu ca biu thc:

5
.
6 1

126

b) Gii h phng trnh:

2x y 7
.

2
y

Cu 2 (2im)

4a
a
a 1

.
vi a >0 v a 1 .

a2
a 1 a a

Cho biu thc: P

a) Rt gn biu thc P.
b) Vi nhng gi tr no ca a th P = 3.
Cu 3 (2im)
a) Trong mt phng ta Oxy, ng thng y = ax + b i qua im M(1 ; 2) v song song vi ng
thng y = 2x + 1. Tm a v b.
b) Gi x1, x2 l hai nghim ca phng trnh x2 + 4x m2 5m = 0. Tm cc gi tr ca m sao cho: |x1
x2| = 4.
Cu 4 (3im)
Cho tam gic ABC c ba gc nhn, ni tip ng trn tm O. Hai ng cao AD, BE ct nhau ti H (D
BC, E AC) .
a) Chng minh t gic ABDE ni tip ng trn.
b) Tia AO ct ng trn (O) ti K ( K khc A). Chng minh t gic BHCK l hnh bnh hnh.
c) Gi F l giao im ca tia CH vi AB. Tm gi tr nh nht ca biu thc:
AD BE CF
Q

.
HD HE HF
Cu 5 (1im)
Tm tt c cc gi tr ca tham s m phng trnh sau v nghim:
x2 4x 2m|x 2| m + 6 = 0.
HNG DN CHM THI
Ni dung
5
5( 6 1)

6 1 ( 6 1)( 6 1)

Cu
a) Ta c:

1
b) Ta c:

5( 6 1) 5( 6 1)

6 1
6 1
5

2x y 7 4x 2y 14

x 2y 1 x 2y 1
5x 15
x 3

x 2y 1 y 1

a) Vi 0 a 1 th ta c: P

0,5
0,5
0,5
0,5

4a
a
a 1 4a 1 a 1

. 2

2
a
a
a 1 a a
a

b) Vi 0 a 1 th P = 3

im

4a 1
a2

4a 1
3 3a 2 4a 1 3a 2 4a 1 0
2
a

0,5
0,5
0,5

127

a = 1 (loi) hoc a

1
(tha mn k).
3

a) ng thng y = ax + b song song vi ng thng y = 2x +1 nn:


a = 2, b 1.
V ng thng y = 2x + b i qua im M(1 ; 2) nn ta c pt:
2(-1) + b = 2 b = 4 (tha mn b 1). Vy a = 2, b = 4
b) Ta c : ' 4 m 2 5m (m 1)(m 4) . phng trnh c 2 nghim x1, x2 th ta
c: ' 0 m 4 hoc m 1 (*)

b
c
4 v x1.x 2 m 2 5m.
a
a
2
2
Ta c: x1 x 2 4 (x1 x 2 ) 16 (x1 x 2 ) 4x1.x 2 16
Theo nh l Vi-et, ta c: x1 x 2

16 4(m 2 5m) 16 m 2 5m 0 m = 0 hoc m = 5


Kt hp vi k(*), ta c m = 0 , m = 5 l cc gi tr cn tm.
a) V AD v BE l cc ng cao nn ta c:

ADB
AEB
90o

A
E

F
H
O
B
D

Hai gc ADB,
cng nhn cnh AB di
AEB
mt gc 90o nn t gic ABDE ni tip ng
trn.

b) Ta c: ABK
ACK
90o (gc ni tip chn na
ng trn) CK AC, BK AB (1)
Ta c H l trc tm ca tam gic ABC nn:
C BH AC, CH AB (2)

K
4

T (1) v (2), suy ra: BH // CK, CH // BK.


Vy t gic BHCK l hnh bnh hnh (theo nh
ngha)

t SBHC = S1, SAHC = S2, SAHB = S3, SABC = S. V ABC nhn nn trc tm H nm bn
trong ABC , do : S = S1 + S2 + S3 .
AD SABC S
BE SABC S
CF SABC S

(1),

(2),

(3)
Ta c:
HD SBHC S1
HE SAHC S2
HF SAHB S3

0,5
0,5
0,5
0,25
0,25
0,25
0,25
0,5
0,5

0,5

0,5
0,25
0,25

Cng v theo v (1), (2), (3), ta c:

1 1 1
AD BE CF S S S

S

HD HE HF S1 S2 S3
S
1 S2 S3

p dng bt ng thc Csi cho 3 s dng, ta c:


1 1 1
3

S S1 S2 S3 3 3 S1.S2 .S3 (4) ;
(5)
S1 S2 S3 3 S1.S2 .S3
Nhn v theo v (4) v (5), ta c: Q 9 . ng thc xy ra S1 S2 S3 hay H l
trng tm ca ABC , ngha l ABC u.
5

Ta c: x2 4x 2m|x 2| m + 6 = 0 (*). t x 2 t 0 th pt (*) tr thnh: t2 2mt +


2 m = 0 (**), '(t) m 2 m 2 (m 1)(m 2)

0,25

0,25
0,25
128

pt (*) v nghim th pt(**) phi v nghim hoc c 2 nghim t1, t2 sao cho: t1 t 2 0

0,25

Pt (**) v nghim '(t) 0 (m 1)(m 2) 0 2 m 1 (1)


Pt (**) c 2 nghim t1, t2 sao cho: t1 t 2 0 . iu kin l:
' 0
' 0

2m 0 m 0 m 2 (2)
2m 0 m 2

0,25

Kt hp (1) v (2), ta c k cn tm ca m l: m <1.

0,25

Ch : Mi cch gii ng u cho im ti a, im ton bi khng quy trn.

S GIO DC V O TO
BNH DNG
chnh thc

Bi 1 (1 im): Cho biu thc: A =

K THI TUYN SINH LP THPT


Nm hoc 2012 2013
Mn thi: Ton
Thi gian lm bi: 120 pht
(Khng k thi gian pht )

2
3
50 x
8x
5
4
129

1/ Rt gn biu thc A
2/ Tnh gi tr ca x khi A = 1
Bi 2 (1,5 im):
1/ V th (P) hm s y =

x2
2

2/ Xc nh m ng thng (d): y = x m ct (P) ti im A c honh bng 1. Tm tung ca im A


Bi 3 (2 im):
2x y 4
3x y 3

1/ Gii h phng trnh:

2/ Gii phng trnh: x4 + x2 6 = 0


Bi 4 (2 im): Cho phng trnh x2 2mx 2m 5 = 0 (m l tham s)
1/ Chng minh rng phng trnh lun c hai nghim phn bit vi mi gi tr ca m
2/ Tm m x1 x2 t gi tr nh nht (x1; x2 l hai nghim ca phng trnh)
Bi 5 (3,5 im): Cho ng trn (O) v im M ngoi ng trn. Qua M k cc tip tuyn MA, MB v
ct tuyn MPQ (MP < MQ). Gi I l trung im ca dy PQ, E l giao im th 2 gia ng thng BI v
ng trn (O). Chng minh:
1/ T gic BOIM ni tip. Xc nh tm ca ng trn ngoi tip t gic
2/ BOM = BEA
3/ AE // PQ
4/ Ba im O; I; K thng hng, vi K l trung im ca EA
HNG DN GII:
Ni dung
Bi 1 (1 im):
1/ KX: x 0

2
3
50 x
8x
5
4
2
3
25.2 x
4.2 x
=
5
4
3
2x
= 2 2x
2
1
2x
=
2
1
2x
Vy vi x 0 thi A =
2
1
2x = 1
2/ Khi A = 1
2
2x = 2
A=

130

2x = 4
x = 2 (Tha iu kin xc nh)
Vy khi A = 1 gi tr ca x = 2
Bi 2 (1,5 im):
x2
1/ V th (P) hm s y =
2
-Bng gi tr
x
-4
-2
0
2
4
2
x
y=
8
2
0
2
8
2
- th (P) l ng parabol nh O(0; 0) nm pha trn trc honh, nhn trc tung lm trc i xng v i
qua cc im c ta cho trong bng trn.

2/ Cch 1.
V (d) ct (P) ti im A c honh bng 1 nn x = 1 tha mn cng thc hm s (P) => Tung ca im
12 1
A l: yA =
=
2 2
1
1
A(1; ) (d) nn
=1m
2
2
1
1
m=1
=
2
2
1
1
Vy vi m =
th (d): y = x m ct P ti im A c honh bng 1. Khi tung yA =
2
2
Cch 2
Ta c phng trnh honh giao im ca (d) v (P) l:
x2
= x m x2 2x + 2m = 0 (*)
2
(d) ct (P) ti im A c honh bng 1 th phng trnh (*) c nghim bng 1
1
12 2.1 + 2m = 0 m =
2
1
12 1
Vy vi m =
th (d): y = x m ct P ti im A c honh bng 1. Khi tung yA =
=
2
2 2
Bi 3 (2 im):
1/ Gii h phng trnh
2x y 4
x 1
x 1
x 1

3x y 3
3x y 3
3.(1) y 3
y 6
131

Vy h phng trnh c nghim duy nht (-1; -6)


2/ Gii phng trnh
x4 + x2 6 = 0
(1)
2
t x = t (t 0)
Phng trnh (1) tr thnh: t2 + t 6 = 0 (2)
Ta c = 12 4.1.(-6) = 25
1 25
1 25
Phng trnh (2) c hai nghim t1 =
= 2 (nhn) ; t2 =
= -3 (loi)
2.1
2.1
Vi t = t1 = 2 => x2 = 2 x = 2
Vy phng trnh cho c hai nghim x1 = 2 ; x2 = - 2
Bi 4 (2 im): Cho phng trnh x2 2mx 2m 5 = 0 (m l tham s)
1/ Ta c = (-m)2 1 (-2m 5)
= m2 + 2m + 5
= (m + 1)2 + 4
2
V (m + 1) 0 vi mi m
(m + 1)2 + 4 > 0 vi mi m
Hay > 0 vi mi m
Vy phng trnh cho lun c hai nghim phn bit vi mi m
2/ V phng trnh cho lun c hai nghim phn bit vi mi m
x1 x2 2m

(theo nh l Vi-et)
x1.x2 2m 5
t A = x1 x2
A2 = ( x1 x2 )2 = x12 2x1x2 + x22 = (x1 + x2)2 4x1x2
A2 = (2m)2 4(-2m 5) = (2m)2 + 8m + 20
= (2m)2 + 2. 2m. 2 + 4 + 16 = (2m + 2)2 + 16 16
Gi tr nh nht ca A2 = 16
Gi tr nh nht ca A l 4 khi 2m + 2 = 0 m = -1
Vy vi m = -1 th x1 x2 t gi tr nh nht l 4
Bi 5 (3,5 im):
1/ Ta c MB l tip tuyn ca (O) (gt)
OB MB
OBM = 900
B thuc ng trn ng knh OM (1)
Ta c IQ = IP (gt)
OI QP (Tnh cht lin h gia ng knh v dy cung)
OIM = 900
I thuc ng trn ng knh OM (2)
T (1) v (2) => BOIM ni tip ng trn ng knh OM
2/ Ta c BOM = AOM (Tnh cht hai tip tuyn ct nhau)
1
BOM = BOA
2
m BOA = SAB
1
BOM = SAB
2
132

1
SAB (nh l gc ni tip)
2
BOM = BEA
3/ Ta c: T gic BOIM ni tip (Chng minh trn)
BOM = BIM (Cng chn BM)
m BOM = BEA (Chng minh trn)
BIM = BEA
Mt khc BIM v BEA l hai gc v tr ng v
AE // PQ
4/ Ta c OI QP v AE // PQ (chng minh trn);
OI AE (3)
m KE = KA (gt)
OK AE (tnh cht lin h gia ng knh v dy cung) (4)
T (3) v (4), ta thy qua im O c hai ng thng OI v OK cng song song vi AE
OI v OK phi trng nhau
Ba im O, I, K thng hng
Ta li c BEA =

S GIO DC V O TO
THI BNH
chnh thc

K THI TUYN SINH LP 10 THPT


NM HC 2012 2013
Mn thi: TON

Thi gian lm bi: 120 pht, khng k thi gian giao

Bi 1. (2,0 im)
1) Tnh: A

1
9 4 5.
5 2
133

2) Cho biu thc: B

2(x 4)

x3 x 4

x 1

8
vi x 0, x 16.
x4

a. Rt gn B.
b. Tm x gi tr ca B l m t s nguyn.
Bi 2. (2,0 im)
Cho phng trnh: x2 4x + m + 1 = 0 (m l tham s).
1) Gii phng trnh vi m = 2.
2) Tm m phng trnh c hai nghi m tri du (x1 < 0 < x2). Khi nghi m no c gi
tr tuy t i ln hn?
Bi 3. (2,0 im):
Trong m t phng to Oxy cho parabol (P): y = -x2 v ng thng (d): y = mx + 2 (m
l tham s).
1) Tm m (d) ct (P) ti m t im duy nht.
2) Cho hai im A(-2; m) v B(1; n). Tm m, n A thu c (P) v B thu c (d).
3) Gi H l chn ng vung gc k t O n (d). Tm m di on OH ln nht.
Bi 4. (3,5 im)
Cho ng trn (O), dy cung BC (BC khng l ng knh). im A di ng trn
cung nh BC (A khc B v C; di on AB khc AC). K ng knh AA ca ng
trn (O), D l chn ng vung gc k t A n BC. Hai im E, F ln lt l chn
ng vung gc k t B, C n AA. Chng minh rng:
1) Bn im A, B, D, E cng nm trn m t ng trn.
2) BD.AC = AD.AC.
3) DE vung gc vi AC.
4) Tm ng trn ngoi tip tam gic DEF l m t im c nh.
Bi 5.(0,5 im):
Gii h phng trnh:
x 4 x 3 3x 2 4y 1 0

.
x 2 4y 2
x 2 2xy 4y 2

x 2y

2
3

P N

Ni dung
5 2
1.
A
( 5 2) 2 5 2 5 2 4.
(0,5)
5 4
2.
a. (1 )
(1,5) Vi x 0, x 16, th:
2(x 4)
x
8
2x 8 x ( x 4) 8( x 1)

B
( x 1)( x 4)
x 1
x4
( x 1)( x 4)
2x 8 x 4 x 8 x 8
3x 12 x

( x 1)( x 4)
( x 1)( x 4)

im
0,5

0,25
0,25

134

3 x ( x 4)
3 x

( x 1)( x 4)
x 1
3 x
V y B
vi x 0, x 16.
x 1
b. (0,5 )

D thy B 0 (v
Li c: B 3

0,25
0,25

x 0) .

3
3 (v
x 1

3
0 x 0, x 16) .
x 1

Suy ra: 0 B < 3 B {0; 1; 2} (v B Z).


- Vi B = 0 x = 0;
3 x
1
1 3 x x 1 x .
- Vi B = 1
4
x 1
3 x
2 3 x 2( x 1) x 4.
- Vi B = 2
x 1
1
V y B Z th x {0; ; 4}.
4

0,25

0,25

Bi 2.
Ni dung
m = 2, phng trnh cho thnh: x 4x + 3 = 0.
1.
Phng trnh ny c a + b + c = 1 4 + 3 = 0 nn c hai nghi m
: x1 = 1; x2 = 3.
(1,0) V y vi m = 2 th phng trnh cho c hai nghi m phn bi t: x= 1; x = 3.

1
2
Phng trnh cho c hai nghi m tri du ac < 0 m + 1 < 0 m < -1.
x1 x 2 4
2.
Theo nh l Vi-et, ta c:
.
x
x

1
2
(1,0)
Xt hi u : |x1| - |x2| = -x1 x2 = -4 < 0 (v x1 < 0 < x2) |x1| < |x2|.
V y nghi m x1 c gi tr tuy t i nh hn nghi m x2.
2

im
0,5
0,5
0,5
0,25
0,25

Bi 3. (2,0 im):
Ni dung
(d) ct (P) ti m t im duy nht Phng trnh honh ca (d) v (P):
-x2 = mx + 2 x2 + mx + 2 = 0 c nghi m duy nht.
1.
= m2 8 = 0 m = 2 2.
(0,75)
V y gi tr m cn tm l m = 2 2.
2.
m ( 2) 2
A (P)
m 4

(0,75)
n 2
B (d)
n m 2
V y m = -4, n = -2.

im
0,25
0,25
0,25
0,5
0,25
135

- Nu m = 0 th (d) thnh: y = 2 khong cch t O n (d) = 2 OH = 2


(Hnh 1).

0,25

3.
(0,5)

- Nu m 0 th (d) ct trc tung ti im A(0; 2) v ct trc honh ti im


2
B( ; 0) (Hnh 2).
m
2
2

OA = 2 v OB =
.
m |m|

1
1
1
1 m2 m 2 1

OAB vung ti O c OH AB
OH 2 OA 2 OB2 4 4
4
2
OH
. V m2 + 1 > 1 m 0 m 2 1 1 OH < 2.
2
m 1
So snh hai trng hp, ta c OHmax = 2 m = 0.

0,25

Bi 4. (3,5 im)
Ni dung

1.
V ADB
AEB
900 bn im A, B, D, E cng thu c ng trn ng
(0,5) knh AB.
2.
Xt ADB v ACA c:
(1,0)

ADB
ACB
900 ( ACB
900 v l gc n i tip chn na ng trn);

'C (hai gc n i tip cng chn cung AC)


ABD
AA
ADB ~ ACA (g.g)
AD BD

BD.AC = AD.AC (pcm).


AC A 'C

im
0,5

0,5

0,5

136

Gi H l giao im ca DE vi AC.

T gic AEDB n i tip HDC


BAE
BAA
'.

l hai gc n i tip ca (O) nn:


BAA
' v BCA
3.
(1,25

4.
(0,5

0,25

1
1

BAA
' sBA
' ; BCA
sBA
.
2
2
1
1
1

' BCA
sBA
' sBA
sABA
' 900 (do AA l ng knh)
BAA
2
2
2

Suy ra: HDC


HCD
BAA
' BCA
90 0 CHD vung ti H.
Do : DE AC.

0,25

Gi I l trung im ca BC, K l giao im ca OI vi DA, M l giao im ca


EI vi CF, N l im i xng vi D qua I.

0,25

0,25
0,25

Ta c: OI BC OI // AD (v cng BC) OK // AD.


ADA c: OA = OA (gt), OK // AD KD = KA.
DNA c ID = IN, KD = KA IK // NA; m IK BC (do OI BC)
NA BC.

T gic BENA c BEA


' BNA
' 900 nn n i tip c ng trn
'B ENB

EA
.

'B AA
'B ACB

Ta li c: EA
(hai gc n i tip cng chn cung AB ca (O)).

ENB
NE // AC (v c hai gc v tr ng v bng nhau).
ACB

M DE AC, nn DE EN

(1)

Xt IBE v ICM c:

(i nh)
EIB
CIM

IB = IC (cch dng)

(so le trong, BE // CF (v cng AA))


IBE
ICM

137

IBE = ICM (g.c.g) IE = IM


EFM vung ti F, IE = IM = IF.
T gic DENM c IE = IM, ID = IN nn l hnh bnh hnh

(2)

T (1) v (3) suy ra DENM l hnh ch nh t IE = ID = IN = IM

0,25

ID = IE = IF. Suy ra I l tm ng trn ngoi tip DEF.


I l trung im ca BC nn I c nh.
V y tm ng trn ngoi tip tam gic DEF l m t im c nh.

Bi 5.(0,5 im):
Ni dung

im

T (2) suy ra x + 2y 0.
p dng bt ng thc Bunhiacopxki, ta c:
2(x 2 4y 2 ) (12 12 )[x 2 (2y) 2 ] (x 2y) 2
x 2 4y 2

2
Du bng xy ra x = 2y.

(x 2y) 2 x 2y

4
2

M t khc, d dng chng minh c:


Th t v y,

(3)

x 2 2xy 4y 2 x 2y

3
2

(4)

0,25

x 2 2xy 4y 2 x 2y
x 2 2xy 4y 2 (x 2y) 2
(do c hai v

3
2
3
4

u 0)
4(x2 + 2xy + 4y2) 3(x2 + 4xy + 4y2) (x 2y)2 0 (lun ng x, y).
Du bng xy ra x = 2y.
x 2 4y 2
x 2 2xy 4y 2

x 2y .
2
3
Du bng xy ra x = 2y.
Do (2) x = 2y 0 (v x + 2y 0).
Khi , (1) tr thnh: x4 x3 + 3x2 2x 1 = 0 (x 1)(x3 + 3x + 1) = 0
1
x = 1 (v x3 + 3x + 1 1 > 0 x 0) y .
2
1
V y nghi m
ca h cho l (x = 1; y = ).
2
T (3) v (4) suy ra:

0,5

138

S GD & T TR VINH
---------------------------------

K THI TUYN SINH VO LP 10 THPT


NM HC 2011 2012
---------------------------Mn thi: TON
Thi gian lm bi: 120 pht ( khng k thi gian giao

chnh thc

).
Bi 1: ( 1,5 im )
Cho biu thc

A=

1
1

1
x 1
x 1

1) Rt gn biu thc A.
2) Tm x A = - 3
Bi 2: ( 1,0 im )
Gii h phng trnh:
x 2 y 3 13

x 3 y 2 5 6
Bi 3: ( 2,5 im )
x
x2
v y = 1
2
2
1).V th ca hai hm s ny trn cng mt mt phng ta .
Cho hai hm s y

b) Tm ta giao im ca hai th .
Bi 4: ( 2,0 im )
Cho phng trnh: x2 2(m + 4 )x + m2 8 = 0 (1) , vi m l tham s.
1) Tm m phng trnh (1) c hai nghim phn bit l x1 v x2 .
2) Tm m x1 + x2 3x1x2 c gi tr ln nht.
Bi 5: ( 3,0 im )
T mt im M ngoi ng trn O bn knh R, v hai tip tuyn MA, MB n ng trn O bn
knh R ( Vi A, B l hai tip im ). Qua A v ng thng song song vi MB ct ng trn tm O ti E.
on ME ct ng trn tm O ti F. Hai ng thng AF v MB ct nhau ti I.
a) Chng minh t gic MAOB ni tip ng trn.
b) Chng minh IB2 = IF.IA.
c) Chng minh IM = IB.
HNG DN CHM THI
BI
Bi 1
(1,5 im)

P N
1) A

x 1
x 1

x 1 ( x 1) x 1
x 1

( iu kin: x 0, x 1 )

IM
0,25

0,5
139

x 1
3
x 1

2) C A = -3

0,25
0,25

iu kin x 1

0,25

1
x
2
Bi 2
(1.0 im )

0,25

2 x y 6 13 2

H Pt

3 x y 6 15 2
0,25
x2 2
0,25

y3 3
0,25

Vy h phng trnh cho c nghim ( 2 2;3 3 )


Bi 3
( 2,5 im)

x2
1) ( P) : y
2
Tp xc nh D = R

x
2

x
y
2

-2
-2

-1
1
2

0
0

1
1
2

1
x 1
2
Cho x = 0 y = -1, A( 0;-1)
Cho x = 2 y = 0,
B( 2;0)
ng thng (d) i qua hai im A( 0;-1), B( 2;0)
(d):

0,25

-2

y =

0,25

th
0.25

140

2) Phng trnh honh giao im ca ( d ) v ( P ) c :


x2 x
1
2 2

0.25

0.25
0.25

x2 x 2 0
x 1

x 2
1
x 1 y
Vi
2
x = -2 y = -2

0.25

Vy (d) ct (P) ti hai im M ( 1;


Bi 4
(2,0 im)

1
) , N ( -2; -2)
2
0.25
0.25

1) / 8m 24
Phng trnh (1) c hai nghim phn bit
/ f 0 8m 24 f 0
m f 3
2) C : x1 + x2 3x1.x2 = -3m + 2m + 32

0.5
0,25
0.5

1
97 97
3m

3
3
3

1
Du = xy ra m
3
1
Vy m th x1 + x2 3x1x2 t GTLN
3
Bi 5

0,25

Ve hnh:
A

(3,0 im)
E

F
0

M
I

B
1) C MA l tip tuyn
Nn OA MA

OAM
900

Tng t OBM
900

OAM
OBM
1800
T gic MAOB ni tip ng trn c ng knh l OM.
Xt IBA v IFB

C : BIA
l gc chung

0,25

0,5
0,25

141

( cng bng
IAB
IBF

1
)
s o BF
2

IBA ng dng IFB


IB IA

IF IB
IB 2 IF .IA(1)
3) Ta c : AE // MB ( gt)

Nn
IMF
MEA

M MEA
FAM

IMF
FAM
Xt IMF v IAM

C IAM
l gc chung
IMF IAM

( Chng minh trn )


IMF ng dng IAM
IM
IA

IF IM
IM 2 IA.IF (2)
T (1) v ( 2 ) IB2 = IM2
IB = IM (pcm)

0.25

0.25
0.25

0.25

0.25
0.5

142

chnh thc

143

S GIO DC & O TO
TNH KIN GIANG
-------------- CHNH THC
( thi c 01 trang)

K THI TUYN SINH VO LP 10 TRNG CHUYN


NM HC 2012-2013
-------------------Mn thi: TON (Khng chuyn)
Thi gian: 120 pht (Khng k thi gian giao )
Ngy thi: 25/6/2012

Bi 1. (1,5 im)
1/ Rt gn: A = (3 2 11)(3 2 11)
2/ Chng minh rng vi a khng m, a khc 1, b ty , ta c:

ab + a - b a- 1 b a +1

a- 1
1+ a

Bi 2. (1,5 im)
Cho (dm): y

1 m
x (1 m)(m 2)
m 2

1/ Vi gi tr no ca m th ng thng (d m): y

1 m
x (1 m)(m 2) vung gc vi ng thng
m 2

1
x 3
4
(Cho bit hai ng thng vung gc vi nhau khi v ch khi tch h s gc bng -1)
2/ Vi gi tr no ca m th (dm) l hm s ng bin.

(d): y

Bi 3. (3 im)
1/ Chng minh rng phng trnh sau c 2 nghi m phn bi t x1, x2 vi mi gi tr m:
x2 (m 1)x m 3 0. Xc nh cc gi tr ca m tha mn : x1x22 x2x12 3
2/ M t phng hp c 360 ch ngi v c chia thnh cc dy c s ch ngi bng nhau. Nu thm
cho mi dy 4 ch ngi v bt i 3 dy th s ch ngi trong phng khng thay i. Hi ban u s ch ngi
trong phng hp c chia thnh bao nhiu dy?
Bi 4. (1 im)
Cho tam gic ABC vung ti A, ng cao AH. Tnh chu vi tam gic ABC, bit rng:
CH = 20,3cm. Gc B bng 620. (Chnh xc n 6 ch s th p phn).
Bi 5. (3 im)
Cho ng trn (O, 4cm), ng knh AB. Gi H l trung im ca OA, v dy CD vung gc vi
AB ti H. Ly im E trn on HD (E H v E D), ni AE ct ng trn ti F.
a) Chng minh rng AD2 = AE . AF
b) Tnh di cung nh BF khi HE = 1 cm (chnh xc n 2 ch s th p phn)
c) Tm v tr im E trn on HD s o gc EOF bng 900
------ HT ------

144

P N KHNG CHUYN
BI

NI DUNG
2

1.1

A = (3 2 11)(3 2 11) (3 2)2 11 9 6 2 2 11 6 2


Vi a 0, a 1 v b ty ta c:

1.2

ab + a - b a- 1 b a( a-1)+( a-1) (b a +1)( a 1) b a 1

a- 1
(1 a)( a 1)
(1 + a)( a 1)
1 a
1 m
1
x (1 - m)(m +2) ; (d): y x 3
(dm): y
m +2
4
1- m 1
1 1- m = -4(m + 2) (vi m -2 v m 1 )
(dm) (d)
m +2 4
3m = 9 m = 3
1 m >0 m <1

1 m
m +2 >0 m >-2

2 <m <1
(dm) l hm s ng bin khi:
1 m <0 m >1
m +2

(loi)
m +2 <0 m <- 2
Phng trnh: x2 (m - 1)x m - 3 =0 c:

2.1

2.2

= [-(m 1)]2 4 . 1 (m 3) = m2 2m + 1 4m + 12 = (m2 6m + 9) + 4


= (m 3)2 + 4 > 0 vi m
V y phng trnh cho c 2 nghi m phn bi tx1, x2 vi mi m
3.1

x1 x2 m - 1
(I) .
x1x2 =m - 3

Theo nh l Vit ta c:

2
2
Theo ta c: x1x2 x2x1 3 x1x2 (x1 x2 ) 3 (1)
Thay h thc (I) vo (1) ta c: (m 1)(m 3) = 3 m2 4m = 0 m(m 4) = 0
m =0

m =4

3.2

2
2
V y vi m = 0 ho c m = 4 th phng trnh c 2 nghi m tha mn:x1x2 x2x1 3
Gi x (dy) l s dy gh lc u c chia t s ch ngi trong phng hp
(k:x N* v x > 3)
360
S ch ngi mi dy lc u:
(ch)
x
Do thm cho mi dy 4 ch ngi v bt i 3 dy v s ch ngi trong phng khng thay i nn ta c

phng trnh: (

360
+ 4)(x 3) = 360
x

x =18
x =-15 (loi)

x2 3x 270 = 0
4

V y lc u s ch ngi trong phng hp c chia thnh 18 dy.


= 900) c:
*Xt ABC ( A

$ C
B
900 C 900 620 280
145

= 900) c: AC = HC
*Xt AHC ( H
CosC

= 900) c: AB = AC.tanC = HC tanC


*Xt ABC ( A
CosC
AC
HC

V BC =
CosC Cos2C
*Chu vi tam gic ABC l:
HC
HC
HC
tanC +
AB + AC + BC =
+
CosC
CosC Cos2C
HC
1
20,3
1
(tanC +1 +
)
(tan280 1
) 61,254908 (cm)
=
0
CosC
CosC Cos28
Cos280
a. Chng minh: AD2 = AE . AF
AD
(lin hgia k v d
*Ta c: AB CD AC
y cung)

(cc gc nt chn cc cung tng ng bng nhau)


ADC
AFD
*Xt ADE v AFD c:

ADC AFD (cm trn)

A : gc chung

ADE ~ AFD (g-g)

AD AF

AD2 AE . AF
AE AD

b. Tnh di cung nho BF khi HE = 1cm (chnh xc n 2 ch s thp phn)


OA
2 (cm) (V H l trung im ca OA v OA = 4cm)
*Ta c: AH = OH =
2
HE 1

2.BAF
*Xt AHE ( H
900 ) c: tg HAE AH 2 BAF HAE 27 sBF
540
l
BF

.OA.n 2 . 540 1,88 (cm) (Vi n = s

BF 540 )
1800
1800

c. Tim vi tr cua im E trn oan HD s o cua goc EOF bng 900


*Xt EAO c: EH l ng cao (EH AB) cng l ng trung tuyn (v AH = OH) nn EAO

cn ti E EAH
O1 .
O2

O1

2
O

2
*M EAH BAF
(c ng ch
n cung BF)
2

O2

0
0
0
* EOF 90 O1 O2 90 (VO1 EOF O2 180 )
O2 900 3O2 1800
2

HE

O2 600 EAH 300 tan EAH AH (v EAH vung ti H)


2 3

HE =AH . tanEAH 2 . tan300


(cm)
3

146

V y khi im E cch H m t khong HE =

S GIO DC & O TO
TNH KIN GIANG
-------------- CHNH THC
( thi c 01 trang)

2 3

(cm) trn on HD th EOF


900
3

K THI TUYN SINH VO LP 10 THPT


NM HC 2012 2013
-------------------Mn thi: TON
Thi gian: 120 pht (Khng k thi gian giao )
Ngy thi: 06/7/2012

Bi 1. (1,5 im)
1) Rt gn biu thc A = 112 - 45 - 63 +2 20

2) Cho biu thc B = 1

x x
x x
1

, vi 0 x 1

1 x
1 x

a) Rt gn B
b) Tnh gi tr biu thc B khi x =

1
1 2

Bi 2. (1,5 im)
Cho ng thng (dm) : y = - x + 1 m2 v (D): y = x
1) V ng thng (dm) khi m = 2 v (D) trn cng h trc ta , nh n xt v 2 th ca chng.
2) Tm m d trc ta Ox, (D) v (dm) ng quy.
Bi 3. (1,5 im)
Trong t quyn gp ng h ngi ngheo, lp 9A v 9B c 79 hc sinh quyn gp c
975000 ng. Mi hc sinh lp 9A ng gp 10000 ng, mi hc sinh lp 9B ng gp 15000
ng. Tnh s hc sinh mi lp.
Bi 4. (1,5 im)
Cho phng trnh: x2 2(m 2)x m2 5m 4 0 (*)
1/ Chng minh rng vi m < 0 phng trnh (*) lun lun c 2 nghi m phn bi t x1, x2 .
2/ Tm m phng trnh (*) c hai nghi m phn bi t x1, x2 tha h thc

1 1

1
x1 x2

Bi 5. (4 im)
Cho na ng trn tm O ng knh AB v im C trn ng trn sao cho CA = CB. Gi
M l trung im ca dy cung AC; Ni BM ct cung AC ti E; AE v BC ko di ct nhau ti D.
a) Chng minh: DE . DA = DC . DB
b) Chng minh: MOCD l hnh bnh hnh
c) K EF vung gc vi AC. Tnh t s

MF
?
EF

d) V ng trn tm E bn knh EA ct ng trn (O) ti im th hai l N; EF ct AN ti I, ct


ng trn (O) ti im th hai l K; EB ct AN ti H. Chng minh: T gic BHIK n i tip c
ng trn.
147

------ HT ------

BI
1.1
1.2

P N
NI DUNG
A = 112 - 45 - 63 +2 20 4 7 - 3 5 - 3 7 +4 5
a) Vi 0 x 1 ta c:

x x

x x

1
1
B = 1
1 x
1 x

b) Ta c: x =
2.1

1
1 2

x( x 1)

1 x

7+ 5

x( x 1)
(1 + x)(1 - x) 1 x
x 1

2 1
2 1 B =1 - 2 1 2 - 2
2 1

(dm) : y = - x + 1 m2 v (D): y = x
*Khi m = 2 th (dm) tr thnh: y = -x 3
Xt (dm): y = x 3 ta c bng gi tr:
Xt (D): y = x ta c: x = 1 y = 1
* th ca (dm) v (D):
(dm

x
y

0
-3

-3
0
y

): y = -x - 3

(D): y = x

x
-3

-2

-1

-1

-2

-3

2.2

*Nh n xt: ng thng (D) v ng thng (dm) vung gc vi nhau v tch h s ca


chng bng -1
(dm) : y = - x + 1 m2 v (D): y = x
Ta c (D) ct Ox ti O. Ox, (D) v (dm) ng quy th (dm) phi i qua O khi :
1 m2 = 0 m = 1
V y m = 1 th Ox, (D) v (dm) ng quy.
Gi x l s hc sinh lp 9A (x N* v x < 79)
S hc sinh lp 9B l: 79 x (hc sinh)
Lp 9A quyn gp c: 10000x
(ng)
Lp 9B quyn gp c: 15000(79 x) (ng)
Do c hai lp quyn gp c 975000 ng nn ta c phng trnh:
10000x + 15000(79 x) = 975000
10x + 15(79 x) = 975 -5x = - 210 x = 42
148

V y lp 9A c 42 hc sinh; lp 9B c: 79 42 = 37 (hc sinh)


1/ Phng trnh: x2 2(m 2)x m2 5m 4 0 (*)
Ta c: ' = [-(m + 2)]2 (m2 + 5m + 4) = m2 + 4m + 4 m2 5m 4 = -m
Vi m < 0 ' = -m > 0 Phng trnh (*) lun lun c 2 nghi m phn bi t x1, x2
x1 x2 2(m 2)

2/ Theo nh l Vit ta c:

2
x1x2 m +5m +4

Theo ta c:

(I)

x x xx
1 1
1 1 2 1 2 0 (1)
x1 x2
x1x2

Thay (I) vo (1) ta c:

2(m +2) - (m2 +5m +4)


0
(k: m -1 v m -4)
m2 +5m +4
2(m + 2) (m2 + 5m + 4) = 0
2m + 4 m2 5m 4 = 0
m2 + 3m = 0
m(m + 3) = 0
k: m <0)
m =0 (loi vtri
m =-3 (tha iu kin: m <0; m 1 v m -4)

V y vi m = -3 th phng trnh (*) c 2 nghi m phn bi t x1, x2 tha h thc:


1 1
1
x1 x2

5.

a. Chng minh DE . DA = DC . DB

Ta c: ACB
900 (gc n i tip na ng trn (O))

ACD 900 (v k b vi ACB )

Ta li c:

AEB 900 (gc n i tip na ng trn (O))


0

DEB = 90 (v k b vi AEB )
Xt ADC v BDE c:

ACD DEB 900 (cm trn)

D : gc chung
ADC ~ BDE (g-g)
DA DC

DE . DA =DC . DB
DB DE

b. Chng minh MOCD l hinh binh hnh


Ta c: MC = MA (gt) OM AC (lin h gia k v dy cung)

CD AC (v ACD
900 )
OM // CD (cng vung gc vi AC) (1)
M t khc: DAB c: BE v AC l hai ng cao ct nhau ti M M l trc tm
DM l ng cao th ba DM AB
DM // CO (2)
CB
CO AB
M: CA = CB CA
T (1) v (2) suy ra: MOCD l hnh bnh hnh.
149

c. Ke EF AC. Tnh ti s
Xt MFE v MCB c:

MF
?
EF

MFE MCB 900

FME BMC (i nh)

MFE ~ MCB (g g)

MF MC

EF CB

Ta li c: AC = 2MC (gt). M: CB = CA CB = 2MC

MF MC MC 1

EF CB 2MC 2

d. Chng minh t gic BHIK ni tip c ng tron.


1
2

(gc n i tip ng trn tm (O)) (3)


Ta c: K sBE

1
2

sEA)

Ta li c: NHB (sBN
(gc c nh nm trong ng trn (O))

EN

M : EA = EN (bn knh ng trn (E)) EA

1
1 (sBN
sEN)
1 sBE
(4)
NHB (sBN
sEA)
2
2
2

T (3) v (4) suy ra: K NHB

M NHB
l gc ngoi ti H ca t gic BIHK

V y t gic BIHK n i tip c ng trn.

150

S GIO DC V O TO
QUNG BNH
CHNH THC

K THI TUYN SINH VO LP 10 THPT

Mn thi: TON

M : 201 (th sinh ghi m vo sau ch


bi lm)

Thi gian lm bi: 120 pht


1
1
m 1

vi m 0 , m 1
: 2
m m m 1 m 2m 1

Cu 1: (1.5 im): Cho biu thc:: P


a)Rt gn biu thc P

b) Tnh gi tr ca biu thc P khi x=

1
2

Cu 2:(1,5im) : Cho ba ng thng(d1): y= 2x+1; (d2): y=3; (d3): y=kx+5 .


a) Xc nh to giao im ca hai ng thng d1 v d2.
b) Tm k ba ng thng trn ng quy.
Cu 3:(2.5 im) Cho phng trnh bc hai n x: x2-2(m-1)x+2m-4=0 (m l tham s)
(1)
a) Gii phng trnh (1) khi m = 3
b)Chng minh rng phng trnh (1) lun c hai nghim phn bit vi mi m.
c) Gi x1,x2 l hai nghim ca phng trnh (1). Tm gi tr nh nht ca biu
thc
A = x12+x22
Cu 4: (3,5 im): Cho ng trn tm O, ng knh AB=2R. Gi M l mt im
bt k trn na ng trn( M khng trng vi A, B). V cc tip tuyn Ax, By, Mz ca
na ng trn. ng thng Mz ct Ax, By ln lt ti N v P. ng thng AM ct By ti
C v ng thng BM ct Ax ti D.
a) Chng minh t gic AOMN ni tip c trong mt ng trn.
b) Chng minh N l trung im ca AD, P l trung im ca BC
c) Chng minh AD.BC = 4R2
Cu 5: : (1,0im) Cho a, b, c l cc s dng . Chng minh rng :
25a
16b
c

8 .
bc ac ab

151

S GIO DC V O TO
TY NINH

K THI TUYN SINH LP 10 THPT


NM HC 2012 2013
Mn thi: TON(Khng chuyn)
Ngy thi : 02 thng 7 nm 2012
Thi gian lm bi: 120 pht (khng k thi gian giao )

chnh thc

Cu 1 : (1im) Thc hi n cc php tnh


a) A 2. 8
b) B 3 5 20
2
Cu 2 : (1 im) Gii phng trnh: x 2 x 8 0 .

2x y 5
.
3 x y 10

Cu 3 : (1 im) Gii h phng trnh:

Cu 4 : (1 im) Tm x mi biu thc sau c ngha:


a)

1
x 9

b)

4 x2

Cu 5 : (1 im) V th ca hm s y x 2

Cu 6 : (1 im) Cho phng trnh x 2 m 1 x m 3 0 .


a) Tm m phng trnh c nghi m.
b) Gi x1 , x2 l hai nghi m ca phng trnh cho, tm gi tr nh nht ca biu thc
A x1 x2 x1 x2 .
Cu 7 : (1 im) Tm m th hm s y 3 x m 1 ct trc tung ti im c tung bng 4.
Cu 8 : (1 im) Cho tam gic ABC vung ti A c ng cao l AH. Cho bit AB 3cm , AC 4cm . Hy
tm di ng cao AH.
Cu 9 : (1 im) Cho tam gic ABC vung ti A. Na ng trn ng knh AB ct BC ti D. Trn cung
AD ly m t im E. Ni BE v ko di ct AC ti F. Chng minh t gic CDEF l m t t gic n i tip.
Cu 10: (1 im) Trn ng trn (O) dng m t dy cung AB c chiu di khng i b hn ng knh.
sao cho chu vi tam gic AMB c gi tr ln nht.
Xc nh v tr ca im M trn cung ln AB
2

BI GII
Cu 1 : (1im) Thc hi n cc php tnh.
a) A 2. 8 16 4
b) B 3 5 20 3 5 2 5 5 5 .
Cu 2 : (1 im) Gii phng trnh.
x2 2x 8 0 .

' 1 1. 8 9 0 , ' 9 3 .
x1 1 3 4 , x2 1 3 2 .
V y S = 4; 2 .
2

Cu 3 : (1 im) Gii h phng trnh.


152

5 x 15
x3
x3
2x y 5

y 1
3 x y 10 3 x y 10 9 y 10

.
V y h phng trnh cho c nghi m duy nht 3;1
Cu 4 : (1 im) Tm x mi biu thc sau c ngha:
a)

1
c ngha
x 9
2

x2 9 0 x2 9

x 3 .

2
2
4 x 2 c ngha 4 x 0 x 4 2 x 2 .
Cu 5 : (1 im) V th ca hm s y x 2 .

b)

BGT

x
y x2

2
4

1 0 1 2
1

0 1 4

Cu 6 : (1 im)

x 2 2 m 1 x m 2 3 0 .

a) Tm m phng trnh c nghi m.

' m 1 1. m 2 3 m 2 2m 1 m 2 3 2m 2 .
2

Phng trnh c nghi m ' 0 2m 2 0 m 1 .


b) Tm gi tr nh nht ca biu thc A x1 x2 x1 x2 .
iu ki n m 1 .
2
Theo Vi-t ta c : x1 x2 2m 2 ; x1 x2 m 3 .

A x1 x2 x1 x2 2m 2 m 2 3 m 2 2m 5 m 1 4 4 .
2

A min 4 khi m 1 0 m 1 (loai v khng tha iu ki n m 1 ).


M t khc : A m 1 4 1 1 4 (v m 1 )
2

A 8.

A min 8 khi m 1 .
Kt lun : Khi m 1 th A t gi tr nh nht v A min 8 .
Cch 2: iu ki n m 1 .
2
Theo Vi-t ta c : x1 x2 2m 2 ; x1 x2 m 3 .
A x1 x2 x1 x2 2m 2 m 2 3 m 2 2m 5 .
V m 1 nn A m 2 2m 5 12 2.1 5 hay A 8
V y A min 8 khi m 1 .
Cu 7 : (1 im)
153

th hm s y 3 x m 1 ct trc tung ti im c tung bng 4.


m 1 4 m 5 .
V y m 5 l gi tr cn tm.
Cu 8 : (1 im)

Ta c:

Cch 2:

BC AB2 AC 2 32 4 2 5 cm .
AH.BC AB.AC
AB.AC 3.4
AH

2, 4 cm .
BC
5

1
1
1

2
2
AH
AB AC 2
AB2 .AC 2
32.4 2
32.4 2
2
AH

2 .
AB2 AC 2 32 42
5
3.4
AH
2, 4 cm .
5

Cu 9 : (1 im)

AB
900 , na O; ct BC ti D,
ABC , A
GT
2

, BE ct AC ti F.
E AD
KL CDEF l m t t gic n i tip

1
1
1 sAmB
sAED

sAED

sADB
sBD
Ta c : C
2

( C l gc c nh ngoi ng trn).

sBD ( BED
gc n i tip).
2

1 sBD

BED
C
2
T gic CDEF n i tip c (gc ngoi bng gc i trong).

M t khc BED

Cu 10: (1 im)

O ,

dy AB khng i, AB 2R , M AB
(cung ln).
Tm v tr M trn cung ln AB chu vi tam gic
KL
AMB c gi tr ln nht.
GT

154

Gi P l chu vi MAB . Ta c P = MA + MB + AB .
Do AB khng i nn Pmax MA + MB max .

(khng i).

Do dy AB khng i nn AmB
khng i. t sAmB
Trn tia i ca tia MA ly im C sao cho MB = MC .
1 2C
1 (gc ngoi ti nh MBC cn)
MBC cn ti M M
1
1 1
1
1
M1 sAmB
sAmB
(khng i).
2
2 2
4
4
1
im C nhn on AB c nh di m t gc khng i bng .
4
1
C thu c cung cha gc dng trn on AB c nh.
4
MA + MB = MA + MC = AC (v MB = MC ).
MA + MB max AC max AC l ng knh ca cung cha gc ni trn.
1B
2 900
B
0

1B
2 (do B
1 C
1 ) AMB cn M.

ABC 90
A
0

C1 A1 90
MB
M l im chnh gia ca AB
(cung ln).
MA = MB MA
th chu vi MAB c gi tr ln nht.
V y khi M l im chnh gia ca cung ln AB
1
C

155

S GIO DC V O TO
CAO BNG
CHNH THC

K THI TUYN SINH VO LP 10 THPT


NM HC 2012 - 2013
Mn thi: TON
Ngy thi : 22/06/2011
Thi gian lm bi: 120 pht

Cu 1: (4,0 im)
a) Tnh: 36 ; 81 .
b) Gii phng trnh: x 2 = 0.
c) Gii phng trnh: x2 4x + 4 = 0.
Cu 2: (2,0 im)
Mt mnh vn hnh ch nht c chu vi 400m. Bit chiu di hn chiu rng 60m. Tnh chiu di v
chiu rng mnh vn .
Cu 3: (1,0 im)
Cho tam gic ABC vung ti A, bit AB = 3cm, AC = 4cm.
a) Tnh cnh BC.
b) K ng cao AH, tnh BH.
Cu 4: (2,0 im)
Cho ng trn tm O, bn knh R; P l mt im ngoi ng trn sao cho OP = 2R. Tia PO ct
ng trn (O; R) A (A nm gia P v O), t P k hai tip tuyn PC v PD vi (O; R) vi C, D l hai tip
im.
a) Chng minh t gic PCOD ni tip.
b) Chng minh tam gic PCD u v tnh di cc cnh tam gic PCD.
Cu 5: (1,0 im)
Tm gi tr nh nht ca biu thc: A =

x2 4x 1
x2

156

S GIO DC V O TO
LNG SN

CHNH THC

K THI TUYN SINH LP 10 THPT


NM HC 2012 2013
Mn thi: TON
Thi gian lm bi: 120 pht (khng k thi gian giao )
Ngy thi: 27 thng 06 nm 2012
thi gm: 01 trang

Cu I (2 im).
1.tnh gi tr biu thc:
A=

3 1 1

B=

12 27
3

1
1
x 1

:
x 1 1 x x 1 1
x 1
Tm x biu thc P c ngha; Rt gn P . Tm x P l mt s nguyn

2. Cho biu thc P = 2

Cu II (2 im).
1. V th hm s : y = 2x2
2. Cho phng trnh bc hai tham s m : x2 -2 (m-1) x - 3 = 0
a. Gii phng trnh khi m= 2
b. Chng minh rng phng trnh lun c hai nghim phn bit x 1; x2 vi mi gi tr ca m.
x1
x2
Tm m tha mn 2 2 m 1
x2
x1
Cu III (1,5 im).
Trong thng thanh nin on trng pht ng v giao ch tiu mi chi on thu gom 10kg giy vn
lm k hoch nh. nng cao tinh thn thi ua b th chi on 10A chia cc on vin trong lp thnh hai t
thi ua thu gom giy vn. C hai t u rt tch cc. T 1 thu gom vt ch tiu 30%, t hai gom vt ch tiu
20% nn tng s giy chi on 10A thu c l 12,5 kg. Hi mi t c b th chi on giao ch tiu thu
gom bao nhiu kg giy vn?
Cu IV (3,5 im).
Cho ng trn tm O,ng knh AB, C l mt im c nh trn ng trn khc A v B. Ly D l
im nm gia cung nh BC. Cc tia AC v AD ln lt ct tip tuyn Bt ca ng trn E v F
a, Chng minh rng hai tam gic ABD v BFD ng dng
b, Chng minh t gic CDFE ni tip
c, Gi D1 i xng vi D qua O v M l giao im ca AD v CD 1 chng minh rng sooe o gc
AMC khng i khi D chy trn cung nh BC
Cu V (1 im).
Chng minh rng Q = x 4 3 x 3 4 x 2 3x 1 0 vi mi gi tr ca x
p n :
Cu I (2 im).
1. A.

3 1 1 =

12 27
=5
3

2. K : x >1
157

P=

2
x 1

P l mt s nguyn

x 1 U (2) 1; 2

=> x 2;5
Cu II (2 im).
1. HS t v
2. a) x = -1 hoc x = 3
b ) C ' (m 1) 2 3 0m => Pt lun c 2 nghim phn bit
Theo Vi t c : x1 x2 2m 2
x1.x2 3
x1
x2
Theo bi : 2 2 m 1
x2
x1
2
2
3
3
2
=> x1 x2 (m 1)( x1 x2 ) => ( x1 x2 ) ( x1 x2 ) 3 x1 x2 (m 1)( x1 x2 )
2
2
2
=> (2m 2) (2m 2) 3.( 3) ( m 1)(3) => (2m 2) 4m 8m 13 9( m 1)

=> 8m3 16m2 26m 8m 2 16m 26 9m 9 0 => 8m3 24m 2 33m 17 0


m 1
=> (m 1)(8m 2 16m 17) 0 =>
2
8m 16m 17 0(Vn)
Vy m = 1 l gi tr cn tm
Cu III (1,5 im).
Gi s kg giy vn t 1 c b th chi on giao l x (kg) ( k : 0 < x <10)
S kg giy vn t 2 c b th chi on giao l y (kg) ( k : 0 < x <10 )
x y 10
Theo u bi ta c hpt:
1,3 x 1, 2 y 12,5
Gii h trn ta c : (x; y ) = (5;5)
Tr li : s giy vn t 1 c b th chi on giao l 5 kg
S giy vn t 2 c b th chi on giao l 5 kg
Cu IV (3,5 im).
1. ABD v BFD
c : ADB= BDF = 900
BAD = DBF ( Cng chn cung BD)
=>
ABD
BFD
2. C : E = (SdAB- SdBC): 2 ( Gc ngoi ng trn)
= SdAC: 2
= CDA
=> T gic CDFE ni tip
A

3. D dng chng minh c t gic ADBD1 l hnh ch nht


D1
C : AMC = AD1M + MAD1 ( Gc ngoi tam gic AD1M)
= (SdAC: 2) + 900
M AC c nh nn cung AC c nh=> AMC lun khng i khi D chy trn cung nh BC
Cu V (1 im).
Q = x 4 3 x 3 4 x 2 3x 1
158

= ( x 4 2 x3 x 2 ) (1 3 x 3 x 2 x 3 )
= x 2 ( x 1) 2 (1 x)3
1 2 3
1 3
2
2
2
= (1 x ) 2 ( x 2 x 1) = (1 x) ( x x ) = (1 x ) ( x ) 0x
2
4
4 4

S GD & T HA BNH
K THI TUYN SINH VO LP 10 NM HC 2012- 2013
TRNG THPT CHUYN HONG VN TH
THI MN TON (CHUNG)
CHNH THC
Ngy thi: 29 thng 6 nm 2012
Thi gian lm bi: 120 pht (khng k thi gian giao )
thi gm c 01 trang
--------------------------------------------------------------------------------------------------------PHN I. TRC NGHIM(2 im)
(Th sinh khng cn gii thch v khng phi chp li bi, hy vit kt qu cc bi ton sau vo t giy
thi)
1. Biu thc A = 2 x 1 c ngha vi cc gi tr ca x l
2. Gi tr m 2 ng thng (d1): y = 3x 2 v (d2): y = mx + 3m 1 ct nhau ti 1 im trn trc tung
l....
3. Cc nghim ca phng trnh 3 x 5 1 l...
4. Gi tr ca m phng trnh
x2 (m+1)x - 2 = 0
c 2 nghim x1, x2 tha mn
2
2
x1 x2 + x1x2 = 4 l...
PHN II. T LUN (8 im)
Bi 1. (2 im)
1 1
x y 5

a) Gii h phng trnh


2 3 5
x y
b) Cho tam gic ABC vung ti A (AB > AC). ng phn gic AD chia cnh huyn BC thnh 2 on
3
theo t l
v BC = 20cm. Tnh di hai cnh gc vung.
4

Bi 2. (2 im) Tm mt s c hai ch s, bit rng ch s hng chc ln hn ch s hng n v l 5 v nu


em s chia cho tng cc ch s ca n th c thng l 7 v d l 6.
Bi 3.(3 im) Cho tam gic ABC c ba gc nhn ni tip trong ng trn tm O, bn knh R. Cc ng
cao AD, BE, CF ca tm gic ct nhau ti H. Chng minh rng:
a) T gic BCEF ni tip c.
b) EF vung gc vi AO.
c) Bn knh ng trn ngoi tip tam gic BHC bng R.
Bi 4. (1 im) Trn cc cnh ca mt hnh ch nht t ln lt 4 im ty . Bn im ny to thnh mt
t gic c di cc cnh ln lt l x, y, z , t. Chng minh rng
25 x2 + y2 + z2 + t2 50. Bit rng hnh ch nht c chiu di v chiu rng l 4 v 3.
P N
159

PHN I. TRC NGHIM(2 im)


1
2
2. Gi tr m 2 ng thng (d1): y = 3x 2 v (d2): y = mx + 3m 1 ct nhau ti 1 im trn trc tung
1
l m .
3
4
3. Cc nghim ca phng trnh 3 x 5 1 l: x = 2; x = .
3
4. Gi tr ca m phng trnh
x2 (m+1)x - 2 = 0
c 2 nghim x1, x2 tha mn
2
2
x1 x2 + x1x2 = 4 l m = -3.

1. Biu thc A =

2 x 1 c ngha vi cc gi tr ca x l: x

PHN II. T LUN(8 im)


Bi 1. (2 im)

a) Gii h phng trnh:

iu kin: x, y 0.

1 1
5 (1)
x y
2 3
5 (2)
x y

3 2
2x
0 3y 2x y
, th vo (1) ta c pt:
x y
3
1 3
5
1

5
5 2 x 1 x (tha mn k x 0 )
x 2x
2x
2
1
1
Vi x y (tha mn k y 0 )
2
3
Ly (1) cng (2) theo v, ta c:

1 1
2 3

Vy h phng trnh cho c 1 nghim ( x; y ) ( ; )


b) t di cnh AB = x (cm) v AC = y (cm); k: x > y > 0
Theo tnh cht ng phn gic v nh l pitago ta c:

y 3
y x

x 4
x 2 y 2 202 x 2 9 x 2 20 2

16
3

y 12
y x

4
x 16 x 16

y x

2
2

x 16

Vy di cnh AB = 16 (cm) ; AC = 14 (cm)


Bi 2. (2 im) Gi s cn tm c 2 ch s l ab , vi a, b {0,1,2,3,4,5,6,7,8,9}, a 0 .
Theo gi thit ta c h phng trnh:

a b5
a b 5
a8
ab5
a b5

(t/m k)

a 2b 2
a 2b 2
b3
10a b 7(a b) 6 3a 6b 6

160

Vy s cn tm l: 83
Bi 3.(3 im)
a) V BE, CF l ng cao ca tam gic ABC

BE AC ; CF AB BEC
CFB
900
E, F thuc ng trn ng knh BC
T gic BCEF ni tip.

b) EF vung gc vi AO.
Xt AOB ta c:
1
1

OAB
900 AOB 900 s AB 900 ACB (1)
2
2

Do BCEF ni tip nn AFE ACB


(2)
T (1) v (2) suy ra:

OAB
900 AFE OAB
AFE 900 OA EF (pcm)
c) Bn knh ng trn ngoi tip BHC bng R.
Gi H ' AH (O ) . Ta c:

' AC H
' BC (3)
HBC
900 ACB HAC
H

' AB H
' CB (4)
HCB
900 ABC HAB
H
T (3) v (4) BHC BH ' C ( g .c.g )
M BH'C ni tip ng trn tm O, bn knh R BHC cng ni tip ng trn c bn knh R,
tc l bn knh ng trn ngoi tip BHC bng R.
Bi 4. (1 im) Gi s hnh ch nht c di cc cnh c t nh
hnh v.
Vi: 0 a, b, e, f 4 v a+b = e+f = 4;
0 c, d, g, h 3 v c+d = g+h = 3.
Ta c:

x 2 h2 a 2 ; y 2 b2 c 2 ; z 2 d 2 e2 ; t 2 f 2 g 2
x 2 y 2 z 2 t 2 (a 2 b 2 ) (c 2 d 2 ) (e 2 f 2 ) ( g 2 h 2 ) (*)
2
2
2
2
Chng minh: x y z t 50 .
2
2
2
2
2
2
2
2
2
V a, b 0 nn a b ( a b) 16 . Tng t: c d 9; e f 16; g h 9 .
2
2
2
2
T (*) x y z t 16 9 16 9 50 (1)
2
2
2
2
Chng minh: x y z t 25 .
p dng bt ng thc Bu - nhi - a- cp xki , ta c:

(a b) 2 16

2
2
9 2
16 2
9
2
2
2
Tng t: c d ; e f
; g h2 .
2
2
2
16 9 16 9
2
2
2
2
T (*) x y z t
25 (2)
2 2 2 2
2
2
2
2
T (1) v (2) 25 x y z t 50 (pcm)
(12 12 )(a 2 b 2 ) (1.a 1.b) 2 a 2 b 2

161

S GD & T HA BNH
CHNH THC

THI TUYN SINH VO 10 NM HC 2012-2013


THI MN: TON
Ngy thi: 19/ 07/ 2012
Thi gian lm bi: 120 pht (khng k thi gian giao )

Cu 1. (3,0 im)
1. Tm iu kin c ngha ca biu thc:
1
a)
;
b) x 2 .
x 1
2. Phn tch a thc thnh nhn t :
a) x 2 5 x ;
b) x 2 7 xy 10 y 2
3. Cho tam gic ABC vung ti A; AB = 2 cm, AC = 4 cm. Tnh di cnh BC.
Cu 2. (3,0 im)
1. Gii phng trnh: 2(x + 5) + (x 3)(x + 3) = 0.
2. a) V th hm s y = 3x + 2 (1).
b) Gi A, B l giao im ca th hm s (1) vi trc tung v trc honh.
Tnh din tch tam gic OAB.
Cu 3. (1,0 im) Mt phng hp c 320 gh ngi c xp thnh tng dy v s gh mi dy u bng
nhau. Nu s dy gh tng tng thm 1 v s gh mi dy tng thm 2 th trong phng c 374 gh. Hi trong
phng c bao nhiu dy gh v mi dy c bao nhiu gh?
Cu 4. (2,0 im)
Cho ng trn tm O, bn knh R v im M sao cho MO = 2R. Qua im M k cc tip tuyn MA,
MB vi ng trn (O). Hai ng cao BD v AC ca tam gic MAB ct nhau ti H
1) Chng minh t gic AHBO l hnh thoi.

2) Tnh gc AMB
.
162

Cu 5. (1,0 im) Cho hai s thc x, y tha mn: x 2 y 2 x y . Chng minh rng: x y 2
Ht

P N THI TUYN SINH MN TON VO 10 HA BNH NM HC 2012-2013


Cu 1. (3,0 im)
1. Tm iu kin c ngha ca biu thc:
a) iu kin: x 1 0 x 1 ;

b) iu kin: x 2 0 x 2

2. Phn tch a thc thnh nhn t :


a) x 2 5 x x( x 5) ;
b) Cch 1: Phng php tch, thm bt s hng:
x 2 7 xy 10 y 2 ( x 2 2 xy ) (5 xy 10 y 2 ) x( x 2 y ) 5 y ( x 2 y ) ( x 2 y )( x 5 y )
2
Cch 2: S dng nh l: Nu pt bc hai ax bx c 0(a 0) c 2 nghim phn bit x1, x2 th:

ax 2 bx c a(x x1 )(x x 2 ) .
p dng vo bi ton trn ta xem pt: x 2 7 xy 10 y 2 0 nh l 1 pt bc hai n x, tham s y.
Ta c (7y) 2 4.10y 2 9y 2 3y ; x1

7y 3y
7y 3y
2y; x 2
5y
2
2

Suy ra: x 2 7 xy 10 y 2 ( x 2 y )( x 5 y )
3. Cho tam gic ABC vung ti A; AB = 2 cm, AC = 4 cm. Tnh di cnh BC.

C
4 cm

V tam gic ABC vung ti A, nn theo nh l Pitago ta c:

BC 2 AB2 AC 2 22 42 20 BC 20 2 5 (cm)
Cu 2. (3,0 im)
1. Gii phng trnh: 2 x+5 x 3 x 3 0

2 cm

2x 10 x 2 9 0
x 2 2x 1 0

(x 1) 2 0
x 1 0
x 1

2. a) V th hm s y = 3x + 2 (1).

2 O
3

x
163

+ Cho x 0 y 2
+ Cho y 0 x

2
3
2
3

+ th hm s y = 3x + 2 l mt ng thng i qua 2 im (0;2) v ( ;0)


b) T cch v th hm s y = 3x + 2 ta c:
+ Giao ca th hm s (1) vi trc Oy l A(0;2)

2
3

+ Giao ca th hm s (1) vi trc Ox l B ( ;0)


Suy ra din tch OAB l : SOAB

1
1
2 2
OA.OB . | 2 | . | | (vdt)
2
2
3 3

Cu 3. (1,0 im) Mt phng hp c 320 gh ngi c xp thnh tng dy v s gh mi dy u bng


nhau. Nu s dy gh tng tng thm 1 v s gh mi dy tng thm 2 th trong phng c 374 gh. Hi trong
phng c bao nhiu dy gh v mi dy c bao nhiu gh?
Gii: Gi s dy gh trong phng hp l x (dy) ( x * )
Gi s gh trong mi dy l y (gh) ( y * )
V phng hp c 320 gh ngi c xp thnh tng dy v s gh mi dy u bng nhau nn ta c
phng trnh: xy 320 (1)
V s dy gh tng tng thm 1 v s gh mi dy tng thm 2 th trong phng c 374 gh nn ta c
phng trnh: (x 1)(y 2) 374 (2)
T (1) v (2) ta c h phng trnh:

xy 320

(x 1)(y 2) 374

320
320

xy 320
xy 320

y
x

xy 2x y 2 374 2x y 52
320
2
2x
52 x 26x 160 0

x
320
320

x=10
x=16
y
y

x
x
hoc

y 32
y 20
x 2 26x 160 0 x 2 26x 160 0

Vy trong phng hp c 10 dy gh v mi dy c 32 gh
Hoc l trong phng hp c 16 dy gh v mi dy c 20 gh
Cu 4. (2,0 im)
Cho ng trn tm O, bn knh R v im M sao cho MO = 2R. Qua im M k cc tip tuyn MA,
MB vi ng trn (O). Hai ng cao BD v AC ca MAB ct nhau ti H.
1) Chng minh t gic AHBO l hnh thoi.
Ta c: OA MA (V MA l tip tuyn vi ng trn (O))
164

BH MA ( V BH l ng cao trong MAB)

OA // BH (1)
OB MB
OB / /AH (2)
AH MB

Tng t ta c:

T (1) & (2) suy ra t gic AHBO l hnh bnh hnh,


mt khc li c OA = OB nn t gic AHBO l hnh thoi.

2) Tnh gc AMB
.

D thy MO l ng phn gic trong ca gc AMB


.
AMB
2AMO
V tam gic OAM vung ti A nn ta c: sin AMO

OA 1

AMO 300 AMB


600 .
MO 2

Cu 5. (1,0 im) Cho hai s thc x, y tha mn: x 2 y 2 x y . Chng minh rng: x y 2
Cch 1:

(x y) 2
Nhn xt: xy
; x, y .
4
Tht vy: xy

(x y) 2
(x y) 2 4xy (x y) 2 0; x, y (ng)
4

Do t gi thit: x 2 y 2 x y
( x y ) 2 x y 2 xy
( x y)2 x y

( x y)2
2

( x y ) 2 2( x y )
( x y )( x y 2) 0 (*)
V x y x 2 y 2 0; x, y , nn ta xt cc trng hp sau:

Nu x 2 y 2 0 x y 0 x y 0 2

Nu x 2 y 2 0 x y 0 , t (*) suy ra: x y 2 0 x y 2

T suy ra: x y 2 . Du bng xy ra khi x = y = 1.


Cch 2: p dng BT Bu nhi a cp xki: x, y , ta c:

(1.x 1.y) 2 (12 12 )(x 2 y 2 )


(x y) 2 2(x 2 y 2 )
(x y) 2 2(x y)
(x y)(x y 2) 0 (*)
165

V x y x 2 y 2 0; x, y , nn ta xt cc trng hp sau:

Nu x 2 y 2 0 x y 0 x y 0 2

Nu x 2 y 2 0 x y 0 , t (*) suy ra: x y 2 0 x y 2

T suy ra: x y 2 . Du bng xy ra khi x = y = 1.

S GIO DC V O TO
QUNG NAM

K THI TUYN SINH LP 10 THPT CHUYN


Nm hoc: 2012 2013
Kha thi: Ngy 4 thng 7 nm 2012
Mn: TON (Ton chung)
Thi gian lm bi: 120 pht ( khng k thi gian giao )

CHNH THC

Cu 1: (2,0 im)

x 2 3x 3

x 3

Cho biu thc: A

4x 12 .

a) Tm iu kin ca x biu thc A c ngha.


b) Rt gn biu thc A.
c) Tnh gi tr ca A khi x 4 2 3 .
Cu 2: (2,0 im)
a) Xc nh cc h s a, b ca hm s y = ax + b, bit th ca n l ng thng song song vi ng
thng y = 2x + 1 v i qua im M(1 ; 3).
b) Gii h phng trnh (khng s dng my tnh cm tay):

2x y 3

2x y 1

Cu 3: (2,0 im)

Cho parabol (P): y

1 2
x v ng thng (d): y = (m 1)x 2 (vi m l tham s).
2

a) V (P).
b) Tm m (d) tip xc vi (P) ti im c honh dng.
c) Vi m tm c cu b), hy xc nh ta tip im ca (P) v (d).
Cu 4: (4,0 im)
Cho tam gic ABC vung ti A. Qua C k ng thng d vung gc vi AC. T trung im M ca cnh
AC k ME vung gc vi BC (E thuc BC), ng thng ME ct ng thng d ti H v ct ng thng AB
ti K.
a) Chng minh: AMK = CMH, t suy ra t gic AKCH l hnh bnh hnh.
166

b) Gi D l giao im ca AH v BM. Chng minh t gic DMCH ni tip v xc nh tm O ca ng


trn ngoi tip t gic .
c) Chng minh: AD.AH = 2ME.MK.

d) Cho AB = a v ACB
300 . Tnh di ng trn ngoi tip t gic DMCH theo a.

S GIO DC V O TO
QUNG NAM

K THI TUYN SINH LP 10 THPT CHUYN


Nm hoc: 2012 2013
Kha thi: Ngy 4 thng 7 nm 2012
CHNH THC
Mn: TON (Ton chung)
Thi gian lm bi: 120 pht ( khng k thi gian giao )
HNG DN CHM THI
(Bn hng dn ny gm 02 trang)
Cu
Ni dung
im
Cu 1
a)
iu kin: x 0
0,25
(2,0) (0,5)
v x 3
0,25
2
b)
0,25
Bin i c: x 2 3 x 3 x 3
(1,0)
x 3 x 3
x 3
0,25

4 x 12 2
A=
c)
(0,5)
Cu 2
(2,0)

Cu 3
(2,0)

x 3

x 3

x 3

0,25

x 3

.2

Bin i c: x 4 2 3

x 3 2

3 1

x 3

Tnh c: A = 2
+ V ng thng y = ax + b song song vi ng thng y = 2x + 1 nn a =
2 (khng yu cu nu b 1)
+ Thay ta im M (1 ; 3) v a = 2 vo y = ax + b
+ Tm c: b = 1

b)
(1,0)

a)
(0,5)
b)
(1,0)

0,25
0,25

a)
(1,0)

2x y 3

0,25

2y 2

2 x y 3
2 x y 1
Tnh c: y = 1
x= 2
Vy nghim ca h phng trnh cho l: (x ; y) = ( 2 ; 1)
+ Lp bng gi tr ng (chn ti thiu 3 gi tr ca x trong phi c gi tr x =
0).
+ V ng dng ca (P).
+ Phng trnh honh giao im ca (P) v (d):

0,5
0,25
0,25
0,25
0,25
0,25
0,25
0,25
0,25

167

1 2
x (m 1)x 2
2

c)
(0,5)

Cu
Cu 4 Hnh
(4,0)
v
(0,25)

x2 2(m 1)x +4 = 0
' 0
2

m 1 4 0

+ Lp lun c: b '

m 1 0

a 0
cm 3
m 1 ho

m 1
+ Kt lun c: m = 3
b ' m 1 3 1

2
+ Tm c honh tip im: x
a
1
1
+Tnh c tung tip im: y = 2 v kt lun ng ta tip im l (2; 2).
Ni dung

0,25
0,25
0,25
0,25

0,25
0,25

im

0,25

a)
(1,0)

b)
(1,0)

c)
(1,0)

+ AM = MC (gt) , KAM
()
HCM
900 , AMK
CMH

+ AMK CMH g.c.g


+ suy ra: MK = MH
+ V MK = MH v MA = MC nn t gic AKCH l hnh bnh hnh.
+ Nu c: CA BK v KE BC , suy ra M l trc tm tam gic KBC.
+ Nu c: KC // AH v BM KC, suy ra BM AH.

+ HDM
HCM
900 900 1800 => T gic DMCH ni tip.

+ MCH
900 => Tm O ca ng trn ngoi tip t gic DMCH l trung
im MH.
+ Chng minh c hai tam gic ADM v ACH ng dng (g.g)
AM AD

AM . AC AH . AD 2 AM 2 AH . AD vAC=2AM
+
AH AC
AH . AD
AM 2
(1)
2
+ Ta li c: MC2 = ME.MH v MH=MK nn MC2 = ME.MK (2)
+ Mt khc: MC = MA (gt) (3)

0,25
0,25
0,25
0,25
0,25
0,25
0,25
0,25
0,25

0,25
0,25
168

AH . AD
ME.MK => AH.AD = 2ME.MK
2

T (1), (2), (3) =>

d)
+ ABC vung ti A, gc C = 30 nn AC = a 3 .
(0,75)

+ ACB
MHC
300 (cng ph gc CMH) => MH = 2MC
M AC = 2MC nn: MH = AC = a 3 .
+ di ng trn ngoi tip t gic DMCH l:
0

a 3
MH
a 3
2
2
2

C 2

d
+ Tam gic ABC vung ti A nn: AC = AB.cotC = a 3 .
(0,75)

+ CMH
900 ACB
600
MC
AC

AC a 3
=> MH

cosCMH
2cos600
Din tch hnh trn (O):
2

0,25
0,25
0,25
0,25

0,25
0,25
0,25

a 3
3 2
MH
a

4
2
2

+ S(O)

169

S GIO DC V O TO
QUNG NAM

K THI TUYN SINH LP 10 THPT CHUYN


Nm hoc: 2012 2013
Kha thi: Ngy 4 thng 7 nm 2012
Mn: TON (Chuyn Ton)
Thi gian lm bi: 150 pht (khng k thi gian giao )

CHNH THC

Cu 1: (1,5 im)
a) Rt gn biu thc: A =

b) Cho x

a a 6
1

4a
a 2

(vi a 0 v a 4).

28 16 3
. Tnh gi tr ca biu thc: P (x 2 2x 1) 2012 .
3 1

Cu 2: (2,0 im)
a) Gii phng trnh:

3(1 x) 3 x 2 .
x 2 xy 4x 6

b) Gii h phng trnh:

2
y xy 1

Cu 3: (1,5 im)
Cho parabol (P): y = x2 v ng thng (d): y = (3 m)x + 2 2m (m l tham s).
a) Chng minh rng vi m 1 th (d) lun ct (P) ti 2 im phn bit A, B.
b) Gi yA, yB ln lt l tung cc im A, B. Tm m |yA yB| = 2.
Cu 4: (4,0 im)
Cho hnh ch nht ABCD c AB = 4 cm, AD = 2 cm. ng thng vung gc vi AC ti C ct cc
ng thng AB v AD ln lt ti E v F.
a) Chng minh t gic EBDF ni tip trong ng trn.
b) Gi I l giao im ca cc ng thng BD v EF. Tnh di on thng ID.

170

c) M l im thay i trn cnh AB (M khc A, M khc B), ng thng CM ct ng thng AD ti N.


Gi S1 l din tch tam gic CME, S2 l din tch tam gic AMN. Xc nh v tr im M S1

3
S2 .
2

Cu 5: (1,0 im)
Cho a, b l hai s thc khng m tha: a + b 2.
Chng minh:

2 a 1 2b 8

.
1 a 1 2b 7
--------------- Ht ---------------

S GIO DC V O TO
QUNG NAM

K THI TUYN SINH LP 10 THPT CHUYN


Nm hoc: 2012 2013
Kha thi: Ngy 4 thng 7 nm 2012
Mn: TON (Chuyn Ton)
Thi gian lm bi: 150 pht ( khng k thi gian giao )

CHNH THC

Cu
Cu 1
(1,5 im)

HNG DN CHM THI


(Bn hng dn ny gm 03 trang)
Ni dung

a a 6
1

4a
a 2
( a 2)( a 3)
1

A=
(2 a )(2 a )
a 2
a) (0,75) A =

(a 0 v a 4)

a 3
1

2 a 2 a

= 1
b) (0,75) Cho x

(4 2 3) 2

3 1

0,25
0,25
0,25

28 16 3
. Tnh: P (x 2 2x 1) 2012
3 1

4 2 3 ( 3 1) 2 =
3 1

3 1
3 1

x 2x 1 1
P (x 2 2x 1) 2012 1
Cu 2
(2,0 im)

im

0,25
0,25
0,25

a) (1,0) Gii phng trnh: 3(1 x) 3 x 2 (1)


Bnh phng 2 v ca (1) ta c:

3(1 x) 3 x 2 3(1 x)(3 x) 4

3(1 x)(3 x) 1 x

3(1 x)(3 x) 1 2x x 2

x 2 x 2 0 x = 1 hoc x =2
Th li, x = 2 l nghim .

0,25

0,25
0,25
0,25
171

x 2 xy 4x 6 (1)

b) (1,0) Gii h phng trnh:

2
y xy 1

(2)

(I)

Nu (x;y) l nghim ca (2) th y 0.


Do : (2) x

y2 1
(3)
y

Thay (3) vo (1) v bin i, ta c:


4y3 + 7y2 + 4y + 1 = 0
(y + 1)(4y2 + 3y + 1) = 0 (th sinh c th b qua bc ny)
y=1
y=1 x=2
Vy h c mt nghim: (x ; y) = (2 ; 1).

Cu
Cu 3
(1,5 im)

Ni dung
a) (0,75) (P): y = x , (d): y = (3 m)x + 2 2m.
Chng minh rng vi m 1 th (d) lun ct (P) ti 2 im phn bit A, B
Phng trnh honh giao im ca (P) v (d):
x2 = (3 m)x + 2 2m.
x2 + (3 m)x + 2 2m = 0 (1)
= (3m)2 4(2 2m) = m2 + 2m + 1
Vit c: = (m + 1)2 > 0, vi m 1 v kt lun ng.
b) (0,75) Tm m |yA yB| = 2 .
Gii PT (1) c hai nghim: x1 = 2 v x2 = m 1
Tnh c: y1 = 4, y2 = (m 1)2
|yA yB| = |y1 y2| = |m22m3|
|yA yB| = 2 m2 2m 3 = 2 hoc m2 2m 3 = 2
m = 1 6 hoc m = 1 2

0,25
0,25
0,25

0,25

im

0,25
0,25
0,25
0,25
0,25
0,25

Cu 4
(4,0 im)

a) (1,0) Chng minh t gic EBDF ni tip trong ng trn.


Ta c:

ADB
ACB

( cng ph vi BAC
)
AEC
ACB

ADB
AEC
t gic EBDF ni tip

0,25
0,25
0,25
0,25

b) (1,5) Tnh ID
172

Tam gic AEC vung ti C v BC AE nn: BE.BA = BC2

BC2
1
BA
IB BE 1

BE//CD
ID CD 4
BD 3

ID 4
4
ID BD v tnh c: BD = 2 5
3
8 5
ID
(cm)
3
BE

Cu
Cu 4
(tt)

0,25
0,25
0,25
0,25
0,25

Ni dung
c) (1,5 im) Xc nh v tr im M S1 =

im

3
S2
2

t AM = x, 0 < x < 4
MB = 4 x , ME = 5 x
AN AM
BC.AM
2.x

AN

Ta c:
BC MB
MB
4 x

0,25
0,25
0,25

1
1
x2
BC.ME 5 x , S2 AM.AN
2
2
4x
2
3
3 x
S1 = S2 5 x = .
x2 + 18x 40 = 0
2
2 4x
S1

x = 2 (v 0 < x < 4)
Vy M l trung im AB .
Cu 5
(1,0 im)

Cho a, b 0 v a + b 2. Chng minh :

0,25

0,25
0,25
0,25

2 a 1 2b 8

1 a 1 2b 7

Bt ng thc cn chng minh tng ng vi:

1
2
8

1 a 1 2b 7

1
1
1

2
1
2
1

Ta c:
= a 1 b
1 (1) (bt Csi)
(a 1)(b )
a 1 2b 1
2
2
1
a 1 b
1
2 7 (bt C si)
(a 1)(b )
2
2
4
2
8

7 (2)

1
(a 1)(b )
2
1
2
8

T (1) v (2) suy ra:


1 a 1 2b 7
1
3
5
Du = xy ra ch khi : a + 1 = b + v a + b = 2 a =
v b =
2
4
4

0,25
0,25

0,25

0,25

173

174

CHNH THC

175

176

177

178

S GIO DC O TO

K THI TUYN SINH VO LP 10 THPT

VNH LONG

NM HC 2012 2013
Mn thi : TON
Thi gian lm bi : 120 pht, khng k thi gian giao

CHNH THC

Cu 1: (2,5 im) Gii phng trnh v h phng trnh:


a) 2x 1 = 3
b) x 2 12 x 35 0
2 x 3 y 13
c)
3x y 9
Cu 2: (2,5 im)
a) V ng thng (d): y = 2x 1
b) Chng minh rng ng thng (d) tip xc vi parabol (P): y = x2
c) Tm a v b ng thng (d): y = ax + b song song vi ng thng (d) v i qua im M(0; 2).
Cu 3: (1,0 im)
Tm tham, s thc m phng trnh x 2 2mx + m 1 = 0 c mt nghim bng 0. Tnh nghim cn
li.
Cu 4: (1,0 im)

a a
a a
1

Rt gn biu thc: A 1

, vi a 0, a 1

a 1
a 1

Cu 5: (2 im)
Cho tam gic ABC c ba gc nhn ni tip trong ng trn tm O. Gi AH v BK ln lt l cc
ng cao ca tam gic ABC.
a) Chng minh t gic AKHB ni tip ng trn. Xc nh tm ca ng trn ny

b) Gi (d) l tip tuyn vi ng trn (O) ti C. Chng minh rng ABH


v HK OC .
HKC
Cu 6: (1 im)
Tnh din tch xung quanh v th tch ca mt hnh nn c ng knh ng trn y d = 24 (cm) v
di ng sinh l 20 (cm).

179

S GIO DC V O TO
TNH B RA-VNG TU
CHNH THC

K THI TUYN SINH VO LP 10 THPT


Nm hoc 2012 2013
MN THI: TON
Ngy thi: 05 thng 7 nm 2012
(Thi gian lm bi: 120 pht, khng k thi gian giao )

Bi 1: (3,0 im)
a) Rt gn biu thc:
b) Gii phng trnh:

A = 5 3 2 48 300
x2 + 8x 9 = 0
x y 21
c) Gii h phng trnh:
2x y 9
1 2
1
x v ng thng (d): y = x + 2
4
2
a) V (P) v (d) trn cng mt mt phng ta
b) Tm ta giao im ca (P) v (d) bng php tnh.

Bi 2: (1,5 im) Cho parabol (P): y =

Bi 3: (1,5 im)
Hai i cng nhn cng lm mt cng vic. Nu hai i lm chung th hon thnh sau 12 ngy. Nu
mi i lm ring th di mt s hon thnh cng vic nhanh hn i hai l 7 ngy. Hi nu lm ring th mi
i phi lm trong bao nhiu ngy hon thnh cng vic ?
Bi 4: (3,5 im)
Cho ng trn (O) ng knh AB. V tip tuyn Ax vi ng trn (O). Trn Ax ly im M sao
cho AM > AB, MB ct (O) ti N (N khc B). Qua trung im P ca on AM, dng ng thng vung gc
vi AM ct BM ti Q.
a) Chng minh t gic APQN ni tip ng trn.
b) Gi C l im trn cung ln NB ca ng trn (O) (C khc N v C khc B).

Chng minh: BCN


OQN
c) Chng minh PN l tip tuyn ca ng trn (O).
d) Gi s ng trn ni tip ANP c di ng knh bng di on OA.
AM
Tnh gi tr ca
AB
Bi 5: (0,5 im)
2
2
Cho phng trnh x 2 m 1 x m m 1 0 (m l tham s). Khi phng trnh trn c nghim

x1 , x2 , tm gi tr nh nht ca biu thc:

M x1 1 x2 1 m
2

180

p n bi hnh

a) T gic APQN c APQ


ANQ
90o APQ
ANQ
180o
b) Ta c PA = PM v PQ AM QM = QB OQ // AM OQ AB

(cng ph vi ABN
)
OQN
NAB

)
(cng chn NB
BCN
NAB

BCN
OQN

c) Cch 1: OQN
t gic AONQ ni tip.
NAB
Kt hp cu a suy ra 5 im A, O, N, Q, P cng nm trn mt ng trn

ONP
OAP
90o ON NP NP l tip tuyn ca (O)

Cch 2: PAN
(do PAN cn ti P)
PNA

(do ONB cn ti O)
ONB
OBN

Nhng PAN
(cng ph vi NAB
)
OBN

PNA
ONB

M ONB
ONA
90o PNA
ONA
90o PNO
ON PN NP l tip tuyn ca (O)
d) Gi I l giao im ca PO v (O), suy ra I l tm ng trn ni tip tam gic APN

OE EI

R
3
(R l bn knh ng trn (O)) AIE u AE R
2
2

AEO

R 3
AE EO
2PA MA AE
PAO (g-g)

2 3
R
PA AO
2AO AB EO
2

181

S GIO DC V O TO
TNH HU GIANG
CHNH THC

THI TUYN SINH LP 10 THPT


NM HC 2012 2013
MN: TON
Thi gian lm bi: 120 pht (khng k thi gian giao )

thi c 01 trang
Bi 1: (0,5 im) Rt gn biu thc: A

3 6 2 8

1 2
1 2

Bi 2: (1,5 im) Khng s dng my tnh cm tay, hy gii phng trnh v h phng trnh sau:
a) x 2 x 20 0

b)

x 2y 5

2x y 1

Bi 3: (2,0 im)
a) V th (P) ca hm s: y = -2x2
b) Tm to cc giao im ca (P) v ng thng (D): y = x 1 bng php tnh.
Bi 4: (2,0 im) Cho phng trnh x 2 2 m 1 x m 3 0 (m l tham s)
a) Chng minh phng trnh lun c 2 nghim phn bit.
2
2
b) Gi hai nghim ca phng trnh l x1 , x2 . Xc nh m gi tr ca biu thc A x1 x2 nh
nht
Bi 5: (4,0 im) Cho ng trn (O; R) v mt im S bn ngoi ng trn v hai tip tuyn SA, SB v
ng thng a i qua S ct ng trn (O; R) ti M, N vi M nm gia S v N (ng thng a khng i qua
tm O).
a) Chng minh SO AB
b) Gi I l trung im ca MN v H l giao im ca SO v AB; hai ng thng OI v AB ct nhau ti
E. Chng minh: OI.OE = R2
c) Chng minh t gic SHIE ni tip ng trn
d) Cho SO = 2R v MN = R 3 . Tnh din tch tam gic ESM theo R

182

S GIO DC V O TO
BN TRE

CHNH THC

THI TUYN SINH 10


TRUNG HC PH THNG CHUYN BN TRE NM HC 2012 2013
MN TON (chung)
Thi gian 120 pht (khng k pht )
Cu 1 (2,0 im). Khng dng my tnh b ti, hy rt gn cc biu thc sau:

2
3

6 5
6 3
5 3
2x x
x 1
x x 1

b) B =
, (vi x > 0)
x
x 1 x x 1
a) A =

Cu 2 (2,5 im). Gii phng trnh v h phng trnh sau:

a) x 2 x 1 3 x 2 x 1 4 0

2 6
x y 11

b)
4 9 1
x y
Cu 3 (2,5 im).
a) Chng minh rng phng trnh x 2 2mx 3m 8 0 lun c hai nghim phn bit x1; x2 vi mi
m. Vi gi tr no ca m th hai nghim x1; x2 tha mn x1 2 x 2 2 0
b) Cho x, y, z l ba s thc dng tha: x 2 y 2 z 2 1 . Chng minh rng:

1
1
1
x 3 y3 z 3

3
x 2 y2 y2 z2 z2 x 2
2xyz
ng thc xy ra khi no?
Cu 4 (3,0 im). Cho na ng trn tm O ng knh AB. T A, B v cc tip tuyn Ax, By v pha c
cha na ng trn (O). Ly im M thuc on thng OA; im N thuc na ng trn (O). ng trn
(O) ngoi tip tam gic AMN ct Ax ti C; ng thng CN ct By ti D.
a) Chng minh t gic BMND ni tip.
b) Chng minh DM l tip tuyn ca ng trn (O).
3/ Gi I l giao im ca AN v CM; K l giao im ca BN v DM. Chng minh IK song song AB.

183

S GIO DC V O TO
BN TRE

THI TUYN SINH 10


TRUNG HC PH THNG CHUYN BN TRE
NM HC 2012 2013
MN TON CHUYN
Thi gian 120 pht (khng k pht )

CHNH THC

Bi 1: (3 im)
Cho biu thc

x 8

x 2 :

x x 8

x
x2 x 4

2 x

vi x 0

1/ Rt gn biu thc A.

8
x . Tm x biu thc B t gi tr nh nht
x6A

2/ t B
Bi 2:

Gii cc phng trnh v h phng trnh sau


1/ 2x 2 8x x 2 4x 16 4

2
3
2/ 3 x 2 10 x 1

2x y xy 13

1
1
15 x 1 y 2 2

3/
Bi 3:

1/ Xc nh tt c cc gi tr ca m phng trnh x 2 2x 2m 5 0 c hai nghim phn bit


x1; x2 . Vi gi tr no ca m th hai nghim x1; x2 tha iu kin x1 mx 2 x 2 mx1 10
2/ Cho ba s thc dng a, b, c. Chng minh rng

a2
b2
c2
abc

b 3c c 3a a 3b
4

Bi 4:

Cho tam gic ABC nhn, v ng cao AH. Gi E, F ln lt l hnh chiu ca H ln hai cnh AB,
AC. ng thng qua A vung gc vi EF ct cnh BC ti D.
1/ Chng minh ng thng AD i qua tm ng trn ngoi tip ca tam gic ABC.
2/ Gi I, K ln lt l hnh chiu ca D ln hai cnh AB, AC. Chng minh tam gic DIK ng dng
vi tam gic HEF.
3/ Chng minh

BH BD AB2
.

CD CH AC 2

184

185

186

187

188

189

S GIO DC & O TO
AN GIANG
-------------- CHNH THC

THI TUYN SINH VO LP 10


Nm hoc 2012-2013
-------------------Mn: TON
Kha ngy 11 -7 -2012
Thi gian lm bi : 120 pht
(Khng k thi gian pht )
Ngy thi: 12-7-2012

SBDPHNG..

Bi 1. (2,5 im)
a) Rt gn A = 2 16 - 6 9 36
b) Gii phng trnh bc hai : x2 2 2 x +1 = 0
3x y 7
c) Gii h phng trnh :
2x y 3
Bi 2. (2,0 im)
Cho hm s y = x + 1 (*) c th l ng thng ( d )
a) Tm h s gc v v th hm s (*)
b) Tm a (P): y = ax2 i qua im M (1 ;2).Xc nh ta giao im ca ng thng (d) v Parabol (P)
vi a va tm c .
Bi 3. (2,0 im)
Cho phng trnh x2 2 (m+1) x + m2 + 3 = 0
a) Vi gi tr no ca m th phng trnh c hai nghim phn bit.
b) Tm m phng trnh c hai nghim tha tch hai nghim khng ln hn tng hai nghim.
Bi 4. (3,5 im)
Cho ng trn ( O) bn knh R = 3 cm v mt im I nm ngoi ng trn, bit rng OI =
4cm.T I k hai tip tuyn IA v IB vi ng trn (A,B l tip im).
a) Chng minh t gic OAIB ni tip.
b)T I k ng thng vung gc vi OI ct tia OA ti O.Tnh OO v din tch tam gic IOO .

c) T O k OC vung gc BI ct ng thng BI ti C.Chng minh OI l tia phn gic ca AO'C


------ Ht------

190

191

192

193

194

195

196

197

198

199

200

201

202

203

You might also like